You are on page 1of 353

Symbiosis Law

Admission Test (SLAT)

10 Free Mock Tests


TABLE OF CONTENTS
S.No. Title Page No.
1 About This eBook 4
2 Exam Structure And Exam Pattern 5
3 SLAT Syllabus 6
4 Section-wise Preparation Strategy 7
5 How To Attempt The Mock Test Strategy 16
6 Mock Test 1 18
Answer Key 33
Solutions 34
7 Mock Test 2 50
Answer Key 66
Solutions 67
8 Mock Test 3 81
Answer Key 98
Solutions 99
9 Mock Test 4 115
Answer Key 133
Solutions 134
10 Mock Test 5 149
Answer Key 166
Solutions 167
11 Mock Test 6 183
Answer Key 200
Solutions 201
12 Mock Test 7 217
Answer Key 234
Solutions 235

www.careers360.com 2
13 Mock Test 8 252
Answer Key 269
Solutions 270
14 Mock Test 9 286

Answer Key 302

Solutions 303

15 Mock Test 10 318

Answer Key 335

Solutions 336

16 Other Useful Resources 352

CUET Legal Studies Concepts

MH CET Law (5 Years LLB)

MH CET Law (3 Years LLB)

CLAT Free Sample Paper

CLAT Mock Test with Solutions eBook

CLAT Legal Reasoning Previous Year Question Papers with Detailed Solutions

CLAT English Practice Questions with Detailed Solutions ( 50 Sets)

100+ Quantitative Techniques Practice Questions

Criminal Law Concepts and Practice Questions

www.careers360.com 3
ABOUT THIS eBOOK
Dear SLAT Aspirants,
We are thrilled to introduce our comprehensive ebook designed specifically for this exam.

What you will find in this Ebook:


Exam Pattern and Structure
Examine the details of the SLAT by breaking down its pattern and structure.

Mock Test Strategies:


Navigate through the mock test series with confidence using our expert strategies. Learn how to tackle
questions effectively, manage time efficiently, and enhance your overall performance.

Preparation Strategy
Embark on a systematic and proven preparation journey with our comprehensive strategy guide. Gain
insights into how to structure your study plan, focus on key topics, and optimize your learning for max-
imum impact.

10 Mock Test Series:


Elevate your preparation with our specially curated 10 mock test series, each comprising 5 sections and
60 sets of questions. Simulate the actual exam environment to enhance your readiness and identify
areas for improvement.

Answer Key with Detailed Solutions:


After attempting each mock test, check the comprehensive answer key and solutions provided. Under-
stand the rationale behind each answer and reinforce your understanding of critical legal concepts.

www.careers360.com Back to Index 4


EXAM STRUCTURE AND
EXAM PATTERN
EXAM STRUCTURE

Particulars Details
Mode of exam Online

Type of questions Multiple Choice Questions

Time duration 60 minutes

Total number of questions 60

Maximum marks 60

Negative marking No

EXAM PATTERN

Subject Total Questions Total Marks

Legal Reasoning 12 12

Logical Reasoning 12 12

Analytical Reasoning 12 12

Reading Comprehension 12 12

General Knowledge 12 12

Total 60 60

www.careers360.com Back to Index 5


SLAT SYLLABUS
Section List Of Topics
General Knowledge International events, UN bodies, Major developments in the areas of
sports, geopolitics and important environmental agreements, Cur-
rent affairs, Eminent personalities, Sports, Awards and honours

Legal Reasoning Legal terms, Indian Constitution, its structure, sections and sched-
ules, Contracts and Constitutional Law, Torts, Vicarious Liability,
Contracts and Constitutional Law, IPC and the CrPC, Strict Liability,
Law of Crimes, International Law, Intellectual Property Rights, Legal
Awareness covering Constitutional Law and Polity

Logical Reasoning Relationships, Analogies, Logical Sequences, Syllogism Statement &


Assumption, Cause & Effect Inferences, Premises & Conclusion

Analytical Reasoning Mental Ability and Logical/Mathematical Analysis, Deductive Rea-


soning, Algebra, Percentage, Profit and Loss, Odd-Even Discretion,
Logical Fallacy, Analogy, Syllogism, Agreement and Disagreement

Reading Comprehension Vocabulary - Antonyms & Synonyms, Spelling correction, Grammar


- Tense, Part of speech, Idioms and Phrases, Analogies, One-word
Substitution, Sentence Improvement and rearrangement, Errors
detection - Spotting errors, Inappropriate usage of words, Spelling
mistakes, English Comprehension

www.careers360.com Back to Index 6


LOGICAL REASONING

Enhancing your logical reasoning skills involves refining your ability to draw conclusions based on evi-
dence and avoiding personal biases. Here are several strategies to improve your logical reasoning:

Practice Conditional Statements:


Engage in recognizing and applying conditional statements, which are truths dependent on
specific conditions. For instance, observe how certain actions or events lead to predictable
outcomes.

Filter Out Biases:


Strive for objectivity by identifying and minimizing personal biases. Train yourself to make
decisions based on factual information rather than subjective feelings, ensuring a more
logical approach.

Play Logic-based Games:


Challenge yourself with games that require logical thinking, such as chess, card games, or
strategy board games. These activities not only enhance logical reasoning but also foster
attention to detail and decision-making skills.

Note Patterns in Daily Life:


Actively seek and analyze patterns in your personal and professional life. Take note of re-
curring events and consider their implications. Recognizing patterns is crucial for predicting
future behavior and making informed decisions.

Monitor for Misleading Conclusions:


Be vigilant about avoiding a false notion —faulty reasoning that undermines the validity of
an argument. Familiarize yourself with common false or misleading notions and ensure that
your conclusions are based on sound logic.

www.careers360.com Back to Index 7


Make Observations:
Cultivate focus and thoughtfulness by actively observing your surroundings. Pay attention
to details, anomalies, and patterns. Reflect on your observations, consider their significance,
and think backward to understand causation.

Take Logical Reasoning Tests:


Consider taking logical reasoning tests. These tests, such as diagrammatic or verbal logical
reasoning tests, assess your ability to identify patterns, interpret rules, and draw conclu-
sions based on evidence.

www.careers360.com Back to Index 8


LEGAL REASONING

Study Relevant Study Material:

Gather study materials specific to the legal reasoning section. Look for books or resources that
cover the legal concepts tested in the exam. In this eBook, we have covered all the chapters and
concepts that can come in the exam.

Legal Concepts and Basics:

Focus on understanding fundamental legal concepts and principles. Pay attention to topics such
as Contracts, Torts, Constitutional law, and Lehal Maxims

Analytical and Critical Thinking:

Legal reasoning often involves analytical and critical thinking. Practice solving logical puzzles,
critical reasoning questions, and scenarios to enhance these skills.

Legal Vocabulary:

Build a strong foundation in legal vocabulary. Familiarize yourself with legal terms, phrases, and
terminology commonly used.

Stay Informed about Legal Developments:

Stay updated on current legal affairs and developments. This knowledge can be beneficial for
questions related to recent legal events.

Review Legal Maxims and Case Laws:

Familiarize yourself with important legal maxims and landmark case laws, as they may be rele-
vant to legal reasoning questions.

www.careers360.com Back to Index 9


Some quick tips are as follows:

Gain a Quick Overview:


Explore headings, subheadings, summaries, and
introductions to form a mental framework for the
topics you’re about to learn.

Set Clear Learning Goals:


Define specific objectives for your study session.
This helps maintain focus and motivation. Decide if
you aim to grasp fundamental principles, understand
main ideas, or apply concepts to problem-solving.

Take Organized Notes:


Actively take structured notes during your study ses-
sion. Utilize headings, bullet points, and diagrams to
organize information. Summarize key themes in your
own words to enhance understanding and retention.
Remember, Regular practice, revision, and staying
informed about legal updates will contribute to your
success in the CUCET legal reasoning section.

www.careers360.com Back to Index 10


ANALYTICAL REASONING

Emphasize Practice:
Achieving proficiency in math requires more than just reading or listening. Actively engage
in problem-solving to enhance your skills. The more problems you solve, the better equipped
you’ll be. Practice different problem-solving approaches to build versatility.

Learn from Mistakes:


When practicing, thoroughly analyze your solutions, especially if you make mistakes. Under-
stand the shortcomings in your problem-solving approach. Identifying errors helps strength-
en your skills and prevents repeating the same mistakes.

Grasp Key Concepts:


Avoid memorizing processes; instead, focus on understanding the underlying logic. Math-
ematics builds on sequential concepts, so ensure a solid grasp of fundamental principles
before tackling more complex problems. Understanding the “why” is crucial for long-term
success.

Address Doubts:
When stuck on a part of a problem, resist the urge to skip it. Take the time to comprehend
the solution process. Once you understand the initial problem, use it as a foundation to prog-
ress through the entire question. Math mastery requires time, patience, and a willingness to
confront challenges.

Optimize Your Study Environment:


Create a distraction-free study environment for math, a subject demanding high concentra-
tion. Background music, chosen carefully for a relaxing atmosphere, can aid focus. Elimi-
nate distractions to enhance your ability to solve complex problems in geometry, algebra,
or trigonometry.

www.careers360.com Back to Index 11


Build a Mathematical Dictionary:
Compile notes, flashcards, or cheat sheets containing essential math concepts, terminolo-
gy, and definitions. Include meanings, key points, and sample answers for quick reference.
This resource will serve as a handy tool for reviewing and reinforcing your understanding.

Apply Math to Real-World Problems:


Relate mathematical concepts to real-world scenarios to make them more tangible. Apply-
ing math to practical situations can alter your perspective and enhance comprehension.
For instance, use probability to assess risks in everyday decisions, such as buying a lottery
ticket.

www.careers360.com Back to Index 12


READING COMPREHENSION
Here are some tips you can follow while studying for the English section:

Active Reading: Read passages actively, engaging with the content rather than passively
scanning. Highlight key points and take notes.

Identify Main Ideas: Focus on identifying the main idea of each paragraph and the overall
theme of the passage.

Practice Skimming and Scanning: Develop skimming (quickly looking through the text for
the main ideas) and scanning (searching for specific information) skills.

Underline Keywords: Underline or highlight keywords and phrases that convey the central
meaning of the passage.

Answering Techniques: Practice different question types such as inference, main idea,
tone, and detail-oriented questions.

Flashcards for Identification can be used for parts of speech : Create flashcards for each
part of speech with examples. Test yourself regularly.

Sentence Structure: Practice structuring of the sentences to visually understand the struc-
ture. This can be particularly useful for complex sentences.

Identify Subject-Verb Agreement: Pay attention to subject-verb agreement and practice


identifying errors in sentences.

Error Spotting Exercises: Work on error spotting exercises to identify common mistakes in
sentences.

Create Word Maps: Create word maps connecting roots, prefixes, and suffixes to their
meanings. This will help you understand the origins of words.

Memorize Common Prefixes and Suffixes: Memorize common prefixes and suffixes to
decode unfamiliar words.

www.careers360.com Back to Index 13


Grouping Words: Group synonyms and antonyms together for related sets. Practice them
in pairs to reinforce your understanding.

Contextual Usage: Practice using synonyms and antonyms in sentences to understand


their contextual usage.

Simulate Exam Conditions: Take full-length mock tests to simulate the exam environment,
including time constraints.

Review Mistakes: After each mock test, thoroughly review your mistakes. Understand why
you got a question wrong and how to avoid similar errors in the future.

Time Management: Practice managing your time effectively during the full-length mock
tests. Allocate time for each section based on its weightage.

www.careers360.com Back to Index 14


PREPARATION TIPS
FOR GK SECTION

Know the syllabus: Familiarise yourself with the MH CET Law syllabus to understand

1 the topics that may be covered in the GK section. This will help you organise your
study plan.

Read newspapers and magazines: Develop the habit of routinely reading newspapers
like The Hindu, The Economic Times, The Indian Express, etc and periodicals to be
informed about current affairs and significant occurrences. Concentrate on politics,
Economics, Sports, Science, and Technology, as well as local and global news.
2
Follow reliable websites: Follow reliable news websites and portals to get access

3 to the latest information. Websites like GKtoday, The Hindu, The Indian Express and
several others like these are popular for their comprehensive news coverage.

Watch news channels: Watch news channels regularly to understand current affairs
and important issues. This will also help you improve your knowledge about national
and international events.
4
Solve previous years’ papers: To comprehend the test format and the kinds of ques-

5 tions asked in the GK portion, solve sample papers. You can use this to find your
areas of weakness and strengthen them.

Take online quizzes and mock tests: Take online quizzes and mock tests specifically
designed to assess your knowledge and improve your speed and accuracy. This will
also help you get accustomed to the exam environment.
6
Use online resources: Utilise online resources like GK websites, PDF study materials,

7 and apps to enhance your GK knowledge. Websites like GKToday, Jagran Josh, and
AffairsCloud provide daily updates and study materials for GK preparation.

www.careers360.com Back to Index 15


HOW TO ATTEMPT THE MOCK TEST STRATEGY
Why analyzing a mock test is essential?

To maximize your To prevent


To evaluate your strengths and To formulate an repeating the
current level of minimize your appropriate same mistakes.
preparation. weaknesses. strategy.

Analyse
the Scorecard

Identify your
Attempt Preparation Strengths and
the Mock Test Strategy Overview Weaknesses

Revise
the Concept

www.careers360.com Back to Index 16


Mock Test Analysis
Can be done in three ways

Section-wise
Analysis

•I •I •
I
Familiarise the Exam Pattern: BDes exam Decide the order in which you should attempt Concept Revision:Based on the mock test
contains 3 parts out of which GAT consists of the questions: Start with your strong areas, results, revisit the concepts related to the
30% weightage.GAT consists of 5 sections that easy questions, less time-consuming tasks, questions you found challenging. Practice
are English Comprehension Communication and move to your weak areas, difficult problems and exercises related to those
Ability ,Quantitative Ability , Analytical Ability questions, and more time-consuming ones. concepts to reinforce your understanding.

I

,General Knowledge And Current Affairs and
hence, Consistency is Key: Make a study schedule


Determine the time you should allocate to
each question type: With an efficient time and stick to it. Regularly take mock tests and
Start with the Easiest Section: Begin your management plan, you can save time to tackle review your progress. Consistency is vital for
long-term success.


mock test series with the section that you find more challenging questions later. This strategy
the easiest. This will help you build confidence can result in a significantly improved score.


and warm up for more challenging sections.A Take Breaks: It's essential to give your mind
negative attitude can hinder your performance, some rest. Consider taking a break of 2-3 days
while a positive one can boost your score. Questions answered correctly: Many students
before taking another mock test. This will help
Therefore, beginning with an easier section will tend to neglect their strong areas, but there is a
prevent burnout and allow you to approach
bolster your confidence and help you score well better way to leverage your strengths. Check if
each test with a fresh mindset.
in the exam. the solution technique used for a correct


answer was the same as yours or more
time-efficient. Did you spend more time on Stay Positive and Confident: Confidence is a
Time Management:Practice completing the these questions than necessary? significant factor in performing well in any


test within the stipulated time frame. Time exam. Believe in your preparation and stay
management is critical, and you should aim to positive. Don't get discouraged by low scores in
answer all the questions within the allocated Questions answered incorrectly: Compare some mock tests; instead, use them as
time. This will help you get accustomed to the your solution with the provided one. Identify opportunities for improvement.
test's pace. what went wrong in your approach. If the

I

solution technique is the same, it indicates a

I
lack of accuracy, meaning you need to work on Accuracy Over Speed: While taking the mock
Unattempted questions: Initially, try to solve your calculation techniques. If the solution is test, prioritize accuracy over speed. It's better to
these questions on your own. Verify if your different, learn and practice it regularly. answer fewer questions correctly than rush


answer is correct or not. Then, review the

I
through and make mistakes. Mark the
provided solution and adjust your strategy questions you're unsure about for review.
accordingly. Unattempted questions: Initially, try to solve
these questions on your own. Verify if your
answer is correct or not. Then, review the
provided solution and adjust your strategy
accordingly.


I
Accuracy Over Speed: While taking the mock
test, prioritize accuracy over speed. It's better to
answer fewer questions correctly than rush
through and make mistakes. Mark the
questions you're unsure about for review.

www.careers360.com Back to Index 17


MOCK TEST - 1
______________________________________________________________________

LOGICAL REASONING

Q1.Which of the following is a cause and effect relationship?

A) The increase in temperature leads to the melting of ice.


B) Ice cream is a delicious dessert.
C) Dogs bark loudly at strangers.
D) All of the above

Q2. Identify the premise in the following argument: "If it is raining, then the
streets will be wet. The streets are wet. Therefore, it must be raining."

A) If it is raining, then the streets will be wet.


B) The streets are wet.
C) Therefore, it must be raining.
D) None of the above

Q3.What is the conclusion of the following argument? "All mammals have hair.
Whales are mammals. Therefore, whales have hair."

A) All mammals have hair.


B) Whales are mammals.
C) Therefore, whales have hair.
D) None of the above

Q4.Which of the following statements is a premise?

A) Birds fly because they have wings.


B) It is sunny today.
C) The cat meowed loudly.
D) None of the above

www.careers360.com Answer 18
Q5.Which of the following best describes a cause and effect relationship?

A) The sky is blue.


B) Plants grow when they receive sunlight.
C) Dogs bark when they see other dogs.
D) All of the above

Q6. Direction: What do you infer from the following set of statements?
Statements:
I. I have observed many plants' roots.
II. All these plants' roots were found to grow downward.

A) I am a botanist.
B) Some plants' roots grow downward.
C) A plant's root grows downward.
D) Most of the plants' roots grow downward.

Q7..Direction: In the following question, one / two statements are given, each
followed by two conclusions, I and II. You have to consider the statement to be
true, even if it seems to be at variance from commonly known facts. You have to
decide which of the given conclusions, if any, follows from the given statements.
Statement:
Economic security makes people better and happier and has a good influence on
their personality.
Conclusions:
I. People who earn enough money are happier.
II. To have a good personality people should be economically sound.

A) Only I follows
B) Both I and II follow
C) Only II follows
D) Neither I nor II follows

Q8. Direction: Maya said, My mother is the sister of Ranjeet's brother. What is
Ranjeet's relationship with Maya?

A) Cousin
B) Maternal uncle
C) Uncle
D) Brother-in-law

www.careers360.com Answer 19
Q9.Direction: Abhishek is Byomkesh's brother, and Vimal is Jackie's father. Ella is
Byomkesh's mother. Abhishek and Jackie are brothers. How is Ella related to
Vimal?

A) Wife
B) Daughter
C) Mother
D) Sister

Q10.Direction: There are five girls—R, S, T, P, and Q—sitting in a row facing north.
T is sitting exactly in the middle of the row. Q is sitting to the immediate right and
immediate left of P and T, respectively. S is not sitting at the extreme end. Who is
sitting third to the left of R?

A) P
B) Q
C) S
D) T

Q11.Direction: Five girls are sitting in a row. Sudha is sitting next to Padma.
Krishna is sitting next to Rama who is sitting on the extreme left. Tapti is sitting
on the extreme right. Nobody is sitting between Padma and Krishna. Who is
sitting in the middle?

A) Krishna
B) Padma
C) Sudha
D) Tapti

Q12.Direction: Which one of the given responses will be a meaningful order of the
following?
1. Cutting 2. Dish 3. Vegetable 4. Market 5. Cooking

A) 5, 3, 2, 1, 4
B) 3, 2, 5, 1, 4
C) 4, 3, 1, 5, 2
D) 1, 2, 4, 5, 3

www.careers360.com Answer 20
LEGAL REASONING

Q13.Situation: A person, in a carefully planned scheme, administers a lethal


poison to another individual resulting in death.

A) Section 299 - Culpable Homicide not amounting to murder


B) Section 304 - Punishment for culpable homicide not amounting to murder
C) Section 302 - Punishment for murder
D) Section 304A - Causing death by negligence

Q14:Situation: During a bar brawl, an individual, in a fit of anger, hits another


person on the head with a heavy glass bottle, leading to the victim's death.

A) Section 299 - Culpable Homicide not amounting to murder


B) Section 304 Part II - Culpable homicide not amounting to murder
C) Section 302 - Punishment for murder
D) Section 304A - Causing death by negligence
Q 15 Situation: An individual is acquitted of theft charges in a lower court. Later,
new evidence emerges proving their guilt. Can they be retried for the same theft
charges?

A) Yes
B) No
C) Only if the evidence is strong
D) Only if the initial trial was unfair

Q16.Situation: A person is charged with assault and is convicted in a district


court. Later, the same person is charged for the same incident under a different
section of the law. Can they be tried again for the same act?

A) Yes, if the second charge is under a different section of the law


B) Yes, if the initial conviction was not severe
C) No, under the principle of double jeopardy
D) Only if the victim requests a retrial

www.careers360.com Answer 21
Q17.Situation: A person believes their fundamental rights are violated, and they
want to challenge it in court. Which court in India would they approach first?

A) High Court
B) District Court
C) Supreme Court
D) Magistrate Court

Q18:Situation: A dispute involving a substantial amount of money arises between


two parties. They want to appeal the decision of a lower court. Which court would
they typically approach for the appeal in the hierarchy of Indian judiciary?

A) District Court
B) High Court
C) Supreme Court
D) Sessions Court

Q19: Situation: A contracts with B to sell a car, and B later sells the car to C
without informing A. A wants to enforce the original contract against C. Can A do
so?

A) Yes, because C is now in possession of the car.


B) No, because C was not a party to the original contract.
C) Only if C agrees to be bound by the original contract.
D) Yes, but only if A seeks permission from B.

Q20:Situation: A software company contracts with a client to develop a custom


software solution. The client wants to sue the software developer's subcontractor
for delays in delivery. Can the client do so?

A. Yes, because the subcontractor is working on the same project.


B. No, because the subcontractor is not in privity of contract with the client.
C. Only if the client and subcontractor have a separate agreement.
D. Yes, but only if the software company approves.

www.careers360.com Answer 22
Q21 A construction company entered into a contract to build a commercial
complex. Due to unforeseen government regulations, construction was halted,
making it impossible to complete the project. Which doctrine would likely apply in
this situation?

A) Doctrine of Mistake
B) Doctrine of Frustration
C) Doctrine of Duress
D) Doctrine of Impossibility
Q22: A couple with two children has decided to divorce. They both want custody
of the children, and the court is involved in determining the arrangements. The
mother has been the primary caregiver throughout the children's lives, but the
father argues that he can provide a stable environment. What principle is most
likely to influence the court's decision?

A) Best Interests of the Children


B) Maternal Preference Doctrine
C) Father's Right Doctrine
D) Equal Shared Parenting

Q23: In a jurisdiction where marital property is subject to equitable distribution, a


couple is getting divorced. One spouse argues that they should receive a more
significant share of the assets because they sacrificed their career to support the
other spouse's professional advancement. Which legal principle is most likely to
be applied in this situation?

A) Community Property Rule


B) Principle of Equal Division
C) Reimbursement Alimony
D) Spousal Support Formula

Q24: A couple entered into a prenuptial agreement specifying the division of


assets in case of divorce. However, one spouse claims that the agreement is
unfair and should be set aside. What legal ground might the court consider for
invalidating the prenuptial agreement?

A) Unconscionability
B) Retroactive Modification
C) Equitable Distribution

www.careers360.com Answer 23
D) Community Property Presumption

ANALYTICAL REASONING

Q25.Direction: Which one set of letters when sequentially placed at the gaps in
the given letter series shall complete it - pp_ _qp_pqq_pqq_ _qq_q

A) qqpqpqp
B) ppppqpq
C) qqpqpqp
D) qqqpppq

Q26. Convert 25 m/s to km/h.

A] 90 km/h
B] 40 km/h
C] 50 km/h
D] 30 km/h

Q27. If the speed of a boat in still water is 8 km/h and the rate of stream is 4 km/h,
find upstream speed OF boat
A] 4 km/h
B] 5 km/h
C] 3 km/h
D] 10km/h

Q28. A can do a work in 10 days. B can do the same work in 15 days. In how many
days can the work be completed if A and B work together?

A] 6 days.
B] 4 days.
C] 8 days.
D] 10 days.

www.careers360.com Answer 24
Q29. If pipe A and B fills a tank individually in 10 and 15 hrs respectively, then in
how many hrs they will fill the tank together?

A] 6 hrs.
B] 4 hrs.
C] 8 hrs.
D] 10 hrs.

Q30..A room is 16 m long, 9 m wide and 3 m high. It has two doors, each of
dimensions (2 m X 1.5 m) and three windows, each of dimensions (1.6 m X 75
cm). Find the cost of distempering the walls of the room from inside at the rate of
Rs 50 per square metre.

A] 7120

B] 7020

C] 6025

D] 6075

Q31. The ratio of the ages of A and B is 9:5. After 8 years B would reach the
present age of A. Find the present ages of B and A respectively
A] 9, 5
B] 10, 18
C] 18, 10
D] 5, 9

Q32.Directions: If K means subtracted from, L means divided by, M means added


to and D means multiplied by, then

96 L 4 K 6 M 11 D 9 = ?

A) 117
B) 125
C) 120
D) 145

www.careers360.com Answer 25
Q33.Directions: In the following question, a series is given with one number
missing. Select the missing number from the given alternatives.
672, 666, 660, 654, ?
A) 648
B) 646
C) 650
D) 640

Q34.Directions: In the following question, which one set of letters, when


sequentially placed at the gaps in the given letter series, will complete it?
_ yzaa _ y _ xy _ aazy _
A)xxzzx
B) xxaza
C) xzxzx
D) aazzx

Q35.Directions: In the given figure, how many papers are also hard?

A) 14

B) 23

C) 37

D) 32

Q36.Directions: Among four books, Book 1 is twice as heavy as Book 2. Book 3's
weight is half of Book 2's weight. Book 4 is 60 grams more as compared to Book
2, but 60 grams less as compared to Book 1. Which book is the heaviest?

www.careers360.com Answer 26
A) Book 1
B) Book 2
C) Book 3
D) Book 4

READING COMPREHENSION

Q37. Select the option that expresses the given sentence in active voice.

I am not interested in this assignment.

A) This assignment has not interested me.

B) This assignment is not interesting me.

C) This assignment did not interest me.

D) This assignment does not interest me.

Q38.Select the option that expresses the given sentence in passive voice.

He likes people to flatter him.

A) He likes to be flattered by people.

B) He likes to have been flattered by people.

C) He likes people to be flattered by him.

D) He liked to be flattered by people.

Q39.Select the most appropriate meaning of the given idiom.


cut a sorry figure

A) render an apology

B) make a sculpture

C) create a poor impression

D) break a record

www.careers360.com Answer 27
Q40.Select the word which means the same as the group of words given.

an animal or person that eats a variety of food of both plant and animal origin.

A) Omnivore

B) Herbivore

C) Carnivore

D) Cannibal

Q41.Select the option that expresses the given sentence in passive voice.

India is exporting handicrafts to several countries.

A) Handicrafts were exported to several countries by India.

B) Handicrafts are exported to several countries by India.

C) Handicrafts have been exported to several countries by India.

D) Handicrafts are being exported to several countries by India.

Q42.Identify the segment that contains a grammatical error. If there is no error,


select 'No error'.

The most talented of these three boys are my friend's son.

A) of these three boys

B) No error

C) The most talented

D) are my friend's son

Q43.Identify the segment that contains a grammatical error.

As soon the gun shot was heard, people ran out in panic.

A) As soon the

B) in panic

www.careers360.com Answer 28
C) people ran out

D) gun shot was heard

Q44.Select the option that will improve the underlined part of the sentence. In
case no improvement is needed, select 'No Improvement'.

The little boy did many mischiefs in school.

A) performed many mischiefs

B) made much mischief

C) No Improvement

D) did much mischiefs

Q45.Identify the segment that contains a grammatical error.

He said that he had forgot to lock the door before leaving.

A) to lock the door

B) He said that

C) he had forgot

D) before leaving

Q46.Fill in the blank with the most appropriate word.

The annual meeting of our club has been ____________till next month.

A) dragged

B) deferred

C) disrupted

D) deranged

Q47.Select the most appropriate direct form of the given sentence.

Taru asked Kavya if she would help her in setting up her new dressing table

www.careers360.com Answer 29
A) Taru said to Kavya, “Will you help me in setting up her new dressing table?”

B) Taru said to Kavya, “Will you help me in setting up my new dressing table?”

C) Taru said to Kavya, “How would you help me in setting up my new dressing table?”

D) Taru said to Kavya, “Would you help her in setting up my new dressing table?”

Q48.Identify the segment that contains a grammatical error.

You must reach the airport on time either you will miss your flight.

A) either you will

B) miss your flight

C) the airport on time

D) You must reach

GENERAL KNOWLEDGE

Q49. Ahmedabad is situated along the bank of _____.

A. Sabarmati river
B. Narmada river
C. Banas river
D. Luni river

Q50. Govind Ballabh Pant Sagar reservoir is situated in

A. Chhattisgarh
B. Jharkhand
C. Uttarakhand
D. Uttar Pradesh

Q51.'Agronomy' is the practice of raising _________

A. plants and animals


B. crop plants
C. agriculture
D. fruit plants only

www.careers360.com Answer 30
Q52. Preservation of historical articles is done in science of

A. Iconography
B. Numismatic
C. Museology
D. Epigraphy

Q53. What is the primary purpose of clothing?

A. Fashion
B. Protection
C. Status symbol
D. Comfort

Q54. Which natural fiber is commonly used to make jeans?

A. Silk
B. Cotton
C. WoolL
D. Linen

Q55. In which year did the Kyoto Protocol come into force?

A) 2004
B) 2005
C) 2006
D) 2007

Q56. Limestone is a raw material used by which industry ?

A) Aluminum
B) Fertilizers
C) Cement
D) Petrochemicals

www.careers360.com Answer 31
Q57. 'Indica' was written by

A) Justin
B) Megasthenese
C) Pliny
D) Strabo

Q58. Who wrote the book--"The Test of My Life"?

A) Yuvraj Singh
B) Sachin Tendulkar
C) Brian Lara
D) Adam Gilchrist

Q59. Which institution is known as 'Soft Loan Window' of World Bank?

A) IDBI
B) IDA
C) IMF
D) RBI

Q60.Who presides over the Lok Sabha?

A) Speaker
B) President of India
C) Prime Minister of India
D) None of these

www.careers360.com Answer 32
ANSWER KEY
______________________________________________________________________

1 2 3 4 5 6 7 8 9 10
A B C A B C A B A C
11 12 13 14 15 16 17 18 19 20
B C C C C C A B B B
21 22 23 24 25 26 27 28 29 30
B A A A B A A A A B
31 32 33 34 35 36 37 38 39 40
B A A C B A D A C A
41 42 43 44 45 46 47 48 49 50
D D A B C B B A A D
51 52 53 54 55 56 57 58 59 60
B C B B A C B A B A

www.careers360.com Solutions 33
SOLUTIONS
______________________________________________________________________

LOGICAL REASONING
1-A
A cause and effect relationship indicates that one event (the cause) leads to another
event (the effect). In this case, the increase in temperature causes the ice to melt.

2-B
The premise is a statement that provides evidence or support for the conclusion. In this
argument, "The streets are wet" is the premise upon which the conclusion "Therefore, it
must be raining" is based.

3-C
The conclusion is the statement that follows logically from the premises. In this
argument, the conclusion is "Therefore, whales have hair," which is drawn from the
premises "All mammals have hair" and "Whales are mammals."

4-A
A premise is a statement that provides the basis for an argument or inference. In this
case, "Birds fly because they have wings" is a premise because it offers a reason or
explanation for why birds fly.

5-B
A cause and effect relationship indicates that one event leads to another. In this case,
the cause is sunlight, and the effect is the growth of plants.

6-C
According to the above-given statement,

First option: I am a botanist.


A botanist refers to a person who studies plants. From the above statement, it can be
said that anybody can observe the plants. It's not necessary to become a Botanist to
observe the plant.
Second option: Some plants' roots grow downward.
It's clear from the above statement that all plants' roots grow downwards, not some.
Hence, it's incorrect.
Third option: A plant's root grows downward.

www.careers360.com Back to Index 34


This option proves the above statement correct as it states that a plant's root grows
downward.
Fourth option: Most of the plants' roots grow downward.
This option states about most of the plants' roots but, according to the statement, it's
clear that all plants' roots grow downward.

Therefore, from the above, we can say that the third option satisfies the statement.
Hence, the third option is correct.

7-A
According to the given statement, let's analyze the conclusions
Conclusion I: People who earn enough money are happier.
Economic security refers to the ability of people to earn and fulfill their essential needs
in life which in turn makes people happier and this even influences their personality as
per the given statement.
Conclusion II: To have a good personality people should be economically sound.
According to the given statement, personality is dependent on people being happy and
fulfilling their basic needs. Being economically sound alone would not contribute to a
better personality.
Therefore, the correct answer is only the Conclusion I follow. Hence, the first option is
correct.

8-B
As per the instructions given, the family tree will be –

Now, we see that Maya's mother is the sister of Ranjeet's brother. So, Ranjeet's brother
is the maternal uncle of Maya. Hence, the second option is correct.

www.careers360.com Back to Index 35


9- A
Following the instructions of the question, the family tree will be –

Note: + denotes male and – denotes female.

So, from the above family tree, it is clear that Byomkesh, Abhishek, and Jackie are
siblings and Ella is the wife of Vimal. Hence, the first option is correct.

10-C
According to the given information, the family tree is as follows –

Here, the circular figure represents the female and the quadrilateral represents the male
in the figure.

So, from the above family tree, P is the uncle of N. Hence, the third option is correct.

www.careers360.com Back to Index 36


11-B
Given:
(i) T is sitting exactly in the middle of the row.

(ii) Q is sitting to the immediate right and immediate left of P and T, respectively. S is not
sitting at the extreme end.

From the final seating arrangement, Q is sitting third to the left of R.

Hence, the second option is correct.

12-C
Given:
1. Cutting 2. Dish 3. Vegetable 4. Market 5. Cooking

The stages of cooking a dish involve –


First, a person has to go to the market, then he will buy some vegetables from the
market. Then cut the vegetables properly, after cutting the vegetables, cook those
vegetables. When the cooking is finished, he will get a dish.
So, Market → Vegetable → Cutting → Cooking → Dish

The order is 4, 3, 1, 5, 2. Hence, the third option is correct.

LEGAL REASONING

13-C
Section 302 of IPC 1860 deals with murder. In this scenario, the intentional act of
administering a lethal poison to cause death fits the criteria for culpable homicide
amounting to murder.

www.careers360.com Back to Index 37


14-C
Section 302 of IPC 1860 applies when there is an intention to cause death. In this
scenario, the intentional act of hitting the person on the head with a glass bottle,
resulting in death, falls under culpable homicide amounting to murder.

15-C
The principle of double jeopardy prevents individuals from being retried for the same
offense once they have been acquitted, regardless of new evidence. Option B is correct.

16-C
The principle of double jeopardy prohibits an individual from being tried again for the
same offense. Even if the charge is under a different section of the law, the core incident
remains the same. Therefore, Option C is correct.

17-A
In India, individuals can approach the High Court for the violation of fundamental rights
as guaranteed by the Constitution. High Courts have the jurisdiction to hear writ
petitions related to fundamental rights. Therefore, Option A is correct.

18-B
In the hierarchy of the Indian judiciary, appeals from lower courts, except certain cases,
typically go to the High Court of the respective state or union territory. Therefore, Option
B is correct.

19-B
The doctrine of privity of contract states that only parties to the original contract can
enforce its terms. Since C was not a party to the original contract between A and B, A
cannot enforce the contract against C. Option B is correct.

20-B
The doctrine of privity of contract generally prevents a third party, in this case, the client,
from suing a subcontractor with whom they do not have a direct contractual relationship.
Option B is correct.

21-B
The doctrine of frustration applies when an unforeseen event occurs, making it
impossible to fulfill the contract's obligations. In this scenario, the unexpected
government regulations have frustrated the contract, as the construction company can
no longer complete the project as initially agreed upon. This is distinct from a mistake,

www.careers360.com Back to Index 38


duress, or general impossibility, making the doctrine of frustration the most relevant in
this case.

22-A
Family courts typically prioritize the best interests of the children when deciding custody
disputes. While the historical Maternal Preference Doctrine and Father's Right Doctrine
have been challenged, the modern approach focuses on ensuring the well-being and
stability of the children, considering factors such as their relationship with each parent,
living arrangements, and the ability to provide a nurturing environment.

23-A
In cases where one spouse has made significant career sacrifices for the benefit of the
other, reimbursement alimony may be awarded. This principle acknowledges the
financial sacrifice made during the marriage and aims to compensate the disadvantaged
spouse for their contributions, even if the division of assets is not strictly equal.

24-A
Courts may invalidate a prenuptial agreement if it is deemed unconscionable, meaning
it is excessively unfair or one-sided. To be enforceable, prenuptial agreements should
be entered into voluntarily, with full disclosure of assets, and should not be so one-sided
that enforcing them would lead to an unconscionable result.

ANALYTICAL REASONING
25-B
Given:
pp _ _ qp _ pqq _ pqq _ _ qq _ q

To fill the series we have to divide the series – pp_ _q / p_pqq / _pqq_ / _qq_q
Let's check the options –
First option: qqpqpqp; ppqqq / pppqq / qpqqp / qqqpq (No repeated pattern has been
found)
Second option: ppppqpq; ppppq / pppqq / ppqqq / pqqqq (The pattern in the series is
that p decreases by 1 and q increases by 1 when moving from one part to another.)
Third option: qqpqpqp; ppqqq / pppqq / qpqqp / qqqpq (No repeated pattern has been
found)
Fourth option: qqqpppq; ppqqq / pqpqq / ppqqp / pqqqq (No repeated pattern has been
found)
So, the series becomes → ppppqpppqqppqqqpqqqq. Hence, the second option is
correct.

www.careers360.com Back to Index 39


26-A

We know that
am/s = a×185 km/h
25 m/s = 25185 km/h = 90 km/h

27-A

Given that
Speed of boat = x = 8 km/h
Speed of stream = y = 4 km/h
Speed of upstream = (x-y) = 8 – 4 = 4 km/h

28-A
Method 1 : work done by A in 1 day = 1/10

Work done by B in 1 day = 1/15


Work done by A and B together in 1 day = 1/10 + 1/15 = 1/6
They can complete it in 6 days.
Method 2 : using formula A and B can do the work in
10x15/10+15 = 150/25 = 6 days.
Method 3 : calculate LCM ( 10,15 ) = 30
The answer in how days they will complete the work together will be
30/(30/10 +30/15) = 6 days.
By the method of LCM the problems in which there are more than 2 persons working
can also be solved easily. example 2 will make it very clear

29-A

Sol : part of tank filled in 1 hr by pipe A = 1/10

part of tank filled in 1 hr by pipe A = 1/15

therefore the part of tank filled by both A and B together = 1/10 + 1/15 =1/6

and hence total time taken by them together = 6 hrs.

method 2 : use direct formula ab/a+b i.e. 10 x 15/10+15 =6

www.careers360.com Back to Index 40


method 3: by concept of lcm i.e. take lcm of 10 & 15 =30

so time taken together = 30/(30/10 + 30/15)

This method can also be used if there are more than 2 inlet pipes.

Hence we find that the problems of pipes and cisterns can be solved almost in the same
manner as time and work.

The man got replaced by pipe and time taken by man is replaced by time taken by pipe
to fill a tank.

30-B

31-B
Ratio if Ages = 9 : 5
9x = 5x + 8
= > 4x = 8
=>x=2
So B, A ages equals to 10 , 18

32-A

Given:
96 L 4 K 6 M 11 D 9 = ?

www.careers360.com Back to Index 41


On replacing the letters with the mathematical signs, the equation becomes –
= 96 ÷ 4 – 6 + 11 × 9
= 24 – 6 + 11 × 9
= 24 – 6 + 99
= 123 – 6
= 117

So, 117 is the required answer to the given equation. Hence, the first option is the
correct.

33-A
Given:
672, 666, 660, 654, ?
Here, subtract 6 from each number to get the next number.
672 – 6 = 666; 666 – 6 = 660; 660 – 6 = 654; 654 – 6 = 648
648 is the missing number of the series. Hence, the first option is correct.

34-C

Given:
_ yzaa _ y _ xy _ aazy _

First, divide the series before filling it→_ yz/ aa/ _ y _/ xy _/ aa/ zy _
Now, check the order of the letters in the given series→xyz / aa / zyx / xyz / aa / zyx
(xyzaazyx is repeated in the series.)

The series becomes→xyzaazyxxyzaa​z​ yx. Hence, the third option is correct.

35-B
The shaded part in the diagram given below represents papers that are also hard.

Papers that are also hard→14 + 9 = 23

www.careers360.com Back to Index 42


23 papers are also hard. Hence, the second option is correct.

36-A

Let W1 be the weight of Book 1, W2​be the weight of Book 2, W3 be the weight of Book
3 and W4 be the weight of Book 4.

Book 1 is twice as heavy as Book 2. So, W1 > W2.


Book 3's weight is half of Book 2's weight. So, W2 > W3.
Book 4 is 60 grams more than Book 2, but 60 grams less than Book 1. So, W1 > W4 >
W2.
Combining all these, we get W1 > W4 > W2 > W3.

Book 1 is the heaviest of all. Hence, the first option is correct.

READING COMPREHENSION

37-D

● The given sentence is in the passive voice. We have to change the sentence into
the active form.
● The given sentence is written in 'Present indefinite tense' in passive form. It is
also a negative sentence.
● The structure of these kinds of sentences is:
○ Passive Voice: Oblyct + am/is/are + not + V3 + by + subject.
○ Active Voice: Subject + am/is/are/do/does + not + V1 + Object.
● The object of the given sentence is "I," and the subject of the given sentence is
'this assignment.'
● So, in the active voice, the position of 'object and 'subject will be interchanged
● The object 'I' will be changed into 'me' in the active voice
● Also here, 'am not interested' will be changed into 'does not interest.'

Therefore, the correct answer is "this assigment does not interest me."

38-A

● The given sentence is in the active voice. We have to change the sentence into
passive form.
● The given sentence is written in 'present indefinite tense' form.
● The structure of these kinds of sentences is:

www.careers360.com Back to Index 43


○ Active Voice: Subject + V1 + Object + infinitive (to + V1)
○ Passive Voice: Subject + V1 + (to + be + V3) + by + Object
● The subject of the given sentence is 'He' and the object of the given sentence is
'people'
● So, in the passive voice, the position of the "subject" will not change, and the
'object' will come at the end.
● Also here, 'to flatter' will be changed into 'to be flattered'
● The pronoun 'him' will be omitted in the passive voice

Therefore, the correct answer is "he likes to be flattered by people."

39-C

● Let's look at the meaning of the given idiom.


● The phrase "cut a sorry figure" means "to appear in a way or make an
unlikeable impression that causes oneself to be regarded unfavourably by
others."

Hence, the correct answer is "to create a poor impression."

40-A

● Let's look at the meaning of the marked option:


○ Omnivore: an animal that is naturally able to eat both plant-based food
and other animal's meat.
● Let's look at the meanings of the other given options:
○ Herbiore: an animal that eats only plants.
○ Carnivore: an animal that eats meat.
○ Cannibal: a person who eats human flesh, or an animal that eats the flesh
of animals of its own type.
○ Therefore, the correct answer is "omnivore."

41-D

● The given sentence is in the active voice. We have to change the sentence into
passive form.
● The given sentence is written in 'present continuous tense.'
● The structure of these kinds of sentences is:
○ Active Voice: Subject + is/am/are + V1 + ing + Object.
○ Passive Voice: Object + is/am/are + being + V3 + by + Subject
● The subject of the given sentence is 'India' and the object of the given sentence
is 'handicrafts.'

www.careers360.com Back to Index 44


● So, in the passive voice, the position of 'subject' and object' will be interchanged
● Also here, "is exporting" will be changed to 'are being exported.' As
'Handicrafts' is plural in number, we have to use 'are' with it.

Therefore, the correct answer is "handicrafts are being exported to several


countries by India."

42-D

● The error lies in the part "are my friend's son."


● The given sentence talks about a single person, "the most talented" among
three other boys, who is my friend's son.
● As per the rule of grammar, we have to apply "be verb" according to the single
person.
● So, we need to use "is" instead of "are" to make the sentence grammatically
correct

Hence, the correct answer is: "are my friend's son."

43-A

● The error lies in the part "as soon the."


● The given sentence contains two parts: "the gunshot was heard" and "people
ran out in panic."
● Here, both incidents are happening one after another.
● In such cases, "as soon as" is used as a conjunction to join the sentences.
● "As soon as" is used when something happens immediately after some other
event in the present and past tense.
● So, we need to use "as soon as" instead of "as soon" to make the sentence
grammatically correct.

Hence, the correct answer is "as soon the."

44-B

● The given sentence is grammatically incorrect.


● Here, 'made much mischief' should be used instead of 'did many mischiefs'.
● According to grammar, "much" is used with uncountable nouns, and 'mischief
is an uncountable noun.
● In the given context, 'make mischief is a phrase. It means 'to intentionally say
or do something that causes other people to be upset or annoyed with each
other'.

www.careers360.com Back to Index 45


Thus, the correct answer is "made much mischiefs."

45-C

● The given sentence is grammatically incorrect.


● Here, 'he had forgotten' should be used instead of "he had forgot."
● We know that after 'had' we have to use the '3rd form of the verb'.
● Here, the '2nd' form is used. So, it is incorrect.

Thus, the correct answer is "he had forgot."

46-B

● The most appropriate word for the given blank is 'deferred'


● It meant "to leave until a later time."
● We use the word "defer" to "decide" to do something later on.
● In the given context, it is referring to the club meeting.

Thus, the correct answer is "deferred."

47-B

● The given sentence is in indirect speech, and we need to convert it to direct


speech.
● When we change an indirect speech to a direct speech, we must follow these
rules:
○ If the indirect part of the sentence uses 'asked' as reporting verb, then a
"question mark" should be added in the direct speech part
○ The indirect speech part is always stated in statement form. so a "full
stop" is used instead of a question tag.
○ The reporting verb 'asked' changes to 'said to'
○ The conditional 'if is replaced with inverted commas ("....") to show the
direct speech part
○ A comma (,) is added after the direct speech part.
○ The first word of the direct speech is in capital letters.
○ The modal 'would hanges to 'will' to show the direct speech part
○ The pronoun'she' changes to 'you' and 'her' changes to "me" and "my"
accordingly.

Therefore, the correct answer is that Taru said to Kavya, "Will you help me in setting
up my new dressing table?"

www.careers360.com Back to Index 46


48-A

● The given sentence is grammatically incorrect.


● Here, 'or you will' should be used instead of 'either you will'.
● Either...Or ('one or another') is used in affirmative sentences to offer a choice
between two possibilities or to express a cause-and-effect relationship
● In the given sentence, only 'or' should be used.
● Because the conjunction 'or' means 'if not; otherwise.

Thus, the correct answer is "either you will."

GENERAL KNOWLEDGE

49-A
Ahmedabad is situated on the banks of river Sabarmati. The Sabarmati River is a
significant west-flowing river in India that rises in the Aravalli Mountains in the Udaipur
District and joins the Gulf of Khambhat in the Arabian Sea. The principal tributaries
include the Watrak, Wakal, Hathmati, Harnav, and Sei rivers.

50-D
Constructed in 1962, the Govind Ballabh Pant Sagar Dam, also recognized as the
Rihand Dam, stands as a concrete gravity dam situated on the Rihand River. This
impressive structure is found in the Sonbhadra District of Uttar Pradesh, and its
reservoir area straddles the border of Madhya Pradesh and Uttar Pradesh. Rihand
River serves as a tributary to the Son River.

51-B
Agronomy is the science of growing crops and plants that provide us with food, fiber, oil,
medicinal resources, and other essentials for human well-being. It covers everything
from managing soil and breeding plants to dealing with pests and understanding plant
growth. Agronomists aim to maximize crop yields while keeping in mind sustainability,
environmental concerns, and economic effectiveness.

52-C
Science of preservation of historical articles is known as Museology. Historical items are
kept in museums to preserve them, Epigraphy is study of inscriptions, epigraphs &
Numismatics is the study of coins and currencies, Iconography is the study of visual
images and symbols

www.careers360.com Back to Index 47


53-B
The correct answer is B. Clothing primarily serves to protect the body from
environmental elements such as cold, heat, and physical hazards.

54-B
The correct answer is B. Cotton is the natural fiber commonly used to make jeans due
to its durability and comfort.

55-A
The Kyoto Protocol is an international convention that aims to address climate change
by establishing enforceable targets for industrialized countries to reduce greenhouse
gas emissions. It was enacted on December 11, 1997, in Kyoto, Japan, and took effect
on February 16, 2005. The agreement represents a significant step forward in global
efforts to tackle climate change, and it is closely linked to the United Nations Framework
Convention on Climate Change (UNFCCC).

56-C
Limestone is an essential component in the manufacture of cement. It contains calcium
oxide (lime), which combines with other elements in the cement-making process to
generate the binder that keeps concrete and mortar together.

57-B
He was an explorer, ancient Greek historian, and the envoy in the Mauryan court at
Pataliputra, who had been sent by the Greek tyrant Seleucus to Chandragupta Maurya's
court. Megasthenes stayed at Pataliputra, In the Mauryan empire, and published a book
known as "Indica" about his impressions of India.

58-A
The Test of My Life is an autobiography of a famous Indian cricketer Yuvraj singh.
Yuvraj singh played for Indian cricket team for a long time, also he was a member of the
world cup winning team of 2007, 2011. This book tells about those tough days when he
was fighting with cancer and how he fought & came out of that phase. Sachin
Tendulkar's book is Playing it my way.

59-B
The institution known as the "Soft Loan Window" of the World Bank is the
International Development Association (IDA). It is a development finance
organization with 174 member countries that offers loans as well as grants to poor
countries all over the world with advantageous terms. Its head office is in Washington

www.careers360.com Back to Index 48


D.C., in the United States. Ajay Banga was appointed as the World Bank Group's 14th
President on June 2, 2023.

60-A
Lok Sabha is presided over by a Speaker. The speaker is the constitutive head of the
Lok Sabha. Also, the joint session is presided over by the speaker and chosen by the
simple majority of the members of the Lok Sabha. The speaker has a casting vote, he
or she can only vote in a tie situation. The speaker is also responsible for appointing the
chairpersons of all parliamentary committees.

www.careers360.com Back to Index 49


MOCK TEST - 2
____________________________________________________________________________

LOGICAL REASONING
Q1.Direction: In the following question below, some statements are followed by
some conclusions. Taking the given statements to be true, even if they seem to
be at variance from commonly known facts, read all the conclusions and then
decide which of the given conclusions logically follows from the given
statements.

Statements:
I. We are going back again to our ancestors and finding out the importance of
Yoga and Pranayam.
II. People in the West have already opted it. It does not require any external
equipment but only body and soul.
Conclusions:
I. Ancient science is the treasure of many cures and natural remedies for various
diseases.
II. Technology has overshadowed these ancient sources and introduced new
concepts of fitness called gym.

A) Only Conclusion II follows


B) Conclusions I and II both follow
C) Neither I nor II follows
D) Only Conclusions I follows

Q2.Direction: In the following question below, some statements are followed by


some conclusions. Taking the given statements to be true, even if they seem to
be at variance from commonly known facts, read all the conclusions and then
decide which of the given conclusions logically follows from the given
statements.

Statements:
I. Children play in the playground.
II. It gives them a different approach to problem-solving and decision-making.
Conclusions:
I. Playing outdoors is very necessary for the overall development of a child.
II. There should be a balance between playing and studying for a child.

www.careers360.com Answer 50
A) Only Conclusion II follows
B) Conclusions I and II both follow
C) Neither Conclusion I nor II follows
D) Only Conclusion I follows

Q3.Assertion: The sky appears blue during the day.

Reason: Rayleigh scattering causes shorter wavelengths of light, such as blue, to


be scattered in all directions.

A) Both the assertion and reason are true, and the reason is a correct explanation of the
assertion.

B) Both the assertion and reason are true, but the reason is not a correct explanation of
the assertion.

C) The assertion is true, but the reason is false.

D) The assertion is false, but the reason is true.

Q4. Assertion:The ozone layer in the Earth's atmosphere protects us from


harmful ultraviolet (UV) radiation.

Reason:Ozone molecules in the stratosphere absorb and dissipate UV radiation.

A) Both the assertion and reason are true, and the reason is a correct explanation of the
assertion.

B) Both the assertion and reason are true, but the reason is not a correct explanation of
the assertion.

C) The assertion is true, but the reason is false.

D) The assertion is false, but the reason is true.

Q5.According to the given statements, Which of the following options is correct.

Statement 1:
Water boils at 100 degrees Celsius at standard atmospheric pressure.

Statement 2:
Water boils at 50 degrees Celsius at standard atmospheric pressure.

www.careers360.com Answer 51
A) Statement 1 is correct, but Statement 2 is incorrect.
B) Statement 2 is correct, but Statement 1 is incorrect.
C) Statement 1 is incorrect.
D) Statement 2 is incorrect.

Q6.According to the given statements, Which of the following options is correct.


Statement 1:
Photosynthesis is the process by which plants convert sunlight into energy.
Statement 2:
Respiration is the process by which plants convert carbon dioxide into oxygen.

A) Statement 1 is correct, but Statement 2 is incorrect.


B) Statement 2 is correct, but Statement 1 is incorrect.
C) Statement 1 is incorrect.
D) Statement 2 is incorrect.

Q7. Direction: In the following question, a statement is given, followed by two


arguments, I and II. You have to consider the statement to be true, even if it
seems to be at variance from commonly known facts. You have to decide which
of the given arguments, if any, follows from the given statement.

Statement:
"Should social media companies be required to monitor and remove harmful
content?"
Arguments:
I. Yes, it can help reduce cyberbullying and the spread of false information.
II. No, it infringes on freedom of speech and can lead to censorship.

A) Only Argument I follows.


B) Only Argument II follows.
C) Both Arguments I and II follow.
D) Neither Argument I nor Argument II follows.

Q8 Direction: In the following question, a statement is given, followed by two


arguments, I and II. You have to consider the statement to be true, even if it
seems to be at variance from commonly known facts. You have to decide which
of the given arguments, if any, follows from the given statement.

Statement:

www.careers360.com Answer 52
"Should single-use plastic products be banned to reduce environmental
pollution?"
Arguments:
I. Yes, it will reduce plastic waste and benefit the environment.
II. No, it will lead to job losses in the plastic manufacturing industry.

A) Only Argument I follows.


B) Only Argument II follows.
C) Both Arguments I and II follow.
D) Neither Argument I nor Argument II follows.

Q 9.Direction: In each of the following question, select the odd letter from the
given alternatives.

A) QP
B) DC
C) XY
D) ML

Q10.Direction: In the following question, find the odd one out from the given
alternatives.

A) DCF
B) NMP
C) VUX
D) JIM

Q11. W is married to X. W is father of Y. C is mother in law of X. C is married to


A.B is only grandson of C. How is Y related to W?

A) Aunt
B) Daughter
C) Daughter-in-law
D) Daughter-in-law

Q12.Direction: Sameer is younger than Meera. Prayas is older than Sameer but
younger than Sakshi. Who is the eldest of the four?

www.careers360.com Answer 53
A) Meera
B) Prayas
C) Sameer
D) Meera or Sakshi

LEGAL REASONING
Q13: In a criminal trial, the defense attorney discovers that the prosecution failed
to disclose a key piece of evidence that could exonerate the defendant. What
constitutional right is at stake, and what remedy might the defense seek?

A) Right to a Speedy Trial; Motion for Dismissal


B) Right to Confront Witnesses; Motion to Suppress
C) Right to Due Process; Motion for Discovery
D) Right to Legal Counsel; Motion for Mistrial

Q14 :During a contract dispute, the parties decide to resolve their issues through
arbitration. The arbitrator, however, exhibits bias towards one party, resulting in
an unfair decision. What legal doctrine might the affected party invoke?

A) Res Judicata
B) Arbitration Estoppel
C) Forum Non Conveniens
D) Manifest Disregard of the Law

Q15: In an employment discrimination case, the plaintiff claims they were


wrongfully terminated due to their gender. The employer argues that the
termination was based on poor performance. Which legal doctrine might the
court apply to assess the employer's motives?

A) Respondeat Superior
B) Bona Fide Occupational Qualification
C) McDonnell Douglas Test
D) Collateral Estoppel

www.careers360.com Answer 54
Q16: A person intentionally spreads false and damaging information about a
competitor's product, causing significant financial harm. What tort might the
competitor pursue?

A) Slander
B) Libel
C) Tortious Interference
D) Nuisance

Question 17:In a negligence lawsuit, the plaintiff argues that the defendant
breached the duty of care, leading to the plaintiff's injuries. The defendant claims
that the plaintiff's own actions contributed to the harm. What defense is the
defendant asserting?

A) Contributory Negligence
B) Assumption of Risk
C) Comparative Fault
D) Res Ipsa Loquitur

Q18 :During a search warrant execution, law enforcement seizes evidence not
specified in the warrant. What constitutional violation might this scenario
involve?

A) Fourth Amendment Violation


B) Fifth Amendment Violation
C) Eighth Amendment Violation
D) Tenth Amendment Violation
Answer A) Fourth Amendment Violation

Q19 :A company is accused of antitrust violations for monopolistic practices.


What federal agency is primarily responsible for investigating and enforcing
antitrust laws?

A) Securities and Exchange Commission (SEC)


B) Federal Trade Commission (FTC)
C) Department of Justice (DOJ)
D) Consumer Financial Protection Bureau (CFPB)

www.careers360.com Answer 55
Q20: A person is charged with a crime but cannot afford legal representation.
What constitutional right ensures the provision of legal counsel in such a
situation?

A) Right to a Speedy Trial


B) Right to Remain Silent
C) Right to Legal Counsel
D) Right to a Jury Trial

Q21.A company advertises its product as "100% natural," but laboratory tests
reveal the presence of synthetic additives. What legal claim might consumers
pursue against the company?

A) Fraudulent Misrepresentation
B) Breach of Contract
C) Product Liability
D) Unjust Enrichment

Question 22: A person creates a parody of a famous painting, incorporating


humorous elements. The original artist claims copyright infringement. What legal
doctrine might the defendant rely on as a defense?

A) Fair Use Doctrine


B) Transformative Use Doctrine
C) Substantial Similarity Doctrine
D) Doctrine of Originality

Q23:In a territorial dispute between two neighboring countries, one claims


historical sovereignty based on ancient treaties, while the other relies on the
principle of uti possidetis. Which principle is often favored in contemporary
international law to resolve such disputes?
A. Historical Sovereignty
B. Uti Possidetis
C. Principle of Necessity
D. Doctrine of Territorial Integrity

www.careers360.com Answer 56
Q 24: If a state seeks reparation for environmental damage caused by another
state's activities within its jurisdiction, which legal doctrine is likely to be invoked
under international law?
A. Sovereign Immunity
B. Erga Omnes
C. Pacta Sunt Servanda
D. Extraterritorial Jurisdiction

ANALYTICAL REASONING
Q25.A shopkeeper marks the price of an article at Rs. 80. Find the cost price if
after allowing a discount of 10% he still gains 20% of the cost price.

A] 60
B] 120
C] 75
D] 45

Q26.125 toffees cost Rs. 75, Find the cost of one million toffees if there is a
discount of 40% on the selling price for this quantity.

A] 3,20,000
B] 3,40,000
C] 3,60,000
D] 3,00,000

Q27. Find the largest number which divides 606, 732 and 915 leaving remainders
of 6, 12 and 15 respectively.

A] 60
B] 75
C] 90
D] 120

www.careers360.com Answer 57
Q28. The interest on a certain sum lent at compound interest, the interest being
compounded annually, in the 2nd year is Rs.1200. The interest on it in the 3rd
year is Rs 1440. Find the rate of interest per annum.

A] 20%
B] 10%
C] 30%
D] 40%

Q29. If Rs.2000 amounts to Rs.2880 in 2 years at compound interest, what is the


rate of interest per annum if the interest is being compounded annually?

A] 10%
B] 20%
C] 25%
D] 30%

Q30. A train covers a distance of 200 km with a speed of 10 km/h. What time is
taken by the train to cover this distance?

A] 20 hours.
B] 30 hours.
C] 40 hours.
D] 10 hours.

Q31. A bike crosses a bridge with a speed of 108 km/h. What will be the length of
the bridge if the bike takes 8 minutes to cross the bridge?

A] 1400m
B] 1382 m
C] 14400m
D] 12000m

www.careers360.com Answer 58
Q32. A man can row with a speed of 6 km/h in still water. What will be his speed
with the stream, if the speed of the stream is 2 km/h?

A] 8 km/h
B] 10 km/h
C] 6 km/h
D] 2 km/h

Q33. If A, B, C, and D can complete a piece of work in 10, 15, 20 and 25 days
respectively. find in how many days they will complete the work working
together?
A] 300/ 77 days.
B] 200/ 77 days.
C] 100/ 77 days.
D] 500/ 77 days.

Q34. Tap A takes 4 hrs to fill a tank. Tap B takes 6 hrs to fill the same tank. If both
the taps are opened simultaneously, by the time the tank was full, what fraction of
the tank was filled by tap A?

A] 2/5 part.
B] 3/5 part.
C] 4/5 part.
D] 1/5 part.

Q35.Directions: If 4 * 9 % 2 = 47 and 9 * 0 % 6 = 84, then 5 * 3 % 7 = ?

A) 38
B) 51
C) 42
D) 46

www.careers360.com Answer 59
Q36.Directions: A series is given below with one term missing. Choose the
correct alternative from the given ones that will complete the series.
CD, FA, IX, LU, ?

A) TU
B) RS
C) OP
D) OR

READING COMPREHENSION

Q37.Select the option that contains a grammatical error in the underlined portion.

When Mrs. Sinha came to see Varanasi in 2019, Nupur has already been teaching
there for five years.

A) When Mrs. Sinha came to see Varanasi in 2019, Nupur has already been teaching
there for five years.

B) When Mrs. Sinha came to see Varanasi in 2019, Nupur has already been teaching
there for five years.

C) When Mrs. Sinha came to see Varanasi in 2019, Nupur has already been teaching
there for five years.

D) When Mrs. Sinha came to see Varanasi in 2019, Nupur has already been teaching
there for five years.

Q38.Select the most appropriate idiomatic expression to fill in the blank.


Anita keeps on repeating that she is innocent, but there is ___________.

A) no smoke without fire

B) no pulling up of the socks

C) no rubbing of the shoulders

D) no love lost between

www.careers360.com Answer 60
Q39.Select the option that expresses the given sentence in active voice.
Those four pages were torn by Jack.

A) Jack tears those four pages.


B) Jack tore those four pages.
C) Jack tear these four pages.
D) Jack tore these four pages.

Q40. Select the most appropriate ANTONYM of the underlined word.


We lived a very frugal life during our childhood.

A) Reckless
B) Weary
C) Severe
D) Extravagant

Q41. Select the option that can be used as a one-word substitute for the given
group of words.
Which cannot be read.

A) Illiterate
B) Inaudible
C) Invisible
D) Illegible

Q42. Select the INCORRECTLY spelt word.

A) Prefferance
B) Preservation
C) Preponderance
D) Perseverance

Q43.Select the most appropriate ANTONYM of the underlined word to fill in the
blank.

While most friends persuaded him to choose the career, there were a few
who_________ him from doing so.

A) evaded
B) bombarded
C) dissuaded
D) situated

www.careers360.com Answer 61
Q44. Select the most appropriate option to fill in the blank.
Shakuntala Devi was born on 4th November 1929. In her childhood she________
to school.

A) never gone

B) went never

C) had never been

D) ever been

Q45. Select the sentence that contains no spelling errors.

A) The childrens playing in the kindergarden allmost broke the window of the principal’s
car.

B) The children playing in the kindergarten almost broke the window of the principal’s
car.

C) The childrens playing in the kindergarten almost broke the window of the principal’s
car.

D) The chilldren playing in the kindergarten almost broke the window of the principal’s
car.

Q46.Parts of the following sentence have been given as options. Select the option
that contains an error.

She is in an hurry to meet her brother.

A) to meet

B) She is in

C) her brother

D) an hurry

Q47. Select the most appropriate option to fill in the blanks.


Soon after the doctor gave her the next ______ of the medicine, the patient began
to______.

A) dose, dozed

www.careers360.com Answer 62
B) dosed, doze

C) dose, doze

D) doze, dose

Q48. Sentences of a paragraph are given below in jumbled order. Arrange the
sentences in the correct order to form a meaningful and coherent paragraph.
A. In Catholic, Eastern Orthodox, Anglican, Oriental Orthodox and Lutheran
teaching, all of their faithful departed in heaven are considered saints, although
some are deemed deserving of more praise or emulation.
B. In religious belief, a saint is a person who is viewed as possessing an
uncommon degree of holiness, similarity, or proximity to God.
C. The term saint, however, is used differently depending on the setting and
denomination.
D. Some saints receive official ecclesiastical recognition and, as a result, a public
cult of veneration through the process of canonisation in the Catholic Church or
glorification in the Eastern Orthodox Church.

A) BCAD

B) BCDA

C) CBDA

D) BDAC

GENERAL KNOWLEDGE

Q49. Which of the following is a book written by Shashi Tharoor?

A) It's Not About You


B) Invisible People
C) An Era of Darkness
D) Democrats and Dissenters

Q50.Who is the author of the book 'Akhada: The Authorized Biography of Mahavir
Singh Phogat'?

A) Rabi Thapa
B) Saurabh Duggal
C) Salman Rushdie
D) Tana French

www.careers360.com Answer 63
Q51. Abhigyanshakuntalam is a

A) play
B) novel
C) poetry
D) story

Q52. The country having the largest rail network in the world is

A) Russia
B) U.S.A.
C) China
D) India

Q53. According to Paul Appleby the quality of good administrators is

A) intelligence
B) integrity
C) loyalty
D) self -confidence

Q54.Dodo is a bird categorized under _____.

A) extinct
B) endangered
C) critically endangered
D) rare

Q55.Which of the following is not an example of an amphibian?

A) Frog
B) Sea horse
C) Salamander
D) Toad

Q56.Wellington Trophy is associated with

A) rowing
B) chess

www.careers360.com Answer 64
C) hockey
D) bridge

Q57.Limestone is a raw material used by which industry ?

A) Aluminium
B) Fertilizers
C) Cement
D) Petrochemicals

Q58.A stretch of sea water, partly or fully separated by a narrow strip from the
main sea is called

A) bay
B) isthmus
C) lagoon
D) strait

Q59. ’Public health, sanitation conservancy and solid waste management' is


inserted into the Schedule of the Constitution by which of the following
Constitutional Amendments?

A) 42nd Constitutional Amendment.


B) 44th Constitutional Amendment
C) 73rd Constitutional Amendment
D) 74th Constitutional Amendment

Q60.According to Article 74 of the Constitution, which of the following statements


relating to the advice tendered by Ministers to the President would be correct?

A) It shall not be inquired into in any court.


B) It can be inquired into in the Supreme Court.
C) It can be inquired into in all the courts.
D) It cannot be inquired into in the High Courts.

www.careers360.com Answer 65
ANSWER KEY
____________________________________________________________________________

1 2 3 4 5 6 7 8 9 10
D D A A A A C A C D
11 12 13 14 15 16 17 18 19 20
B D C D C C C A C C
21 22 23 24 25 26 27 28 29 30
A A B B A C A A B A
31 32 33 34 35 36 37 38 39 40
A A A B D D B A B D
41 42 43 44 45 46 47 48 49 50
D A C C B D A A D B
51 52 53 54 55 56 57 58 59 60
A B D A B A C C D A

www.careers360.com Solutions 66
SOLUTIONS
____________________________________________________________________________

LOGICAL REASONING
1-D
Given:
Statements:
I. We are going back again to our ancestors and finding out the importance of Yoga and
Pranayam.
II. People in the West have already opted it. It does not require any external equipment
but only body and soul.
Conclusions:
I. Ancient science is the treasure of many cures and natural remedies for various
diseases.
II. Technology has overshadowed these ancient sources and introduced new concepts
of fitness called gym.
Based on the given statements,
Conclusion I logically follow from the given statements. Statement I suggests that
people are exploring the importance of Yoga and Pranayam, which implies that ancient
practices like Yoga and Pranayam offer remedies for various diseases.
Conclusion II doesn't directly follow from the statements. While the statements discuss
the adoption of Yoga and Pranayam, they don't explicitly mention that technology has
introduced gym-based fitness concepts.
Thus, only Conclusion I logically follows from the given statements. Hence, the fourth
option is correct.

2-D
Given:
Statements:
I. Children play in the playground.
II. It gives them a different approach to problem-solving and decision-making.
Given Conclusions:

www.careers360.com Back to Index 67


I. Playing outdoors is very necessary for the overall development of a child.
II. There should be a balance between playing and studying for a child.
Based on the given statements,
Conclusion I logically follow from the statements. The fact that children play in the
playground suggests that playing outdoors is essential for a child's overall development.
Conclusion II doesn't directly follow from the statements. While the benefits of outdoor
play are implied, the statements don't explicitly address the need for a balance between
playing and studying.
So, the correct answer is only Conclusion I logically follows from the given statements.
Hence, the fourth option is correct.

3-A
The sky appears blue during the day because of Rayleigh scattering, which scatters
shorter wavelengths of light, like blue and violet, more than longer wavelengths, making
the blue light dominant.

4-A
The ozone layer absorbs and dissipates harmful UV radiation, which is a crucial
protective mechanism for life on Earth.

5-A
Water indeed boils at 100 degrees Celsius at standard atmospheric pressure (1 atm).
Statement 1 is correct. Statement 2 is incorrect because it does not represent the
boiling point of water under standard conditions.

6-A
Statement 1 accurately describes photosynthesis, where plants convert sunlight into
energy. Statement 2 is incorrect because respiration in plants involves the conversion of
oxygen into carbon dioxide, not the other way around.

7-C
Argument I emphasizes the benefits of content monitoring, while Argument II raises
concerns about potential freedom of speech violations.

www.careers360.com Back to Index 68


8-A
Argument I supports the idea of banning single-use plastics for environmental reasons,
while Argument II is based on potential economic concerns.

9-C
Let's check the options —

First option: QP; Q – 1 = P

Second option: DC; D – 1 = C

Third option: XY; X + 1 = Y

Fourth option: ML; M – 1 = L

From the above, it can be said that the third option is different from the other three.

Hence, the third option is correct.

10-D
Let's check the options —

First option: DCF; D – 1 = C; C + 3 = F

Second option: NMP; N – 1 = M; M + 3 = P

Third option: VUX; V – 1 = U; U + 3 = X

Fourth option: JIM; J – 1 = I; I + 4 = M

From the above, it can be said that the fourth option is different from the other three.

Hence, the fourth option is correct.

www.careers360.com Back to Index 69


11-B

Hence, the correct answer is option 2.

12-D
Given:

Sameer is younger than Meera, i.e. Sameer > Meera

Prayas is older than Sameer but younger than Sakshi, i.e. Shakshi > Prayas > Sameer

So, either Meera or Sakshi is the eldest of four. Hence, the fourth option is correct.

LEGAL REASONING
13-C
The right to due process encompasses the obligation of the prosecution to disclose
exculpatory evidence to the defense. If such evidence is not disclosed, the defense can
file a motion for discovery seeking the missing information.

14 -D
Manifest disregard of the law is a doctrine that allows a court to vacate an arbitration
award if the arbitrator blatantly ignores applicable legal principles, leading to an unjust
decision.

15-C
The McDonnell Douglas Test is a legal framework used in employment discrimination
cases to analyze whether the employer's stated reason for an adverse employment
action is a pretext for discrimination.

www.careers360.com Back to Index 70


16- C
Tortious interference occurs when a third party intentionally disrupts a contractual or
business relationship, causing economic harm. In this scenario, spreading false
information to harm a competitor's product could be considered tortious interference.

17 -C
Comparative fault allows for a proportionate allocation of damages based on each
party's degree of fault in a negligence case. It considers the actions of both the plaintiff
and the defendant in causing the harm.

18-A
The Fourth Amendment protects against unreasonable searches and seizures. If law
enforcement exceeds the scope of a search warrant, it constitutes a violation of this
constitutional right.

19-C
While both the Federal Trade Commission (FTC) and the Department of Justice (DOJ)
play roles in enforcing antitrust laws, the DOJ is primarily responsible for prosecuting
criminal antitrust violations.

20-C
The Sixth Amendment guarantees the right to legal counsel for individuals accused of a
crime, even if they cannot afford to hire an attorney.

21-A
Fraudulent misrepresentation occurs when false information is intentionally
communicated to deceive others. In this case, advertising a product as "100% natural"
while containing synthetic additives could be considered fraudulent misrepresentation.

22-A
The Fair Use Doctrine allows for the limited use of copyrighted material for purposes
such as criticism, commentary, or parody. In this scenario, creating a parody of a
famous painting might be considered fair use.

23-B
In contemporary international law, the principle of uti possidetis is often favored in
resolving territorial disputes. This principle upholds the boundaries existing at the time
of independence, discouraging changes based on historical claims and promoting
stability by respecting established borders.

www.careers360.com Back to Index 71


24-B
Under international law, the doctrine of erga omnes is relevant when a state seeks
reparation for environmental harm that affects the global community or violates
fundamental principles. This doctrine emphasizes obligations owed to the international
community as a whole, allowing states to bring claims on behalf of the collective interest
in protecting the environment.

ANALYTICAL REASONING
25-A

26-C

27-A
606-6=100
732-12=720
915-15= 900
HCF of 600, 720, 900 = 60

28-A
Interest of 2nd year = 1200
Interest of 3rd year = 1440
Difference = 240
In compound interest, interest in next year is calculated on previous year
So, = 240 / 1200 x 100 = 20 %
Increase in next year interest is what percent of the interest of previous year is = to the
rate of interest

29-B
P = 2000, A = 2880, T = 2 years, R =?
A = P [1 + R/100] T
2880 = 2000 [1 + R/100]2

www.careers360.com Back to Index 72


2880/2000 = [1+R/100]2
1.2 = 1 + r
Now, subtract 1 from both sides to isolate r:
1.2 - 1 = r
0.2 = r
So, the annual interest rate (r) is 0.2, which is equivalent to 20%.
So, the correct answer is:20%

30-A
𝐷𝑖𝑠𝑡𝑎𝑛𝑐𝑒
Speed = 𝑇𝐼𝑚𝑒

Where speed = 10 km/h and distance = 200 km


200
10 = 𝑇𝑖𝑚𝑒

Required time = 20 hours.

31-A
Here, length of the bridge = Distance travelled by bike in 8 minutes
= Speed X Time
Given that speed = 108 km/h = 108 X 518 m/s = 30 m/s
Time = 8 minutes = 8 X 60 = 481 seconds
Length of the bridge = 30 X 480 = 14400 m

32-A
Given that
Speed of man = 𝑥 = 6 km/h
Speed of stream = 𝑦 = 2 km/h
Speed downstream = (𝑥 + 𝑦) = 6 + 2 = 8 km/h

33-A
We first find LCM (10, 15, 20, 25) i.e. =300
Now divide this lcm with no. of days in which they complete the work individually
300/10 = 30 , 300/15 = 20 , 300/20 =15 and 300/ 25 =12
Hence the answer will be 300/(30+20+15+12) =300/ 77 days.

34-B
First find the part they together will fill in 1 hr i.e. 1/4 + 1/6 = 5/12
Hence total hrs will b 12/5 in which they will fill the tank.

www.careers360.com Back to Index 73


Now tap A fill the tank 1/4 in 1 hr and hence in 12/5 hr tapA will fill 12/5 x1/4 part = 3/5
part
So 60% part will be filled by tap A and remaining 40% will be filled by tap B.
Method 2: time taken to fill tank is inversely proportional to work done i.e. filling of tank
in case of pipes and so ratio of time taken by A and B are 2:3 therefore efficiency of
work will be in ratio 3:2 and hence tap A will fill 3/(2+3) =3/5 part.

35-D
Given:
4 * 9 % 2 = 47 , 9 * 0 % 6 = 84

Let's deduce the pattern from the solved equations –


4 * 9 % 2 = 47→49 – 2 = 47
And, 9 * 0 % 6 = 84→90 – 6 = 84
Similarly, 5 * 3 % 7→53 – 7 = 46

The required value is 46. Hence, the fourth option is correct.

36-D

Given:
CD, FA, IX, LU, ?

Add 3 to the place value of the first letter and subtract 3 from the place value of the
second letter of the previous terms to get the first and second letters respectively of the
next term.
First letter of the series→C + 3 = F; F + 3 = I; I + 3 = L; L + 3 = O
Second letter of the series→D – 3 = A; A – 3 = X; X – 3 = U; U – 3 = R

So, from the above, OR is the required term of the series. Hence, the fourth option is
correct.

READING COMPREHENSION

37-B

The option with a grammatical error in the underlined portion is the second option i.e.
"When Mrs. Sinha came to see Varanasi in 2019, Nupur has already been teaching
there for five years."

The past perfect tense ("had already been teaching") is used to indicate an action that
was completed before another past action (Mrs. Sinha coming to Varanasi). In the

www.careers360.com Back to Index 74


incorrect option, the present perfect tense ("has already been teaching") is used,
which is not grammatically accurate in this context. The past perfect tense is more
suitable for expressing the sequence of events in the past.

Therefore, the correct sentence should be "When Mrs. Sinha came to see Varanasi in
2019, Nupur had already been teaching there for five years."

38-A

The most appropriate idiomatic expression to fill in the blank is the first option, "no
smoke without fire."

This expression means that there is usually a reason or cause behind a rumour or
accusation. In the context of the sentence, it suggests that there might be some truth to
the claim that Anita is not innocent, as her repeated denials could imply guilt.

The meanings of the other options are as follows:

● Pulling up socks means making an effort to improve your work.


● Rubbing of shoulders means spending time with someone, especially with
someone famous.
● No love lost between means there is a mutual dislike.

Therefore, the correct sentence is, "Anita keeps on repeating that she is innocent, but
there is no smoke without fire."

39-B

The second option expresses the given sentence in active voice i.e. "Jack tore those
four pages."

To convert a passive voice sentence to an active voice, we need to identify the doer of
the action (the subject) and place it before the verb. The structure of the sentence in
active voice is "Subject (Jack) + Verb (tore) + Object (those four pages)."

In this case, "Jack" is the doer of the action, "tore" is the verb, and "these four
pages" is the object.

Therefore, the sentence in active voice is, "Jack tore those four pages."

www.careers360.com Back to Index 75


40-D

The most appropriate antonym for the underlined word "frugal" is "extravagant."

"Frugal" means to be economical, thrifty, or to live a simple and cost-effective life. In


contrast, "extravagant" refers to spending or living in a lavish, wasteful, or excessive
manner.

The meanings of the other options are as follows:

● Reckless means rash or impetuous.


● Weary means showing extreme tiredness.
● Severe means strict or harsh.

41-D

The one-word substitute for the group of words "Which cannot be read" is "illegible."

Explanation: "Illegible" is an adjective that describes something that is difficult or


impossible to read due to poor handwriting, printing, or other factors.

42-A

The incorrectly spelt word is "Prefferance." The correct spelling is "Preference."

Explanation: The word "preference" means a greater liking for one thing over another
or a choice made from a range of options.

43-C

The most appropriate antonym for the underlined word in the blank is "dissuaded."

Explanation: In the context of the sentence, "persuaded" means that most friends
encouraged or convinced him to choose the career. Hence, "dissuaded" is the word that
provides the opposite meaning, as it means to discourage or persuade someone not to
do something.

"Evaded" means to escape or avoid something skillfully or cleverly.


"Bombarded" refers to being hit with a large amount of something, often rapidly and
from various directions.
"Situated" indicates where something is located or positioned within a particular place
or context.

www.careers360.com Back to Index 76


Therefore, the correct sentence is: "While most friends persuaded him to choose the
career, there were a few who dissuaded him from doing so."

44-C

The most appropriate option to fill in the blank is: "had never been."

Explanation: "Had never been" is the correct choice because it indicates that during
her childhood, she never attended school. This construction uses the past perfect tense
to convey that the event or action never happened prior to a specific point in time.

So, the complete sentence is: "Shakuntala Devi was born on 4th November 1929. In her
childhood, she had never been to school."

45-B

The sentence with no spelling errors is: "The children playing in the kindergarten
almost broke the window of the principal’s car."

Explanation: This sentence has correct spelling, grammar, and punctuation. It uses
"children" and "kindergarten" correctly and has no spelling errors in the other words.

So, the correct sentence is: "The children playing in the kindergarten almost broke the
window of the principal’s car."

46-D

The option that contains an error is: "in an hurry."

Explanation: The correct article to be used should be "a" instead of "an" before "hurry."
"An" is used before words that start with a vowel sound, while "a" is used before words
that start with a consonant sound. In this case, "hurry" starts with a consonant sound, so
it should be "a hurry."

Therefore the correct sentence is: "She is in a hurry to meet her brother."

47-A

The most appropriate option to fill in the blanks is the third option: dose, dozed.

Explanation: In this sentence, "dose" is the correct term for a measured quantity of
medicine, and "doze" is the correct term for the patient beginning to fall asleep or
become drowsy. The sentence conveys that the patient took a dose of medicine and
then started to doze or fall asleep.

www.careers360.com Back to Index 77


So, the correct sentence is: "Soon after the doctor gave her the next dose of the
medicine, the patient began to doze."

48-A

To form a meaningful and coherent paragraph, the sentences should be arranged in the
following order:

B. In religious belief, a saint is a person who is viewed as possessing an uncommon


degree of holiness, similarity, or proximity to God.

C. The term saint, however, is used differently depending on the setting and
denomination.

GENERAL KNOWLEDGE

49-D
The correct answer is An Era of Darkness.
Shashi Tharoor, a former UN ambassador and Indian politician, wrote the book An Era
of Darkness: The British Empire in India. The book, which was released in 2017, is a
critical examination of the British Raj and its influence on India.

50-B

The correct option is Saurabh Duggal.


Saurabh Duggal's book Akhada: The Authorised Biography of Mahavir Singh Phogat
delves into the life and career of Mahavir Singh Phogat, the legendary Indian wrestling
coach and father of successful wrestlers Geeta Phogat and Babita Kumari. It was
released on December 19, 2016.

51-A
Abhigyanshakuntalam is written by the famous Mahakavi Kalidas, It's a drama based on
a love story of king Dushyant and Shakuntala. This love story has been told in such a
way that it got loved by people not just in India but across the world. Kalidas was a
famous author who had written many plays and poetry, some of which are
Malvikagnimitram, Meghdoot, Kumarsambhava etc.

52-B
Correct answer is U.S.A
Largest rail network in the world In terms of total operating length is the United States of
America. China, Russia and India have the second, third & fourth largest rail networks

www.careers360.com Back to Index 78


respectively. Indian Railways is the largest rail network in Asia and is divided in 18
zones.

53-D
Paul Appleby, a prominent figure in public administration, emphasized the critical role of
self-confidence in effective administrators. This quality signifies a firm belief in one's
capabilities, judgments, and decisions, even when confronted with challenges or
ambiguity. When an administrator possesses self-confidence, they are inclined to make
resolute decisions, communicate with clarity, and lead with unwavering conviction. This
attribute not only instills trust among team members and stakeholders but also signifies
a leader who is poised and competent in their role.

54-A
The correct option is extinct.
Around the late 17th century, the dodo bird became extinct. The dodo is commonly seen
as a symbol of extinction and the impact of humans on the environment. The lack of
natural predators on its island environment caused it to lack the ability to fly. It had a
huge, bulky body, a rounded beak, and a large head. The quick decrease and eventual
extinction of this species were caused by human activities including hunting and the
introduction of exotic species.

55-B
The correct option is sea horse.
The seahorse is a distinctive marine fish with an erect posture, prehensile tail, and
striking look. It is a member of the family Syngnathidae and is well-known for the
involvement of male seahorses in pregnancy and birth.

56-A
The correct answer is Rowing
An annual rowing tournament known as the Wellington Trophy is conducted in India.
The greatest rowers in India compete in one of the most renowned rowing competitions
in the nation. The Duke of Wellington, who served as the Governor-General of India
from 1836 to 1842, is honored in the name of the trophy.

57-C
The correct option is Cement
Limestone is an essential component in the manufacture of cement. It contains calcium
oxide (lime), which combines with other elements in the cement-making process to
generate the binder that keeps concrete and mortar together.

www.careers360.com Back to Index 79


58-C
The correct answer is lagoon
Lagoons are smaller bodies of water that are often surrounded by a barrier island, reef,
or sandbar, which separates them from the larger sea. Lagoons can be found all around
the world, in both tropical and polar climates. Lagoons are important ecosystems that
provide various benefits to both humans and animals. They can serve as crucial
breeding habitats for fish and other marine life and can also help protect coastal areas
from storms and flooding.

59-D
The 12th Schedule of the Indian Constitution deals with the provisions that specify the
powers, authority and responsibilities of Municipalities. This schedule was added by the
74th Amendment Act of 1992. It has 18 subjects.

60-A
There shall be a Council of Ministers with the Prime Minister at the head to aid and
advise the President, who shall act by such advice in his functions. The question of
whether any and, if so, what advice was tendered by Ministers to the President shall not
be inquired into in any court.

www.careers360.com Back to Index 80


MOCK TEST - 3
____________________________________________________________________________________

LOGICAL REASONING
Q1.Direction: X is elder than Z, Y is younger than Z, Z is elder than W. W is
younger than X, who is the eldest?

A) X
B) Y
C) W
D) Z

Q2. Direction: In the following question below, some statements are followed by
some conclusions. Taking the given statements to be true, even if they seem to
be at variance from commonly known facts, read all the conclusions and then
decide which of the given conclusions logically follows from the given
statements.
Statements:
I. All ears are hands.
II. No hand is nose.
Conclusions:
I. No ear is nose.
II. All nose are ears.
III. Some hands are noses.

A) Only I
B) Only II
C) Only III
D) None of the above

Q3.Direction: In the following question below are given some statements followed
by some conclusions. Taking the given statements to be true even if they seem to
be at variance from commonly known facts, read all conclusions and then decide
which of the given conclusions logically follows from the given statements.
Statements:
No cup is plate.
No plate is spoon.
Conclusions:

www.careers360.com Answer 81
I. No cup is spoon.
II. Some spoon are cups.

A) Only I
B) Only II
C) Only III
D) None of the above

Q4.Assertion: When an object is placed in a vacuum, it does not experience air


resistance.

Reason: Air resistance requires the presence of air molecules to act on the
object.

A) Both the assertion and reason are true, and the reason is a correct explanation of the
assertion.

B) Both the assertion and reason are true, but the reason is not a correct explanation of
the assertion.

C) The assertion is true, but the reason is false.

D) The assertion is false, but the reason is true.

Q5.Assertion: The Moon has a smaller gravitational force than the Earth.

Reason: The gravitational force between two objects is directly proportional to


their masses.

A) Both the assertion and reason are true, and the reason is a correct explanation of the
assertion.

B) Both the assertion and reason are true, but the reason is not a correct explanation of
the assertion.

C) The assertion is true, but the reason is false.

D) The assertion is false, but the reason is true.

www.careers360.com Answer 82
Q6.According to the given statements, Which of the following options is correct.

Statement 1:

The Earth is the third planet from the Sun in our solar system.

Statement 2:

The Moon is the largest natural satellite of Mars

A) Statement 1 is correct, but Statement 2 is incorrect.

B) Statement 2 is correct, but Statement 1 is incorrect.

C) Statement 1 is incorrect.

D) Statement 2 is incorrect.

Q7.According to the given statements, Which of the following options is correct

Statement 1:

The capital of France is Paris.

Statement 2:

The capital of Germany is Berlin.

A) Statement 1 is correct, but Statement 2 is incorrect.

B) Statement 2 is correct, but Statement 1 is incorrect.

C) Statement 1 is incorrect.

D) Statement 2 is incorrect.

Q8. .Direction: In the following question, a statement is given, followed by two


arguments, I and II. You have to consider the statement to be true, even if it
seems to be at variance from commonly known facts. You have to decide which
of the given arguments, if any, follows from the given statement.
Statement:
Should rock shows be allowed to run till midnight at tourist places?
Arguments:
I. Yes, more tourists arrive due to rock shows. Tourism is good for the local
economy.

www.careers360.com Answer 83
II. No, local traditions are harmed due to tourism.

A) If only argument I is strong.


B) If only argument II is strong.
C) If both I and II are strong.
D) If neither I nor I is strong.

Q9. Direction: In the following question, a statement is given, followed by two


arguments, I and II. You have to consider the statement to be true, even if it
seems to be at variance from commonly known facts. You have to decide which
of the given arguments, if any, follows from the given statement.
Statement:
Should songs be eliminated from Indian movies?
Arguments:
I. Yes, Hollywood movies are hits despite having no songs.
II. No, songs help to increase the length of the movie.

A) If only argument I is strong


B) If only argument II is strong.
C) If both I and II are strong.
D) If neither I nor I is strong.

Q10. Direction: In each of the following question, select the odd letters from the
given alternatives.

A) EI – LM
B) AE – RT
C) IO – WY
D) OU – DF

Q11. Direction: In the following question, select the odd letter from the given
alternatives.

A) DU
B) KP
C) JQ
D) GT

www.careers360.com Answer 84
Q12. If ‘Q + R’ means ‘Q is father of R’, ‘Q ÷ R’ means ‘R is brother of Q’, ‘Q × R’
means ‘Q is husband of R’, ‘Q – R’ means ‘Q is sister of R’, then which of the
following expressions shows that E is the father in law of F?

A) E + D – F + H
B) E × D – F ÷ G
C) E + F – D ÷ G
D) E + D – H x F

LEGAL REASONING

Q13:In a contract dispute, party A argues that the contract is voidable due to
undue influence. What must A demonstrate to establish undue influence?

A. Unilateral Mistake
B. Unconscionable Conduct
C. Duress
D. Mutual Mistake

Q14:In a criminal trial, the defense presents evidence that the defendant
committed the crime but lacked the mental capacity to understand the
wrongfulness of the act. Which defense is being raised?

A. Insanity
B. Intoxication
C. Mistake of Fact
D. Duress

Q15: During a search and seizure, law enforcement finds evidence that was not
listed in the search warrant. What legal principle might render this evidence
inadmissible?

A. Fruit of the Poisonous Tree


B. Plain View Doctrine
C. Exigent Circumstances
D. Inevitable Discovery Doctrine

www.careers360.com Answer 85
Q16:In a negligence case, if the plaintiff is found to be partially at fault for their
own injuries, which doctrine is applied to determine the damages?

A. Contributory Negligence
B. Comparative Fault
C. Assumption of Risk
D. Last Clear Chance

Q17: A shipwreck occurs in international waters, and survivors are rescued by a


passing vessel. Which legal principle is likely to govern the rescue vessel's claim
for salvage rights?

A. Doctrine of Necessity
B. Duty to Rescue
C. Doctrine of Laches
D. Salvage Convention

Q18: In a defamation case, which defense may be invoked if the statement in


question is proven to be true?

A. Privilege
B. Consent
C. Justification
D. Fair Comment
Q19. Kamal sends a letter to Vimal offering to sell his flat for Rs. 5,00,000/-
Without knowledge of Kamal's letter, Vimal sends a letter to Kamal saying he
would like to buy Vimal's house for Rs. 5,00,000/- In this situation

(a) there is no binding contract in this case since Vimal has only sent a cross offer.
(b) there is no binding contract since the consideration is inadequate. Kamal's flat would
fetch at least 25,00,000 in the market.
(c) there is a binding contract as Vimal's letter can be treated as an acceptance to
Kamal's offer.
(d) None of the above.

Q20 :In a corporate law context, what doctrine allows a court to disregard the
separate legal personality of a company and hold its shareholders personally
liable?

www.careers360.com Answer 86
A. Ultra Vires Doctrine
B. Piercing the Corporate Veil
C. Doctrine of Indoor Management
D. Doctrine of Constructive Notice

Q21:During a criminal trial, the prosecution introduces evidence obtained


through an illegal wiretap. Which constitutional protection is implicated?

A. Fourth Amendment (Unreasonable Searches and Seizures)


B. Fifth Amendment (Right Against Self-Incrimination)
C. Sixth Amendment (Right to Counsel)
D. Eighth Amendment (Cruel and Unusual Punishment)

Q22: In a bankruptcy proceeding, which type of debt is typically given priority


over other debts in the distribution of assets?

A. Secured Debt

B. Unsecured Debt

C. Priority Debt

D. Subordinated Debt

Q23: During contract negotiations, if one party provides consideration that is


illusory or lacks value, what legal concept may render the contract
unenforceable?

A. Duress

B. Unilateral Mistake

C. Illusory Contract

D. Consideration Doctrine

Q24: In a product liability case, if a defect in a product is shown to exist at the


time it leaves the manufacturer's control, which type of defect is being alleged?

A. Design Defect

B. Manufacturing Defect

C. Marketing Defect

www.careers360.com Answer 87
D. Inadequate Warning Defect

ANALYTICAL REASONING
Q25.Walking at 5/6 of its usual speed, a train is 10 minutes too late. Find the usual
time to cover the journey?

A] 50 min.
B] 10 min.
C] 20 min.
D] 30 min.

Q26. A man travelled from the village to post office at the rate of 25 kmph and
walked backat the rate of 4 kmph. If the whole journey took 5 hrs and 48 min. find
the distance of post office from village?

A] 20 km.
B] 10 km.
C] 30 km.
D] 40 km.

Q27. Shantanu can row upstream at 10 km/h and downstream at 18 km/h. Find the
man’s rate in still water and the rate of the current.

A] 4 km/h
B] 8 km/h
C] 6 km/h
D] 2 km/h

www.careers360.com Answer 88
Q28. A and B together can do a piece of work in 24 days and A alone can
complete the work in 36 days. How long will B alone take to complete the work?

A] 72 days.
B] 62 days.
C] 52 days.
D] 42 days.

Q29. Pipe A fills a tank in 10 hrs, B in 20 hrs and C empties the tank in 60 hrs. if all
three pipes are opened simultaneously then find how much time will the tank
become full?

A] 15/2 hrs
B] 17/2 hrs
C] 11/2 hrs
D] 13/2 hrs

Q30. From 40 liters of 2 : 3 milk-water solution, 8 liters are withdrawn and


replaced by 8 liters of milk. What is the minimum number of times should we
perform the operation of withdrawing and replacing so that the milk
concentration is at least 60%?

A] 1
B] 3
C] more than 3
D] 2

www.careers360.com Answer 89
Q31. The diagonals AC and BD of a parallelogram ABCD intersect each other at
the point O, such that, ∠ DAC = 40° and ∠ BOA = 60°. Then ∠ DBC is

A] 80°

B] 70°

C] 60°

D] 20°

Q32.A solid piece of iron in the form of a cuboid of dimensions (98 cm X 33 cm X


12 cm) is moulded to form a solid sphere. The radius of the sphere is

A] 19

B] 20

C] 21

D] 22

Q33. A metallic solid sphere of radius 9 cm is melted to form a solid cylinder of


radius 4.5 cm. The height of the cylinder is ___ cm.

A] 48

B] 16

C] 32

D] 40

www.careers360.com Answer 90
Q34. IBM and KTC quote for a tender. On the tender opening day, IBM realizes that
their quotations are in the ratio 7:4 and hence decreases its price during
negotiations to make it Rs 1 Lakh lower than KTC's quoted price. KTC realizes
that the final quotes of the two were in the ratio 3:4. What was the price at which
IBM won the bid?

A] Rs 7 Lakh
B] Rs 4 Lakh
C] Rs 3 Lakh
D] Rs 1.5 Lakh

Q35.Directions: In the following figure, the boys who are cricketers and sober are
indicated by which number?

A) 6
B) 5
C) 4
D) 2

Q36.Directions: Some equations are solved on the same basis, find out the
correct answer for the unsolved equation.
If 10 – 3 = 12, 12 – 4 = 13, 14 – 5 = 14, then 16 – 6 = ?

A) 10
B) 15

www.careers360.com Answer 91
C) 16
D) 18

READING COMPREHENSION

Q37.Comprehension:
In the following passage, some words have been deleted. Read the passage
carefully and select the most appropriate option to fill in each blank. We know
infinitely (1)________ about the wealthy people of Egypt than we do about the
ordinary people, as most monuments were made for the rich. Houses in which
ordinary Egyptians lived have not been preserved, and when most people died
they were (2)_______ in simple graves. Most of our traditional sources of
information (3)_______ the Old Kingdom are monuments of the rich (4)_______
pyramids and tombs. Even papyri come mainly from pyramid temples. (5)
________ this does not mean that death was the Egyptians’ only preoccupation.
Select the most appropriate option to fill in blank no. 2.

A) buries
B) buried
C) burying
D) bury

Q38. Select the option that can be used as a one-word substitute for the given
group of words.
Too strong to be defeated or changed.

A) Invincible
B) Headstrong
C) Vigorous
D) Sovereign

Q39.Select the option that expresses the given sentence in passive voice.

You have adopted the plan.

A) The plan should be adopted by you.


B) The plan is adoptod by you.
C) The plan has being adopted by you.
D) The plan has been adopted by you.

www.careers360.com Answer 92
Q40.Select the INCORRECTLY spelt word.

A) Anxiety
B) Preparation
C) Voluntary
D) Stetionery

Q41.Select the most appropriate meaning of the given idiom.


Out of the blue

A) Undoubtedly
B) Unexpectedly
C) Unbelievably
D) Unconcerned

Q42.The following sentence has been split into four segments. Identify the
segment that contains a grammatical error.
The doctor came / after the patient / had / pass away.

A) The doctor came


B) pass away
C) after the patient
D) had

Q43.Select the most appropriate option to fill in the blank.

We _______ off old clothes and bought new ones for Diwali.

A) cost
B) coast
C) caste
D) cast

Q44.Sentences of a paragraph are given below in jumbled order. Arrange the


sentences in the correct order to form a meaningful and coherent paragraph.
A. Louis was enthralled by the concept of a raised dots system.
B. He made up his mind to use the technique to represent an alphabetic code.
C. Blind persons could read large-letter books that are bulky with the aid of this
alphabet code.
D. Additionally, he developed the six-dot cell technology, which was at his
fingertips.

A) DCAB

www.careers360.com Answer 93
B) BCDA
C) ABCD
D) CDBA

Q45.Select the most appropriate meaning of the underlined phrase.

Though very rich, Alexander always saves for a rainy day.

A) Saves money for later


B) Lives like a miser
C) Spends too much
D) Accumulates less wealth

Q46.Select the option that can be used as a one-word substitute for the given
group of words

The murder of a large number of people from a particular nation or ethnic group.

A) Geronticide
B) Honour killing
C) Genocide
D) Mariticide

Q47.Select the INCORRECTLY spelt word.

A) Felicilate
B) Coterminous
C) Ambient
D) Announce

Q48.Sentences of a paragraph are given below in jumbled order. Arrange the


sentences in the correct order to form a meaningful and coherent paragraph.

A. According to him, Rupees 75 lakhs has been spent to make the gymnasium
fully equipped with state-of-the-art fitness machines.
B. The Honorable Commissioner of Police inaugurated the Gymnasium and
Swimming pool and appended the management for their efforts in providing such
amenities to students.
C. A new indoor gymnasium and swimming pool was inaugurated at APJ
International School, Goa.
D. Mr.Mercia, the Principal of the school familiarised the guests with the cost
incurred and the various facilities available.

www.careers360.com Answer 94
A) CBDA
B) BDCA
C) ADBC
D) DABC

GENERAL KNOWLEDGE

Q49.What is the name of the deadly volcano in Indonesia that erupted again in
May 2016 ?

A) Mount Sinabung
B) Mount Merapi
C) Mount Rinjani
D) Mount Agung

Q50.With which of the following country, India has a land dispute near Tawang ?

A) Pakistan
B) China
C) Afghanistan
D) Bangladesh

Q51.The operational period of the 12th Five-Year Plan:

A) 2007-12
B) 2012-17
C) 2015-20
D) 2005-10

Q52. In which session of Congress was the demand of "Poorna Swaraj" accepted
as the aim of the Congress?

A) Calcutta
B) Madras
C) Nagpur
D) Lahore

Q53. French Revolution broke out in the year?

A) 1917
B) 1911

www.careers360.com Answer 95
C) 1789
D) 1790

Q54. Wall Street collapse led to ______.

A) World War II
B) Recession
C) U.S. Attack on Iraq
D) Great Depression

Q55.The ideas of Liberty, Equality and Fraternity, which influenced the Indian
National Movement, were taken from ____.

A) American Revolution
B) Russian Revolution
C) Chinese revolution
D) French Revolution

Q56. Which type of foreign investment is considered unsafe?

A) Foreign Direct Investment (FDI)


B) Portfolio Investment
C) NRI deposits
D) External commercial borrowing

Q57. The term 'Dumping' refers to

A) The sale of a sub-standard commodity


B) Sale in a foreign market of a commodity at a price below marginal cost
C) Sale in a foreign market of a commodity just at marginal cost with too much of profit
D) Smuggling of goods without paying any customs duty

Q58. "Globalisation of Indian Economy" denotes:

A) increase of external borrowings


B) having minimum intervention in economic relations with other countries
C) starting new business units abroad
D) relaxing the programmes of import substitution

www.careers360.com Answer 96
Q59.The speaker of the Lok Sabha has to address his/her letter of resignation to

A) Prime Minister of India


B) President of India
C) Deputy Speaker of Lok Sabha
D) Minister of Parliamentary Affairs

Q60. The red colour of human blood is due to_____________.

A) Myoglobin
B) Haemoglobin
C) Immunoglobulin
D) Haptoglobin

www.careers360.com Answer 97
ANSWER KEY
____________________________________________________________________________________

1 2 3 4 5 6 7 8 9 10
A A C A B A A A D A
11 12 13 14 15 16 17 18 19 20
A D B A A B D C A C
21 22 23 24 25 26 27 28 29 30
A C C B A A A A A D
31 32 33 34 35 36 37 38 39 40
D C A C D B B A D D
41 42 43 44 45 46 47 48 49 50
B B D C A A A A A B
51 52 53 54 55 56 57 58 59 60
B D C D D B C B C B

www.careers360.com Solutions 98
SOLUTIONS
____________________________________________________________________________________

LOGICAL REASONING
1-A
Given:

X is elder than Z, X > Z

Y is younger than Z, Y < Z

Z is elder than W, Z > W

W is younger than X, W < X

From the above data, Y/W < Z < X.

Therefore, X is the eldest.

Hence, first option is correct.

2-A

The Venn diagram is as follows –

Let's analyze the conclusions –


Conclusion (I): No ear is nose – From the Venn diagram, we can see that there is no
overlap between the circles that represent ear and nose. This means that no ear is
nose. Therefore, this conclusion follows.

Conclusion (II): All nose are ears – From the Venn diagram, we can see that there is no
overlap between the circles that represent ear and nose. This means that no nose are
ears. Therefore, this conclusion does not follow.

www.careers360.com Back to Index 99


Conclusion (III): Some hands are noses – From the Venn diagram, we can see that
there is no overlap between the circles that represent hands and nose. This means that
no hands are nose. Therefore, this conclusion does not follow.

So, from the above, it is clear that only Conclusion (I) follows. Hence, the first option is
correct.

3-C

The Venn diagram is as follows –

Let's analyze the conclusions –


Conclusion (I): No cup is spoon – From the Venn diagram, we can see that the circles
that represent cup and spoon do not overlap. But, there is a possibility that they do. So,
no cup is spoon is not a definite statement. Therefore, this conclusion does not follow.

Conclusion (II): Some spoon are cups – From the Venn diagram, we can see that the
circles that represent cup and spoon do not overlap. There could be a possibility but it is
not definite. Therefore, this conclusion does not follow.

So, from the above, it is clear that neither Conclusion I nor II follows. Hence, the third
option is correct.

4-A

a) Both the assertion and reason are true, and the reason is a correct explanation of the
assertion. In a vacuum, there are no air molecules to create air resistance, so the object
does not experience air resistance.

5-B

Both the assertion and reason are true, but the reason is not a correct explanation of
the assertion. While it is true that the Moon has a smaller gravitational force than the
Earth due to its smaller mass, the reason does not provide a direct explanation of this
fact; it oversimplifies the gravitational force concept.

www.careers360.com Back to Index 100


6-A

Statement 1 correctly identifies Earth as the third planet from the Sun. Statement 2 is
incorrect because the Moon is the natural satellite of Earth, not Mars.

7-A

Paris is indeed the capital of France, but Berlin is the capital of Germany.

8-A

According to the given statement –


Argument I: Yes, more tourists arrive due to rock shows. Tourism is good for the local
economy – If rock shows attract more tourists, and tourism is beneficial for the local
economy, allowing rock shows till midnight supports economic growth. Therefore, this
argument strongly supports the given statement.
Argument II: No, local traditions are harmed due to tourism – The statement doesn't
mention any direct conflict between local traditions and tourism due to rock shows.
Therefore, argument II does not support the given statement.

Therefore, the only argument I strongly support the given statement. Hence, the first
option is correct.

9-D

According to the given statement –


Argument I states that Hollywood movies are successful despite not having songs.
While this suggests that songs might not be a crucial factor for success, it doesn't
directly address whether songs should be eliminated from Indian movies.
Argument II points out that songs help to increase the length of the movie. It doesn't
provide a strong reason for keeping songs in Indian movies.

Therefore, neither argument I nor II strongly follows the given statement. Hence, the
fourth option is correct.

10-A

Let's check the options —

First option: EI – LM; E and I both are vowels; L + 1 = M

Second option: AE – RT; A and E both are vowels; R + 2 = T

Third option: IO – WY; I and O both are vowels; W + 2 = Y

www.careers360.com Back to Index 101


Fourth option: OU – DF; O and U both are vowels; D + 2 = F

From the above, it can be said that the first option is different from the other three.

Hence, the first option is correct.

11-A

Let's check the options —

First option: DU; D and U are not opposite in the English alphabetical series.

Second option: KP; K and P are opposite in the English alphabetical series.

Third option: JQ; J and Q are opposite in the English alphabetical series.

Fourth option: GT; G and T are opposite in the English alphabetical series.

Hence, the first option is correct.

12-D

Hence, the correct answer is option 4.

LEGAL REASONING
13-B
To establish undue influence, party A must demonstrate that there was unconscionable
conduct or abuse of power that influenced the contract. This involves one party taking
advantage of the other's vulnerability.

www.careers360.com Back to Index 102


14-A
The defense of insanity asserts that the defendant, at the time of the crime, lacked the
mental capacity to understand the wrongfulness of their actions. It focuses on the
mental state of the accused.

15-A
The "Fruit of the Poisonous Tree" doctrine holds that evidence obtained illegally or
through a violation of the Fourth Amendment is tainted and, therefore, inadmissible in
court.

16-B
Under the comparative fault doctrine, the damages awarded to the plaintiff are reduced
in proportion to their degree of fault. This allows for a more equitable distribution of
responsibility.

17-D
The Salvage Convention provides a legal framework for salvage operations in
international waters, including the rights and responsibilities of salvors.

18-C: The defense of justification in defamation cases involves proving that the
statement is true. Truth is an absolute defense against a defamation claim.

19.A.
There is no binding contract in this case since Vimal has only sent a cross offer. As
there was no acceptance thus no promise. Hence option a is correct.

20-C
Piercing the Corporate Veil" is a legal doctrine that allows a court to hold shareholders
personally liable for the debts or actions of a corporation if the corporate form is abused.

21-A
The Fourth Amendment protects against unreasonable searches and seizures.
Evidence obtained through illegal means, such as an unauthorized wiretap, may be
deemed inadmissible.

22-C
Priority debts in bankruptcy have a higher claim to the debtor's assets than other debts.
Examples include certain taxes and domestic support obligations.

23-C :
An illusory contract occurs when one party's promise is so uncertain that it does not

www.careers360.com Back to Index 103


constitute a commitment. Such contracts lack the mutuality of obligation required for
enforceability.

24:B
A manufacturing defect occurs when a product deviates from its intended design due to
an error in the manufacturing process, making it different from other products of the
same type.

ANALYTICAL REASONING
25-A
New speed = 5/6 of the usual speed
New time taken = 6/5 of the usual time
So, 6/5 of usual time –usual time = 10 min.
Usual time/6 = 10 min
Usual time = 50 min.

26-A
here we use concept of average speed i.e. 2xy/x+y in the above case x is 25 and y is 4
So av speed =(2 x 25 x 4) /25+4 =200/29 kmph.
Total time is 5 +48/60 = 29/ 5 hrs
Hence total distance travelled will be (200/29) x (29/5) = 40km
Required distance = 40/2 = 20 km.

27-A
According to the formula,
1
Man’s rate in still water = 2
(Speed downstream + Speed upstream)
1 28
= 2
(18 + 10) = 2

= 14 km/h
1
Speed of current = 2
(Speed downstream – Speed upstream)

www.careers360.com Back to Index 104


1 8
= 2
(18 – 10) = 2

= 4 km/h
28-A
Work done by A alone in 1 day = 1/36
Work done by both in 1 day = 1/ 24
Hence work done by B alone in 1 day = 1/24- 1/36 = 1/72
And hence B will complete the work in 72 days.

29-A
Part of tank filled by pipe A = 1/10
Part of tank filled by pipe B = 1/20
Part of tank filled by pipe C = - 1/60 (since pipe C is outlet pipe)
Part of tank filled by all three pipes = 1/10 +1/20 – 1/60 =8/60 =2/15
Hence total time to fill = 15/2 hrs

30-D

(1-8/40)n = final / initial


(4/5)n = 2/3
N=2
0.64 ≈ 0.6

31-D

www.careers360.com Back to Index 105


∠ AOB + ∠ AOD = 180° (Linear pair)

60° + ∠ AOD = 180°

∠ AOD = 120°

in triangle AOD, we have

40° + 120° + ∠ ADO= 180°

∠ ADO= 20°

Now,

∠ OBC = ∠ ADO = 20° (alternate interior angle)

∠ OBC = ∠ DBC

32-C

33-A

34-C
IBM initially quoted Rs 7x Lakhs
KTC initially quoted Rs 4x lakhs
IBM final quote = (4x – 1) lakh = > (4x-1)/4x = 3/4 => x = 1

www.careers360.com Back to Index 106


IBM’s bid winning price = Rs. 3 lakh
So x wins the bid at Rs. 3 lakhs

35-D
In the figure given below, the shaded part is the overlapping region for the boys who are
cricketers and sober.

So, the boys who are cricketers and sober are indicated by 2. Hence, the fourth option
is correct.

36-B
Given:
10 – 3 = 12, 12 – 4 = 13, 14 – 5 = 14

On, adding 5 to LHS in each equation we get –


10 – 3 = 12 ⇒ 10 – 3 + 5 = 12
12 – 4 = 13 ⇒ 12 – 4 + 5 = 13
14 – 5 = 14 ⇒ 14 – 5 + 5 = 14

Similarly, 16 – 6 = 15 ⇒ 16 – 6 + 5 = 15

Hence, the second option is correct.

READING COMPREHENSION

37-B
The most appropriate option to fill in blank no. 2 is "buried."

www.careers360.com Back to Index 107


Explanation: In the context of the sentence, "buried" is the past tense form of the verb
"bury," which is used to indicate the action of placing a body in a grave. It is the correct
past tense form to fit the sentence.

The correct sentence is: "Houses in which ordinary Egyptians lived have not been
preserved, and when most people died they were buried in simple graves."

38-A
The one-word substitute for "Too strong to be defeated or changed" is the first option,
"Invincible."

"Invincible" means incapable of being defeated, and it signifies a high degree of


strength or power that cannot be overcome.

The meanings of the other options are as follows:

● Headstrong means energetically willful and determined


● Vigorous means strong, healthy and full of energy.
● Sovereign means possessing supreme or ultimate power.

39-D
The option that expresses the given sentence in passive voice is: "The plan has been
adopted by you."

The original sentence is in active voice with the subject "you" performing the action of
adopting the plan. The passive voice structure is "Object of the active sentence + "to
be" verb (in the corresponding tense) + past participle of the main verb +
Agent(optional)."

In this case, "the plan" becomes the subject, "has been adopted" is the passive
construction of "adopted," and "by you" indicates the agent.

40-D

The fourth option is the incorrectly spelt i.e. "stetionary."

The correct spelling is "stationary," which means not moving or not intended to be
moved.

The meanings of other options are as follows:

● Anxiety: A state of uneasiness or apprehension about future uncertainties


● Preparation: The action or process of making ready or being made ready for use
or consideration

www.careers360.com Back to Index 108


● Voluntary: Done, given, or acting of one's own free will.

41-B

The correct choice is the second option, "Unexpectedly".

The idiom "out of the blue" means "unexpectedly" or "suddenly," and it is used to
describe something that happens without warning or preparation. When something
happens "out of the blue," it means it comes as a surprise or shock, much like
something appearing suddenly from a clear, blue sky. There is no prior indication or
warning.

The meanings of the other options are as follows:

● Undoubtedly means without any doubt.


● Unbelievably means in a manner that is hard to believe.
● Unconcerned means without any concern.

42-B
The segment that contains a grammatical error is the second option, "pass away."

The correct phrase should be "had passed away," not "had pass away." "Had passed"
is the past perfect tense form which is the correct way to express that the patient died
before the arrival of the doctor. The past perfect tense is used to denote the action that
happened earlier than the other action in the past. "Pass away" is a phrasal verb that
means "to die."

Therefore, the correct sentence is,"The doctor came after the patient had passed
away."

43-D
The correct option to fill in the blank is the fourth option, "cast".

The word "cast" in this context means getting rid of or disposing of old clothes. "Cast
off" is the correct phrasal verb to use when indicating the act of discarding something.

Therefore, the correct sentence is, "We cast off old clothes and bought new ones for
Diwali."

44-C
The correct order for the sentences is:

www.careers360.com Back to Index 109


A. Louis was enthralled by the concept of a raised dots system.

B. He made up his mind to use the technique to represent an alphabet code.

C. Blind persons could read large-letter books that are bulky with the aid of this alphabet
code.

D. Additionally, he developed the six-dot cell technology, which was at his fingertips.

The paragraph starts with sentence A which tells us that Louis was enthralled and it is
continued in sentence B where he made up his mind to use the technique, which is
followed by sentence C which tells us that blind persons could read books with the aid
of this alphabet. The sentence D comes in last as it is additional information.

Thus, the correct choice is the third option, "ABCD"

45-A
The most appropriate meaning of the underlined phrase "a rainy day" in this context is
"saves money for later."

Explanation: The phrase "saves for a rainy day" is an idiom that means to set aside
money for unforeseen future expenses or emergencies. It's a prudent financial practice.

The correct sentence is: "Though very rich, Alexander always saves money for a rainy
day."

46-A
The one-word substitute for the given group of words is "genocide."

Explanation: "Genocide" is a term that specifically refers to the systematic and


intentional killing of a large number of people from a particular nation or ethnic group,
with the aim of destroying that group in whole or in part.

The correct sentence is: "The genocide targeted a large number of people from a
particular nation or ethnic group."

47-A

The first option is the incorrectly spelled word i.e. "Felicilate."

The correct spelling is "Felicitate" which means to congratulate or express happiness


for someone's achievements.

www.careers360.com Back to Index 110


The meanings of other options are as follows:

● Coterminous: Sharing a common boundary; coextensive


● Ambient: Relating to the immediate surroundings of something; creating a
certain atmosphere
● Announce: To make known or officially declare something.

48-A

To form a meaningful and coherent paragraph, the sentences should be arranged in the
order of CBDA.

The correct paragraph is as follows:

C. A new indoor gymnasium and swimming pool was inaugurated at APJ International
School, Goa.

B. The Honorable Commissioner of Police inaugurated the Gymnasium and Swimming


pool and appended the management for their efforts in providing such amenities to
students.

D. Mr. Mercia, the Principal of the school familiarized the guests about the cost incurred
and the various facilities available.

A. According to him, Rupees 75 lakhs have been spent to make the gymnasium fully
equipped with state-of-the-art fitness machines.

Explanation: The paragraph begins with the introduction of the new gymnasium and
swimming pool (sentence C), Then, it mentions the inauguration ceremony and praise
from the Commissioner of Police (sentence B), followed by information about the cost
and facilities provided (sentence D) and finally, it provides details about the expenses
(sentence A).

GENERAL KNOWLEDGE
49-A
Correct answer is Mount Sinabung
In Indonesia there are many active volcanoes and Mount Sinabung is one of the
deadliest amongst them. It is located on the Indonesian island of Sumatra. Mount

www.careers360.com Back to Index 111


Rinjani is located on the Island of Lombok. Mount Merapi is the most active volcano in
Indonesia. Mount Agung is also an active volcano in bali, Indonesia.

50-B
Correct answer is China
India shares it's largest land boundary with Bangladesh and then China. Tawang is a
region in the north-eastern state Arunachal pradesh near LAC. China claims that this
region belongs to them. In recent past, Chinese army intruded in this region and clashes
takes place between Indian army and Chinese army. Indian army gave befItting reply to
the chinese army.

51-B
The correct option is 2012-17.

The Twelfth Five-Year Plan was in force from 2012 to 2017 and began on April 1, 2012.
The Twelfth Plan's tagline was "Faster, More Sustainable, and More Inclusive Growth".
The National Development Council (NDC) was the ultimate decision-maker who granted
the five-year plans its final approval.

52-D
The answer is Lahore.

Purna Swaraj was established as the aim of the Indian National Congress in its Lahore
Session in December 1929. Jawaharlal Nehru served as the president of this session.
The Indian National Congress decided to observe January 26, 1930, as Purna Swaraj
Day. This day has special significance in the Indian history of independence, as on the
same date, i.e., January 26, 1950, the Constitution of India was enforced.

53-C
The correct answer is 1789.

The French Revolution was a time of profound social and political transformation in
France that affected not just that country, but also the entire Europe. The Revolution
also brought about the establishment of many modern democratic principles, such as
citizenship and division of powers.

54-D
The correct answer is the Great Depression.

The Great Depression began in 1929 after the Wall Street crashed and the 20th
century's worst economic downturn occurred. It began in the US in 1929 and quickly

www.careers360.com Back to Index 112


spread to other countries. A protracted period of extreme poverty, despair and
significant unemployment characterised the Great Depression. The Wall Street is eight
blocks long and lies in the financial district of Lower Manhattan. It's situated in New York
City.

55-D
The answer is the French Revolution.

The Declaration of the Rights of Man and the Citizen was adopted by the French
National Assembly in 1789 and contained the principles of Liberty, Equality and
Fraternity. The right to liberty, property, security and resistance to persecution are
among the rights that are guaranteed to all at their birth under the Declaration of
Independence.

56-B
The correct option is Portfolio Investment.

A portfolio investment is an investment made in a collection of assets rather than a


single asset to earn returns. Examples of these assets include debt, stock, mutual
funds, bitcoins, and derivatives.

Here are some essential details regarding investing in a portfolio:


1. Investing in a portfolio entails a variety of assets to generate returns that are
consistent with the investor's risk tolerance.
2. There are four different sorts of portfolio investments: low-risk, medium-risk, high-risk
and risk-free.
3. Allocating money to a variety of financial instruments, such as mutual funds, stocks,
bonds, exchange-traded funds (ETFs), or any other kind of securities, to generate a
return in line with the risk profile and return expectations, is known as portfolio
investment management.

57-C
The answer is the sale in a foreign market of a commodity at a price below
marginal cost.

The phrase "dumping" refers to the practice of selling products in a foreign market at a
price lower than their typical worth. It can be performed to drive out local manufacturers
or acquire market share. Dumping has the potential to hurt domestic industry and
consumers. When overseas companies sell items below cost, domestic companies may
find it challenging to compete with one another. This might result in reduced
employment and increased consumer prices.

www.careers360.com Back to Index 113


58-B
The correct option is having minimum intervention in economic relations with other
countries.

The process of connecting the Indian economy with the global economy through
liberalisation and privatisation policies is referred to as "globalisation of the Indian
economy". Through this procedure, foreign enterprises are now able to participate in the
Indian market, and Indian companies are now allowed to grow internationally. The
social, financial, cultural and political spheres of India have all been significantly
impacted by the country's economy worldwide.

59-C
The correct answer is Deputy Speaker of Lok Sabha

Articles 93-96 of the Indian Constitution deal with the powers and functions of the
Speaker and Deputy Speaker of the Lok Sabha. As per the convention, the Speaker of
Lok Sabha is from the ruling party and the Deputy Speaker is from the opposition. The
Speaker of Lok Sabha is responsible for the orderly functioning of the Lower House,
conducting regular proceedings and maintaining law and order. As per the Constitution,
the Speaker of Lok Sabha can resign by addressing his resignation to the Deputy
Speaker of Lok Sabha.

60-B
The correct option is Haemoglobin

The red colour of human blood is caused by a molecule known as haemoglobin. The
protein haemoglobin is present in red blood cells and delivers oxygen from the lungs to
the rest of the body's tissues and organs. Haemoglobin also aids in the transfer of
carbon dioxide, a byproduct of metabolism, back to the lungs for expiration. The red
colour of haemoglobin is a result of its iron content. When it binds with oxygen in the
lungs, it forms a complex called oxyhaemoglobin, which is bright red.

www.careers360.com Back to Index 114


MOCK TEST - 4
______________________________________________________________________

LOGICAL REASONING
Q1. Direction: In the following question below, some statements are followed by
some conclusions. Taking the given statements to be true, even if they seem to
be at variance from commonly known facts, read all the conclusions and then
decide which of the given conclusions logically follows from the given
statements.
Statements:
(I) No women can vote.
(II) Some women are politicians.
Conclusions:
(I) Male politicians can vote.
(II) Some politicians can vote.

A) Conclusion I follows
B) Conclusion II follows
C) Neither I nor II follows
D) Both I and II follow

Q2.Direction: In the following question, two statements are given followed by two
conclusions I and II. You have to consider the two statements to be true even if
they seem to be at variance from commonly known facts. You have to decide
which of the given conclusions, if any, follow from the given statements.
Statements
All men are aggressive.
All children are aggressive.
Conclusions
I. Men are children
II. Children are men.

A) Only conclusion II follows


B) Both conclusions I and II follow
C) Neither conclusion I nor conclusion II follows
D) Only conclusion I follow

www.careers360.com Answer 115


Q3. Assertion: The greenhouse effect contributes to global warming.

Reason: Greenhouse gases trap heat energy in the Earth's atmosphere.

A) Both the assertion and reason are true, and the reason is a correct explanation of the
assertion.

B) Both the assertion and reason are true, but the reason is not a correct explanation of
the assertion.

C) The assertion is true, but the reason is false.

D) The assertion is false, but the reason is true.

Q4.Assertion: Fossils are found in sedimentary rocks.

Reason: Sedimentary rocks are formed by the accumulation of sediments over


time.

A) Both the assertion and reason are true, and the reason is a correct explanation of the
assertion.

B) Both the assertion and reason are true, but the reason is not a correct explanation of
the assertion.

C) The assertion is true, but the reason is false.

D) The assertion is false, but the reason is true.

Q5.According to the given statements, Which of the following options is correct.

Statement 1:The capital of France is Paris.

Statement 2:The capital of Germany is Berlin.

A) Statement 1 is correct, but Statement 2 is incorrect.

B) Statement 2 is correct, but Statement 1 is incorrect.

C) Statement 1 is incorrect.

D) Statement 2 is incorrect.

www.careers360.com Answer 116


Q6.According to the given statements, Which of the following options is correct

Statement 1:The chemical symbol for gold is Go.

Statement 2: The chemical symbol for silver is Ag.

A) Statement 1 is correct, but Statement 2 is incorrect.

B) Statement 2 is correct, but Statement 1 is incorrect.

C) Statement 1 is incorrect.

D) Statement 2 is incorrect.

Q7. Direction: In the following question, a statement is given, followed by two


arguments, I and II. You have to consider the statement to be true, even if it
seems to be at variance from commonly known facts. You have to decide which
of the given arguments, if any, follows from the given statement.

Statement:
"Should standardized tests be the primary criteria for college admissions?"
Arguments:
I. Yes, standardized tests provide a fair and objective way to assess students.
II. No, they can perpetuate inequality and disadvantage underprivileged students.

A) Only Argument I follows.


B) Only Argument II follows.
C) Both Arguments I and II follow.
D) Neither Argument I nor Argument II follows.

Q8. Direction: In the following question, a statement is given, followed by two


arguments, I and II. You have to consider the statement to be true, even if it
seems to be at variance from commonly known facts. You have to decide which
of the given arguments, if any, follows from the given statement.
Statement:
"Should public transportation be made free for all citizens?"
Arguments:
I. Yes, it can reduce traffic congestion and lower pollution.
II. No, it will lead to increased taxes to cover the cost.

A) Only Argument I follows.


B) Only Argument II follows.

www.careers360.com Answer 117


C) Both Arguments I and II follow.
D) Neither Argument I nor Argument II follows.

Q9.Direction: In the following question, select the odd letter from the given
alternatives.

A) IM
B) DH
C) MS
D) UY

Q10.Direction: In the following question, select the odd letter cluster from the
given alternatives.

A) AD
B) VY
C) MQ
D) IL

Q11. Ben says to Zed, “You are the only nephew of my only maternal uncle. My
mother has only one sibling.” How is Ben related to Zed?

A) Brother
B) Sister
C) Cousin
D) Cannot be determined

Q12.Direction: C has more money than E and E has more money than B. C is the
second richest after A. Who has the least money out of A, B, C, and E?

A) C
B) A
C) B
D) E

www.careers360.com Answer 118


LEGAL REASONING
Q13: In a defamation case, a public figure sues a media outlet for publishing false
information about them. To prevail, what standard of fault must the public figure
generally prove?

A. Strict Liability
B. Negligence
C. Recklessness
D. Intentional Misconduct

Q14: During a criminal trial, the defense presents evidence that the defendant
committed the crime under extreme emotional distress. What defense is being
raised?

A. Insanity
B. Duress
C. Provocation
D. Mistake of Fact

Q15:In a contract dispute, party A argues that the contract is void due to mutual
mistake. What must A demonstrate to establish mutual mistake?

A. One Party's Unilateral Mistake


B. Fraudulent Misrepresentation
C. Lack of Consideration
D. A Mistake Shared by Both Parties

Q16:During a search and seizure, law enforcement relies on a search warrant that
is later found to be based on false information. What legal principle might render
the evidence obtained inadmissible?

A. Good Faith Exception


B. Inevitable Discovery Doctrine
C. Exclusionary Rule
D. Fruit of the Poisonous Tree

www.careers360.com Answer 119


Q17: In a discrimination case, if an employer can demonstrate a legitimate,
nondiscriminatory reason for an adverse employment action, what burden shifts
to the employee?

A. Burden of Proof
B. Burden of Production
C. Burden of Persuasion
D. Burden of Explanation

Q18.During a criminal trial, if the defense argues that the defendant committed
the crime to prevent imminent harm, what defense is being raised?

A. Necessity
B. Self-Defense
C. Duress
D. Entrapment

Q19: In a negligence case, if the plaintiff voluntarily assumed a known risk, which
defense may be raised by the defendant?

A. Contributory Negligence
B. Assumption of Risk
C. Comparative Fault
D. Last Clear Chance

Q20: In a property law context, if someone openly and continuously uses another
person's land without permission for a certain period, what legal concept might
grant them ownership?

A. Adverse Possession
B. Eminent Domain
C. License
D. Escheat

Q21:In a criminal trial, if a defendant is found not guilty by reason of insanity,


what legal consequences may follow?

A. Acquittal
B. Incarceration

www.careers360.com Answer 120


C. Civil Commitment
D. Community Service

Q22: In a family law case, if a court grants sole legal custody to one parent, what
does this typically mean?

A. Shared Decision-Making
B. Physical Custody
C. Visitation Rights
D. Exclusive Authority to Make Legal Decisions

Q23: In a contract dispute, if one party breaches the contract but the other party
fails to mitigate their damages, what might happen to the damages awarded?

A. Damages Increased
B. Damages Reduced
C. Damages Stay the Same
D. Liquidated Damages Imposed

Question 24: In a corporate law context, what type of merger involves the
combination of two companies in the same industry at the same stage of
production?

A. Horizontal Merger
B. Vertical Merger
C. Conglomerate Merger
D. Hostile Merger

ANALYTICAL REASONING
Q25. A thief is spotted by a policeman from a distance of 100 metres. When the
policeman starts the chase, the thief also starts running. If the speed of the thief
be 8 km/hr and that of the policeman 10 km/hr, how for the thief will have run
before he is overtaken?

A] ⅖ km
B] ⅗ km

www.careers360.com Answer 121


C] 2km
D} 5km

Q26. A bike crosses a bridge with a speed of 108 km/h. What will be the length of
the bridge if the bike takes 8 minutes to cross the bridge?
A] 14400 m
B] 12400 m
C] 13400 m
D] 14000 m

Q27. A man can row 8 km/hr in still water. When the river is running at 2 km/hr, it
takes him 3 hrs 12 min to row to a place and back. How far is the place?

A] 6 km.
B] 8 km.
C] 3 km
D] 12 km

Q28. A and B together complete a work in 36 days, B and C together complete in


48 days, and A and C completes in 72 days. How long would each take to do the
job?

A] 388 days.
B] 588 days.
C] 288 days.
D] 200 days.

Q29. Three pipes A, B and C together can fill an empty tank in 4 hours. After 1 hr,
C is closed and the tank is full in 6 more hrs. find the time in which C alone can
fill the empty tank?
A] 8 hrs.
B] 10 hrs.

www.careers360.com Answer 122


C] 9 hrs.
D] 6 hrs.

Q30. A 20 litre mixture of milk and water contains milk and water in the ratio 3 : 2.
10 litres of the mixture is removed and replaced with pure milk and the operation
is repeated once more. At the end of the two removals and replacements, what is
the ratio of milk and water in the resultant mixture?

A] 17 : 3
B] 9 : 1
C] 3 : 17
D] 5 : 3

Q31. ABCD is a parallelogram, consider the adjoining figure, The measure of


∠CBD is

A] 60°

B] 50°

www.careers360.com Answer 123


C] 20°

D] 30°

Q32.A cubical ice-cream brick of edge 44 cm is to be distributed among some


children by filling ice-cream cones of radius 4 cm and height 14 cm up to its brim.
How many children will get the ice-cream cones?

A] 163

B] 263

C] 363

D] 463

Q33. A mason constructs a wall of dimensions (540 cm X 300 cm X 350 cm) with

bricks, each of size (22.5 cm X 11.25 cm X 17.5 cm) and it is assumed that
space is covered by the mortar. Number of bricks used to construct the wall is

A] 11000

B] 11200

C] 11500

D] 11400

Q34. Rs 4830 is divided among Abhishek, Dishant and Prashant such that if
Abhishek's share diminishes by Rs 5, Dishant's share diminishes by Rs 10 and
Prashant's share diminishes by Rs 15, their shares will be in the ratio 5:4:3. Find
the Dishant's original share

A] 1610
B] 2010
C] 2410
D] 1590

www.careers360.com Answer 124


Q35 Directions: If 2 + 3 + 5 = 30, 3 + 4 + 6 = 72, 5 + 6 + 2 = 60, then 5 + 4 + 0 = ?
A) 40
B) 30
C) 0
D) None

Q36.Directions: A word is represented by only one set of numbers as given in any


one of the alternatives. The sets of numbers given in the alternatives are
represented by two classes of alphabets as in the two matrices, given below. The
columns and rows of Matrix (I) are numbered from 0 to 4 and that of Matrix (II) are
numbered from 5 to 9. A letter from these matrices can be represented first by its
row and next by its column, e.g. 'P' can be represented by 12, 24, etc., and 'O' can
be represented by 57, 68, etc. Similarly, you have to identify the set for the word
given in the question.
WARD
Matrix I
0 1 2 3 4

0 P K E A S

1 A S P K E

2 K E A S P

3 S P K E A

4 E A S P K

Matrix II

5 6 7 8 9

5 R D O W C

6 W C R D O

7 D O W C R

8 C R D O W

www.careers360.com Answer 125


9 O W C R D

A) 58, 10, 67, 75


B) 77, 22, 67, 88
C) 96, 42, 79, 87
D) 89, 34, 86, 96

READING COMPREHENSION

Q37. Select the most appropriate ANTONYM of the underlined word.


The musician gave a dismal performance of a few old melodies at last night's
concert.

A) smart

B) melancholy

C) exhausting

D) lively

Q38. Select the most appropriate synonym for the highlighted word.

His modest nature made all the difference.

A)courageous

B) horrific

C) shy

D) dry

Q39. Select the option that can be used as a one-word substitute for the given
group of words. An outer protective layer of a tree.

A) Bark

B) Shrub

C) Rind

D) Peel

www.careers360.com Answer 126


Q40. Parts of the following sentence have been given as options. Select the
option that contains an error.
My mother is a honest and well-known woman in the society.

A) in the society

B) My mother

C) is a honest

D) and well known woman

Q41.Sentences of a paragraph are given below. While the first and the last
sentences (S1 and S6) are in the correct order, the sentences in between are
jumbled up. Arrange the sentences in the correct order to form a meaningful and
coherent paragraph.
S1. A moment comes
A. long suppressed
B. and when the soul of a nation
C. which comes but rarely in history
D. when we step out from the old to the new, when an age ends
S6. finds utterance.

A) CBAD

B) CDBA

C) ABCD

D) DCBA

Q42. Identify the sentence that contains no spelling errors.

A)Manisha got overwhelmed by the tremendus offer by a company to work in


collaboration with internet personel.

B)Manisha got overwhelmed by the tremendous offer by a company to work in


collaboration with internet personnel.

C) Manisha got overwellmed by the tremendous offer by a company to work in


collaborasion with internet personnel.

D) Manisha got overwhelmed by the tremendus offer by a company to work in collaborat

www.careers360.com Answer 127


Q43. Select the most appropriate meaning of the given idiom.
In a soup

A) Extremely short of money

B) To exhibit cowardice

C) To get nothing

D) In trouble

Q44. Select the most appropriate ANTONYM of the underlined word.


They praised her for her bravery.

A) Acclaimed

B) Rebuked

C) Defended

D) Assaulted

Q45. Select the most appropriate ANTONYM of the given word.


Intentional

A) Cordial

B) Unplanned

C) Amiable

D) Inanimate

Q46 Select the most appropriate ANTONYM of the given word.


Abandon

A) Quit

B) Surrender

C) Shorten

D) Continue

www.careers360.com Answer 128


Q47. Directions: In the given question, a sentence is given with a blank to be filled
in with an appropriate word. Four alternatives are suggested for each question.
Choose the correct alternative out of the four alternatives.

The masks worn by the men helped them ______ their identity.

A) conceal

B) congeal

C) masquerade

D) cheat

Q48.Directions: In the given question, a sentence is given with a blank to be filled


in with an appropriate word. Four alternatives are suggested for each question.
Choose the correct alternative out of the four alternatives.

On the _________ occasion of Lakshmi Puja, the Mathurs bought a new car.

A) officious

B) auspicious

C) fotutitous

D) prosperous

GENERAL KNOWLEDGE
Q49.Which one of the following cities of Iraq is located on Tigris river ?

A) Baghdad
B) Mousul
C) Kirkuk
D) Basra

Q50.Which of these straits separates Asia from Africa ?

A) Malacca
B) Hormuz
C) Bab-al-Mandeb

www.careers360.com Answer 129


D) Bosphorus

Q51. Which one of the following countries was the first to establish a modern
democracy?

A) France

B) England

C) America

D) India

Q52. Which of the following is the name of the US Parliament?

A) Diet

B) Senate

C) Congress

D) House of Commons

Q53. The first country to make the constitution is __________.

A) India

B) England

C) USA

D) Sri Lanka

Q54. In the Indian constitution, the method of election of the President has been
taken from which country?

A) Britain

B) USA

C) Ireland

D) Australia

Q55. Which of the following countries do not have a written constitution?

www.careers360.com Answer 130


A) United Kingdom

B) Australia

C) United States of America

D) Bangladesh

Q56. Which cell organelles are the site of photosynthesis and also contain
chlorophyll?

A) Chloroplast

B) Vacuole

C) Cytoplasm

D) Nucleolus

Q57. The suicidal bags of the cell are

A) Lysosomes

B) Ribosomes

C) Dictyosomes

D) Phagosomes

Q58. What is the main purpose of white blood corpuscles?

A) To carry nutrients

B) To combat infection

C) To carry oxygen

D) To give strength

Q59.Which of the following is not connective tissue?

A) Bone

B) Cartilage

C) Blood

www.careers360.com Answer 131


D) Skeletal muscle

Q60.The chemical component that is invariably found in all viruses is

A) Proteins

B) Lipids

C) DNA

D) RNA

www.careers360.com Answer 132


ANSWER KEY
______________________________________________________________________

1 2 3 4 5 6 7 8 9 10
C C A A A A C C C C
11 12 13 14 15 16 17 18 19 20
B C C C D C C A B A
21 22 23 24 25 26 27 28 29 30
C D B A A D D C A B
31 32 33 34 35 36 37 38 39 40
B C B A C B D D A C
41 42 43 44 45 46 47 48 49 50
B B D B B D A B A C
51 52 53 54 55 56 57 58 59 60
C C C C A A A B D A

www.careers360.com Solutions 133


SOLUTIONS
______________________________________________________________________

LOGICAL REASONING
1-C
(I) No women can vote.
(II) Some women are politicians.
Let's analyze the conclusions
Conclusion (I): Male politicians can vote.
This conclusion isn't directly supported by the given statements. We know that no
woman can vote, but the statements don't give us any information about whether male
politicians can vote or not. So, this conclusion does not follow.
Conclusion (II): Some politicians can vote.
This conclusion isn't directly supported by the given statements. The statements do not
mention anything about whether politicians can vote or not. So, this conclusion does not
follow.

Hence, the third option is correct.

2-C

Given:
The possible Venn diagram according to the statements given in the question is –

Let's analyze the conclusions –


Conclusion I: Men are children

www.careers360.com Back to Index 134


From the Venn diagram, it is clear that there is no overlap between men and children.
So, this conclusion does not follow.
Conclusion II: Children are men.
From the Venn diagram, it is clear that there is no overlap between children and men.
So, this conclusion does not follow.

So, both the conclusion does not follow from the given statement. Hence, the third
option is correct.

3-A

The greenhouse effect does contribute to global warming because it involves


greenhouse gases trapping heat energy in the atmosphere, leading to a rise in
temperatures.

4-A

Fossils are typically found in sedimentary rocks because these rocks form by the
accumulation of sediments, which can bury and preserve organic remains, leading to
fossilization.

5-A

Paris is indeed the capital of France, but Berlin is the capital of Germany.

6-A
The chemical symbol for gold is Au, and the chemical symbol for silver is Ag.

7-C
Argument I highlights the objectivity of standardized tests, while Argument II points out
the potential negative consequences, such as perpetuating inequality.

8-C
Both Argument I and Argument II follow. Argument I supports the idea of free public
transportation for the benefits it can bring, such as reducing traffic congestion and
pollution. Argument II points out a potential drawback, which is the need for increased
taxes to cover the cost, highlighting a possible financial implication.

9-C
First option: IM; I + 4 = M

Second option: DH; D + 4 = H

www.careers360.com Back to Index 135


Third option: MS; M + 6 = S (The third option is different from the other three.)

Fourth option: UY; U + 4 = Y

Hence, the third option is correct.

10-C

Let's check the options –

First option: AD; A + 3 = D

Second option: VY; V + 3 = Y

Third option: MQ; M + 4 = Q (The third option is different from the other three.)

Fourth option: IL; I + 3 = L

Hence, the third option is correct.

11-B

The term “only nephew” implies that Ben is a woman. Hence, option 2 is the correct
answer.

12-C
C has more money than E and E has more money than B, i.e. C > E > B
C is the second richest after A, i.e. A > C > E > B

So, B has the least money. Hence, the third option is correct.

www.careers360.com Back to Index 136


LEGAL REASONING

13-C
Public figures typically need to prove that the false statement was made with actual
malice, meaning with reckless disregard for the truth. Recklessness involves a
conscious disregard for the accuracy of the statement.

14-C
The defense of provocation, also known as "heat of passion" or "diminished capacity,"
involves arguing that the defendant committed the crime while under extreme emotional
distress, which led to a temporary loss of self-control.

15-D
To establish mutual mistake, both parties must have made a mistake regarding a
fundamental aspect of the contract. This shared misconception must go to the root of
the agreement.

16- C
The Exclusionary Rule prohibits the use of evidence obtained in violation of the Fourth
Amendment. If a search warrant is based on false information, the evidence may be
excluded under this rule.

17-C
In a discrimination case, if the employer provides a legitimate reason, the burden of
persuasion shifts back to the employee to prove that the employer's stated reason is a
pretext for discrimination.

18-A
The necessity defense is invoked when a defendant commits a crime to prevent greater
harm. It requires demonstrating that the harm prevented was more significant than the
offense committed.

19-B
Assumption of risk is a defense in negligence cases where the plaintiff, with knowledge
of the risk, voluntarily accepts and proceeds in the face of that risk. It may limit or
eliminate the defendant's liability.

20-A
Adverse possession allows an individual to acquire legal ownership of land by openly
and continuously using it without the owner's permission for a specified period. This

www.careers360.com Back to Index 137


concept is based on the idea that long-term use can result in a transfer of property
rights.

21–C
If a defendant is found not guilty by reason of insanity, they may be subject to civil
commitment for psychiatric treatment rather than incarceration.

22-D
Sole legal custody grants one parent exclusive authority to make legal decisions on
behalf of the child, such as those related to education, healthcare, and religious
upbringing.

23-B
The duty to mitigate requires the non-breaching party to take reasonable steps to
minimize their damages. Failure to do so may result in a reduction of the damages
awarded.

24-A
A horizontal merger occurs when two companies in the same industry and at the same
stage of production combine their operations. This type of merger is subject to antitrust
scrutiny.

ANALYTICAL REASONING
25-A
Relative speed of the policeman

Time taken by policeman to cover 100 m =

In hrs, the thief covers a distance of

26-D
Here, length of the bridge = Distance travelled by bike in 8 minutes
= Speed X Time
Given that speed = 108 km/h = 108 X 518 m/s = 30 m/s

www.careers360.com Back to Index 138


Time = 8 minutes = 8 X 60 = 481 seconds
Length of the bridge = 30 X 480 = 14400 m

27-D .
Speed downstream = (8 + 2) km/hr = 10 km/hr.
Speed upstream = (8 – 2) km/hr = 6 km/hr.
Let the required distance be x km. Then,

Required distance = 12 km.

28-C
A+B work in 1 day = 1/36…………..(1)
B+C work in 1 day = 1/48…………..(2)
A+C work in 1 day = 1/72…………..(3) Adding (1)+(2)+(3), we get
2 (A+B+C)’s 1 day work = 1/36+1/48+1/72 = 9/144 = 1/ 16
And hence (A+B+C)’s 1 day work = 1/ 32
Now 1 day work of A = 1/32-1/48 =1/96 therefore A completes the work in 96
days.
Now 1 day work of B = 1/32-1/72 =5/288 therefore A completes the work in 288/5
days.
Now 1 day work of C = 1/32-1/36 =1/288 therefore A completes the work in 288
days.

29-A
Tank filled in 1 hr by all three pipes = 1/4
Remaining part=3/4 which is filled by A and B together in 6 hrs
Hence complete tank will be filled by A and B =6x(4/3)=8 hrs
So we have part of tank filled by A and B in 1 hr i.e. = 1/8
Part that C alone can fill = 1/4- 1/8 =1/8. So
C alone can fill it in 8 hrs.

www.careers360.com Back to Index 139


30-B
(1-10/20)2 = x/8
Y= 10; n= 2
x/8 = ¼ , x=2
Final quantity of water = 2
=milk = 20 -2
=18
=Ratio = 18/2 = 9:1

31-B

∠ A = ∠ C = 60° (Opposite angle of parallelogram)

In triangle BCD

∠ BDC + ∠ C + ∠ DBC = 180°

70° + 60° + ∠ DBC = 180°

∠ DBC = 50°

32-C:

Let the number of cones be n. Then

33-B

www.careers360.com Back to Index 140


34-A

x + y +z = 4830
x, y, z are diminished by 5 , 10, 15 their net share Rs 4800
y = (4/12)* 4800 = 1600
y = 1600 + 10 = 1610

35-C
Given:
2 + 3 + 5 = 30
3 + 4 + 6 = 72
5 + 6 + 2 = 60

Here, the pattern followed is –


2 + 3 + 5 = 30 → 2 × 3 × 5 = 30
3 + 4 + 6 = 72 → 3 × 4 × 6 = 72
5 + 6 + 2 = 60 → 5 × 6 × 2 = 60
Similarly, follow the same pattern for – 5 + 4 + 0 = ? → 5 × 4 × 0 = 0

So, 0 is the required answer. Hence, the third option is correct.

36-B
Given:
1, 2, 6, 24, 120, ?
Multiply the previous term of the series by consecutive natural numbers from 2
onwards.
1×2=2
2×3=6
6 × 4 = 24
24 × 5 = 120
120 × 6 = 720

So, the missing number is 720. Hence, the second option is correct.Therefore, WARD
can be represented by 58, 10, 67, 75. Hence, the first option is correct.

www.careers360.com Back to Index 141


READING COMPREHENSION

37-D

The most appropriate antonym of the underlined word dismal is lively.

Explanation: Dismal means gloomy or depressing, while lively means energetic and full
of life. In the context of a musical performance, lively indicates a positive and
enthusiastic presentation, which contrasts with a dismal one.

Smart: Intelligence that often brings clarity but sometimes invites complexity.
Melancholy: A deep, pensive sadness that colours perceptions and emotions.
Exhausting: Draining of energy, physically or emotionally, due to prolonged effort or
stress.

The correct sentence is: The musician gave a lively performance of a few old melodies
at last night's concert.

38-D
The most appropriate synonym for the highlighted word modest is shy.

Explanation: This is the most appropriate synonym. Shy means being reserved or not
wanting to draw attention to oneself, which is similar to being modest. In the context of
the sentence, it describes a person who is unassuming and not seeking attention.

● Courageous: It means having or showing courage, which is the opposite of


modesty.

● Horrific: This word means causing horror or shock. It is not a synonym for
modest at all. In fact, it has a very different meaning.

● Dry: It refers to something that lacks moisture or is boring and uninteresting.

www.careers360.com Back to Index 142


39-A

The correct answer is bark.

Explanation:

Shrub: A small, woody plant, typically with multiple stems and a shorter height than a
tree.

Rind: The tough, outer layer or bark of a fruit, vegetable, or tree.

Peel: The outer skin or covering of a fruit or vegetable, often removed before eating or
cooking.

Bark refers to the tough protective outer sheath of the trunk, branches, and twigs of a
tree or woody shrub. Thus, the outer protective layer of a tree is called bark.

40-C
The error lies in the third option. The error in this option is the incorrect use of the
indefinite article a before honest. The correct form should be an honest because
honest begins with a vowel sound.

So, the correct sentence should be: My mother is an honest and well-known woman in
the society.

41-B

The correct order of the sentences to form a meaningful and coherent paragraph is:
CDBA
C. which comes but rarely in history

D. when we step out from the old to the new, when an age ends

B. and when the soul of a nation

A. long suppressed

The paragraph is describing a significant moment in the history of a nation when a


long-suppressed soul or spirit finds expression as an old age or era comes to an end,
giving way to the new. This moment is portrayed as a rare occurrence in history,
signifying a significant transformation or change in the nation's identity or direction.

www.careers360.com Back to Index 143


42-B

The sentence that contains no spelling errors is: Manisha got overwhelmed by the
tremendous offer by a company to work in collaboration with internet personnel.

● Tremendous is the correct spelling of the word meaning very large or great.

● Personnel is the correct spelling for the word referring to the staff or employees
of a company or organization.

● Collaboration is the correct spelling for the act of working together or


cooperating with others.
● The other options contain various spelling errors.

43-D

The most appropriate meaning of the idiom In a soup is In trouble.

Explanation: The idiom In a soup is used to describe a situation where someone is


facing trouble or difficulty, not specifically a financial issue. In trouble conveys the idea
that someone is facing difficulties, challenges, or problems.

44-B

The most appropriate antonym of the underlined word is rebuked.

Explanation: Praised means to express approval, admiration, or compliments for


someone's actions, qualities, or achievements. Rebuked means to criticise or
reprimand someone for their actions or behaviour.

● Acclaimed: It means to be praised or celebrated.

● Defended: It means to protect or support.

● Assaulted: It means to attack or physically harm.

www.careers360.com Back to Index 144


45-B

The most appropriate antonym of the given word Intentional is unplanned.

Explanation: Unplanned is the antonym that best represents something not


intentionally done, in contrast to intentional, which means something done with
purpose or by design.

● Cordial: It means warm, friendly, and polite.

● Amiable: It means friendly and pleasant in disposition.

● Inanimate: It refers to objects that lack life or consciousness, like non-living


things.

46-D

The most appropriate antonym of the given word Abandon is Continue.

Explanation: Continue means to persist in doing something or to keep an activity or


process going without interruption, which is the opposite of abandon which means to
give up, leave behind, or stop supporting or caring for something or someone.

● Quit: It means to stop doing something or resign from a job or activity.

● Surrender: It means to give up or yield to an opponent.

● Shorten: It means to make something shorter or reduce its length.

47-A
The correct choice is the first option: conceal.

Explanation: It means to hide or keep something secret, which is the right word in this
context.

Congeal- It means to solidify or thicken, usually referring to liquids or substances


becoming more solid.

Masquerade- It refers to wearing a disguise or pretending to be someone else.

Cheat- It means to deceive or act dishonestly to gain an advantage.

www.careers360.com Back to Index 145


So, the sentence should be: "The masks worn by the men helped them conceal their
identity."

48-B
The correct choice is the second option: auspicious

Explanation: It means favourable, lucky, or indicating a positive outcome. It is the right


word to describe the occasion of Lakshmi Puja.

Officious- It means overly eager to offer help or advice, often in an annoying or


intrusive way.

Fortuitous- It means happening by chance or luck.

Prosperous- It means having success or wealth,

So, the sentence should be: "On the auspicious occasion of Lakshmi Puja, the Mathurs
bought a new car."

GENERAL KNOWLEDGE
49-A
Correct answer is Baghdad
Tigris river is the second largest river in the western asia. Baghdad is the capital of Iraq
and Tigris river flows through it. Tigris river flows through Iraq and drains into the
Persian gulf.

50-C
Correct answer is Bab-al-Mandeb
Bab-al-Mandeb strait separates Asia from Africa and it is located between Yemen and
Djibouti which is in the African continent. Most of the trade is done through this strait by
Asian countries with the Western countries. Bosphorous strait is between the Turkey,
Hormuz strait is found between Iran and Oman, Malacca strait is found betweeen
Malaysia and Indonesia.

51-C
The correct option is America

The United States of America was the first country to establish a modern democracy.
It gained independence from British rule through the American Revolutionary War,
which lasted from 1775 to 1783. The United States Constitution was drafted in 1787

www.careers360.com Back to Index 146


and is considered one of the first examples of a modern democratic constitution. A
system of checks and balances and a division of powers between the legislative,
executive, and judicial branches were established by the Constitution.

52-C
The correct answer is Congress

Congress, the United States' parliament, was established in 1789. The House of
Representatives and the Senate make the two Houses of this bicameral legislature.
There are 435 members of the House of Representatives and 100 senators among the
535 total members of Congress.

53-C
The correct option is the USA.

A legally sanctified document called the constitution contains the fundamental tenets
that guide the government. The United States of America created the first constitution
ratified on June 21, 1788. The text established a standard for republicanism and
codified subsequent constitutions.

54-C
The correct option is Ireland.

The Indian Constitution incorporates several clauses and elements from other
constitutions. The Irish Constitution inspired two Indian laws: the directive principles
and the presidential election. In addition, the Irish Constitution governs the selection of
Rajya Sabha members. Article 55 of the Constitution specifies the process for electing
the president.

55-A
The correct answer is the United Kingdom.

A written constitution is a carefully planned document that outlines all the fundamental
laws, institutions and rules. An 'unwritten' constitution is what the UK is said to have.
The various texts that comprise the laws of the land are not all codified in writing. Many
countries, including Saudi Arabia, the United Kingdom, China, Israel, Canada, and
New Zealand, have unwritten constitutions.

56-A
The correct option is Chloroplast.

A chloroplast is a specialised organelle found in plant and algal cells. It is a


photosynthesis site where light energy is collected and turned into chemical energy

www.careers360.com Back to Index 147


via intricate molecular events. Chloroplasts contain chlorophyll, the green pigment found
in plants that allows them to absorb sunlight and perform photosynthesis.

57-A
The correct option is lysosomes.

The lysosome is the organelle known as the suicidal bag in cells. Lysosomes are
membrane-bound organelles within cells that contain enzymes that degrade cellular
waste items such as worn-out organelles, cellular debris and foreign substances.
They are critical components of cellular maintenance, recycling and defence systems.

58-B
The correct option is To combat infection.

White blood corpuscles, commonly known as white blood cells (WBCs) or


leukocytes, are a kind of blood cell that helps the immune system fight illnesses and
foreign substances. They are engaged in a variety of immunological responses,
including pathogen detection and defence, antibody production and immune
reaction coordination. There are several kinds of white blood cells, each having a
distinct purpose to defend the body from hazardous intruders.

59-D
The correct option is skeletal muscles.

Skeletal muscle tissue, commonly known as striated muscle tissue, is the main
component of skeletal muscle. Connective tissue performs a variety of roles, including
mechanical support, shape maintenance, organ protection, energy storage, and
immune response participation. Bone, cartilage, tendons, ligaments, adipose tissue (fat)
and blood are examples of connective tissues.

60-A
The correct option is proteins.

Viruses are microscopic infectious organisms made up of genetic material (DNA or


RNA) encased in a protein sheath. Proteins are essential components of viruses,
helping to build the capsid (protective coat), attachment spikes, and replication,
assembly, and release within host cells.

www.careers360.com Back to Index 148


MOCK TEST - 5
____________________________________________________________________________

LOGICAL REASONING

Q1. Direction: In the following question, two statements are given, followed by
two conclusions, I and II. You have to consider the two statements to be true,
even if they seem to be at variance from commonly known facts. You have to
decide which of the given conclusions, if any, follow from the given statements.
Statements:
A graduate is a man.
This thief is a graduate.
Conclusions:
I. This thief is a man.
II. Some men are thieves.

A) Only conclusion I follows


B) Only conclusion II follows
C) Neither conclusion I nor conclusion II follows
D) Both conclusion I and conclusion II follow

Q2.Direction: In the following question, two statements are given followed by two
conclusions I and II. You have to consider the two statements to be true even if
they seem to be at variance from commonly known facts. You have to decide
which of the given conclusions, if any, follow from the given statements.
Statements:
1. All men are prisoners.
2. No prisoners are educated.
Conclusions:
I. All prisoners are educated.
II. No men are educated.

A) Only conclusion I follows


B) Only conclusion II follows
C) Neither I nor II follows
D) Both conclusion I and conclusion II follows

www.careers360.com Answer 149


Q3. Assertion:A mirage is an optical illusion that makes distant objects appear
closer and inverted.

Reason:Mirages are caused by the bending of light due to temperature gradients


in the atmosphere.

A) Both the assertion and reason are true, and the reason is a correct explanation of the
assertion.

B) Both the assertion and reason are true, but the reason is not a correct explanation of
the assertion.

C) The assertion is true, but the reason is false.

D) The assertion is false, but the reason is true.

Q4. Assertion: A metal rod expands when heated.

Reason:When heated, the kinetic energy of the metal atoms increases, causing
them to vibrate more and push each other apart.

A) Both the assertion and reason are true, and the reason is a correct explanation of the
assertion.

B) Both the assertion and reason are true, but the reason is not a correct explanation of
the assertion.

C) The assertion is true, but the reason is false.

D) The assertion is false, but the reason is true.

Q5.According to the given statements, Which of the following options is correct.

Statement 1:The Earth orbits the Sun.

Statement 2:The Sun orbits the Earth.

A) Statement 1 is correct, but Statement 2 is incorrect.

B) Statement 2 is correct, but Statement 1 is incorrect.

C) Statement 1 is incorrect.

D) Statement 2 is incorrect.

www.careers360.com Answer 150


Q6.According to the given statements, Which of the following options is correct

Statement 1:Oxygen is essential for combustion to occur.

Statement 2:Carbon dioxide supports combustion.

A) Statement 1 is correct, but Statement 2 is incorrect.

B) Statement 2 is correct, but Statement 1 is incorrect.

C) Statement 1 is incorrect.

D) Statement 2 is incorrect.

Q7. Direction: In the following question, a statement is given, followed by two


arguments, I and II. You have to consider the statement to be true, even if it
seems to be at variance from commonly known facts. You have to decide which
of the given arguments, if any, follows from the given statement.

Statement:
"Should the government impose stricter regulations on social media platforms?"
Arguments:
I. Yes, it can help combat the spread of misinformation and hate speech.
II. No, it infringes on the freedom of expression and innovation.

A) Only Argument I follows.


B) Only Argument II follows.
C) Both Arguments I and II follow.
D) Neither Argument I nor Argument II follows.

Q8. Direction: In the following question, a statement is given, followed by two


arguments, I and II. You have to consider the statement to be true, even if it
seems to be at variance from commonly known facts. You have to decide which
of the given arguments, if any, follows from the given statement.

Statement:
"Should cell phones be allowed in schools?"
Arguments:
I. Yes, they can be used as educational tools and for communication.
II. No, they distract students and may be used for cheating.

www.careers360.com Answer 151


A) Only Argument I follows.
B) Only Argument II follows.
C) Both Arguments I and II follow.
D) Neither Argument I nor Argument II follows.

Q9.Direction: In the following question, select the odd letter cluster from the
given alternatives.

A) CY
B) NJ
C) RN
D) SN

Q10.Direction: In the following question, select the odd one from the given
alternatives.

A) RQP
B) XWV
C) ZYX
D) AZX

Q11.Direction: In a row of people, Manu is 7th from the bottom end of the row.
Shrey is 10 ranks above Manu. If Shrey is 8th from the top, then how many people
are there in this row?

A) 25
B) 26
C) 24
D) 23

Q12.Direction: In a certain code language, PEN is written as QFO. How is CAT


written in that code language?

A) DBU
B) DCU
C) CBU
D) DBV

www.careers360.com Answer 152


LEGAL REASONING

Q13: In a contract dispute, Party A claims that the contract is voidable due to
duress. What element must Party A demonstrate to prove duress?

A. Undue Influence

B. Unilateral Mistake

C. Threat of Physical Harm

D. Economic Pressure

Q14: During a criminal trial, if the prosecution introduces evidence obtained


through a warrantless search, which legal doctrine might be invoked to exclude
the evidence?

A. Good Faith Exception

B. Inevitable Discovery Doctrine

C. Exclusionary Rule

D. Plain View Doctrine

Q 15In a tort case, if the plaintiff's own negligence contributes to their injury, and
the jurisdiction follows a pure comparative negligence system, how does this
impact the plaintiff's recovery?

A. Plaintiff recovers nothing

B. Plaintiff's recovery is reduced by their percentage of fault

C. Plaintiff's recovery is doubled

D. Plaintiff's recovery is increased

www.careers360.com Answer 153


Q16: In a defamation case, if the defendant can prove that the statement was
privileged due to the speaker's legal or moral duty to communicate it, which
defense is being raised?

A. Absolute Privilege

B. Qualified Privilege

C. Fair Comment

D. Consent

Q17: In a criminal trial, if the prosecution introduces the defendant's prior


criminal record as evidence to show a propensity to commit the charged crime,
what legal principle might be violated?

A. Hearsay Rule

B. Best Evidence Rule

C. Character Evidence Rule

D. Authentication Rule

Q18: In an international law context, if a state allows another state to use its
territory for military operations, and this results in harm to a third state, what
principle might be invoked to hold the assisting state responsible?

A. Jus Cogens

B. State Immunity

C. State Responsibility

D. Pacta Sunt Servanda

www.careers360.com Answer 154


Q19During a criminal trial, if a defendant argues that they were intoxicated and
incapable of forming the requisite intent for the crime, what defense is being
raised?

A. Insanity

B. Diminished Capacity

C. Involuntary Intoxication

D. Voluntary Intoxication

Q20-In a product liability case, if a defect in a product makes it unreasonably


dangerous, and the plaintiff used the product in a foreseeable manner, what type
of defect is being alleged?

A. Design Defect

B. Manufacturing Defect

C. Warning Defect

D. Market Share Liability

Q21. Principle: An injury caused by an activity within the statutory authority of an


agency cannot give rise to a tort claim.

Facts: The road between Park Circus and Park Street in Kolkata has developed a
series of potholes and around half a day's work is required to repair it. The
Municipal Corporation of Kolkata blocks off the road and repairs it from 8 am to 6
pm. As a result of the blockage, several people are late for work. Some of them
particularly suffer tremendous loss as they are late for an important meeting.
They bring a suit for nuisance against the MCK. Will they succeed?

A. Yes, MCK's acts amounted to nuisance


B. No, since this is public nuisance
C. No, since MCK is doing its duty
D. No, as MCK is a statutory authority

www.careers360.com Answer 155


Q 22: In a contract dispute, if a party claims that they were induced to enter the
contract based on fraudulent misrepresentations, what remedy might they seek?

A. Specific Performance
B. Rescission
C. Damages
D. Injunction

Q23:In a constitutional law context, if a law limits speech based on its content
and there is no compelling governmental interest, what constitutional scrutiny
does it likely face?

A. Strict Scrutiny
B. Intermediate Scrutiny
C. Rational Basis Review
D. Exacting Scrutiny

Q24: In a property law scenario, if a person openly occupies another's property


without permission and maintains continuous possession for a statutory period,
what legal doctrine might allow them to claim ownership?

A. Eminent Domain
B. License
C. Escheat
D. Adverse Possession

ANALYTICAL REASONING
Q25.A person crosses a 600 m long street in 5 minutes. What is his speed in km
per hour?

A] 5
B] 7
C] 9
D] 7.2

www.careers360.com Answer 156


Q26. An aeroplane covers a certain distance at a speed of 240 kmph in 5 hours to

cover the same distance in 1 hours, it must travel at a speed of:

A] 300 kmph
B] 360 kmph
C] 400 kmph
D] 720 kmph

Q27.To cover a certain distance at a speed of 60 km/h, a train takes 15 hours. If it


covers the same distance in 12 hours, what will its speed be?

A] 9
B] 10
C] 5
D] 12

Q28. A can do a work in 18 days. When he had work for 2 days, B joined him. If
they complete the remaining work in 4 more days . In how many days B alone
finish the whole work?

A] 6 days.
B] 5 days.
C] 10 days.
D] 8 days.

Q29. A pump can fill a tank with water in 2 hours. Because of a leak, it took
22⁄3 hours to fill the tank. The leak can drain all the water of the tank in:

A] 4 hours
B] 6 hours
C] 2 hours
D] 8 hours

www.careers360.com Answer 157


Q30. There are two mixtures of honey and water, the quantity of honey in them
being 25% and 75 % of the mixture. If 2 gallons of the first are mixed with three
gallons of the second, what will be the ratio of honey to water in the new
mixture?

A] 11 : 2
B] 11 : 9
C] 9 : 11
D] 2 : 11

Q31.ABCD is a rhombus, then the measure of ∠ CDB is

A] 30°

B] 60°

C] 40°

D] 50°

Q32. Three cubes each of volume 216 cm3 are joined end to end to form a cuboid.
Then the total surface area of the resulting cuboid is (in cm2)

A] 304

B] 404

C] 504

D] 600

Q33. If the radius of the base of a right circular cylinder is halved, keeping the
height the same, then the ratio of the volume of the cylinder thus obtained to the
volume of original cylinder is

A] 1:2
B] 2:3
C] 1:4
D] 1:5

www.careers360.com Answer 158


Q150. The monthly salaries of two persons are in the ratio of 4:7. If each receives
an increase of Rs.25 in the salary, the ratio is altered to 3: 5. Find their respective
salaries.

A] 120 & 210


B] 80 & 140
C] 180 & 300
D] 200 & 350

Q35.Directions: In the following question, select the related number from the
given alternatives.
36 : 6 :: 441 : ?

A) 11
B) 21
C) 25
D) 32

Q36.Directions: In the following question, which set of letters, when sequentially


placed in the gaps in the given letter series, shall complete it?
KK_LL_K_L_LL

A) LKML
B) LKKK
C) LLKL
D) LKKL

READING COMPREHENSION

Q37.Select the option that can be used as a one-word substitute for the given
group of words.
An outer protective layer of a tree.

A) Bark

B) Shrub

C) Rind

D) Peel

www.careers360.com Answer 159


Q38. Parts of the following sentence have been given as options. Select the
option that contains an error.
My mother is a honest and well-known woman in the society.

A) in the society

B) My mother

C) is a honest

D) and well known woman

Q39.Sentences of a paragraph are given below. While the first and the last
sentences (S1 and S6) are in the correct order the sentences in between are
jumbled up. Arrange the sentences in the correct order to form a meaningful and
coherent paragraph.
S1. A moment comes
A. long suppressed
B. and when the soul of a nation
C. which comes but rarely in history
D. when we step out from the old to the new when an age ends
S6. finds utterance.

A) CBAD

B) CDBA

C) ABCD

D) DCBA

Q40. Identify the sentence that contains no spelling errors.

A) Manisha got overwhelmed by the tremendus offer by a company to work in


collaboration with internet personel.

B) Manisha got overwhelmed by the tremendous offer by a company to work in


collaboration with internet personnel.

C) Manisha got overwellmed by the tremendous offer by a company to work in


collaborasion with internet personnel.

www.careers360.com Answer 160


D) Manisha got overwhelmed by the tremendus offer by a company to work in collaborat

Q41.Select the most appropriate meaning of the given idiom.


In a soup

A) Extremely short of money

B) To exhibit cowardice

C) To get nothing

D) In trouble

Q42. Select the most appropriate ANTONYM of the underlined word.


They praised her for her bravery.

A) Acclaimed

B) Rebuked

C) Defended

D) Assaulted

Q43.Select the option that contains a grammatical error in the underlined portion.
He was always been a grumpy man so I never liked him and avoided his company.

A) He was always been a grumpy man so I never liked him and avoided his company.

B) He was always been a grumpy man so I never liked him and avoided his company

C) He was always been a grumpy man so I never liked him and avoided his company

D) He has always been a grumpy man so I never liked him and avoided his company

Q44.Select the option that can be used as a one-word substitute for the given
group of words.
A person who does not drink alcohol.

A) Vegetarian

B) Celibate

C) Teetotaller

www.careers360.com Answer 161


D) Abstain

Q45.Describe how you will tell your parents that Suraj is loved by Siva in an
active voice.

A) Siva is been loving Suraj.

B) Suraj loved Siva.

C) Siva loves Suraj.

D) Siva were loving Suraj.

Q46. Select the INCORRECTLY spelt word.

A) Believe

B) Audible

C) Throat

D) Pronounciation

Q47. Select the most appropriate ANTONYM of the underlined word in the given
sentence.
I confronted the journalist with the evidence.

A) Resisted

B) Avoided

C) Challenged

D) Encountered

Q48. Select the most appropriate idiom/phrase that can substitute the given
group of words.
The better you know someone the less you like him.

A) Save for a rainy day

B) Spill the beans

C) The elephant in the room

www.careers360.com Answer 162


D) Familiarity breeds contempt

GENERAL KNOWLEDGE

Q49. What is the minimum age to qualify for Lok Sabha Elections?

A) 25 years
B)30 years
C) 21 years
D) 18 years

Q50. Where was Mahatma Gandhi born?

A) Madhya Pradesh
B) Assam
C) Rajastan
D) Gujarat

Q51.Who decides about the size and membership of the Council of Ministers?

A) Chairman of the Rajya Sabha


B) President
C) Prime Minister
D) Lok Sabha

Q52. The crop development programme of the government of India covers which
of the following groups of commercial crops?

A) Cotton, Jute and Sugarcane


B) Tea ,coffee and spices
C) Jute ,Tea and coffee
D) Tea ,cotton and rubber

Q53.Synapse gap is present between which of the following?

A) Two neurons
B) Brain and spinal chord
C) two kidneys
D) None of these

www.careers360.com Answer 163


Q54. What is the full form of DNA?

A) Diribo nuclei acid


B) Di nucleic acid
C) Dual nitrogen acid
D) Deoxyribonucleic acid

Q55. The speaker of the Lok Sabha has to address his/her letter of resignation to

A) Prime Minister of India


B) President of India
C) Deputy speaker of lok sabha
D) Minister of parliamentary affairs

Q56. What is the final product of Glycolysis?

A) ethyl alcohol
B) carbon dioxide
C) pyruvic acid
D) glucose

Q57.The Battle of Haldighati was fought between which two armies?

A) Babur and Lodi Empire


B) Babur and Rana Sanga
C) Akbar and Hemu
D) Akbar and Maharana Pratap

Q58. Banks in India were nationalized for first time in the year:

A) 1950
B) 1960
C) 1969
D) 1979

Q59.Lysosomes are known as suicidal bags because of their ______.

A) protolytic enzymes
B) parastising nucleus
C) hydrolytic enzymes

www.careers360.com Answer 164


D) glucose

Q60. When was the Mumbai Stock exchange set up?

A) 1947
B) 1900
C) 1857
D) 1875

www.careers360.com Answer 165


ANSWER KEY
____________________________________________________________________________

1 2 3 4 5 6 7 8 9 10
D B A A A A C C D D
11 12 13 14 15 16 17 18 19 20
C A C C B B C C D A
21 22 23 24 25 26 27 28 29 30
D B A D D D A A D B
31 32 33 34 35 36 37 38 39 40
B C C D B C A C B B
41 42 43 44 45 46 47 48 49 50
D B B C C B B D A D
51 52 53 54 55 56 57 58 59 60
C A A D C C D C C D

www.careers360.com Solutions 166


SOLUTIONS
____________________________________________________________________________

LOGICAL REASONING
1-D
According to the given statements, the Venn diagram will be –

Now, let's analyse the conclusions –


Conclusion (I): This thief is a man – From the Venn diagram, we can see that the areas
which represents the man completely overlaps the area which represents the thief. So,
any thief will be a man. So, this conclusion follows.
Conclusion (II): Some men are thieves – From the Venn diagram, we can see that some
part of the area which represents the man is outside the area which represents thieves.
This means that some men are thieves and some are not. So, this conclusion follows.

So, according to the given statements, both conclusion I and conclusion II follow.
Hence, the fourth option is correct.

2-B
Based on the given statements, the Venn diagram is as follows –

www.careers360.com Back to Index 167


Let's analyze the conclusions –
Conclusion (I): All prisoners are educated – From the Venn diagram, we can see that
none of the prisoners are educated as there is no overlap. So, this conclusion does not
follow.
Conclusion (II): No men are educated – From the Venn diagram, we can see that none
of the prisoners are educated and the circle of men lies inside the circle of prisoners.
So, from this, it can be inferred that if no prisoners are educated then no men can be
educated. Thus, this conclusion follows.

So, only conclusion II follows. Hence, the second option is correct.

3-A
Mirages are indeed caused by the bending of light due to temperature gradients in the
atmosphere, which can create optical illusions, including making distant objects appear
closer and inverted.

4-A
When a metal is heated, the increased kinetic energy of the metal atoms causes them
to vibrate more vigorously, pushing each other apart and leading to thermal expansion.

5-A
In our solar system, the Earth orbits the Sun, following a heliocentric model.

6-A
Oxygen is essential for combustion, while carbon dioxide does not support combustion;
it can even act as a fire extinguisher because it can displace oxygen.

7-C
Both Argument I and Argument II follow. Argument I emphasizes the benefits of stricter
regulations, such as combating misinformation and hate speech. Argument II points out
the potential negative consequences, including infringement on freedom of expression
and innovation.

www.careers360.com Back to Index 168


8-C
Both Argument I and Argument II follow. Argument I supports the idea of allowing cell
phones for their potential use as educational tools and for communication. Argument II
points out potential negative consequences, including distractions and cheating, which
suggests reasons against allowing them.

9-D
Let's check the options –

First option: CY; C – 4 = Y

Second option: NJ; N – 4 = J

Third option: RN; R – 4 = N

Fourth option: SN; S – 5 = N (The fourth option is different from the other three.)

Hence, the fourth option is correct.

10-D
Let's check the options –

First option: RQP; RQP is in the reverse alphabetical order.

Second option: XWV; XWV is in the reverse alphabetical order.

Third option: ZYX; ZYX is in the reverse alphabetical order.

Fourth option: AZX; AZX is not in the reverse alphabetical order.

Hence, the fourth option is correct.

11-C
Manu is 7th from the bottom end of the row. Shrey is 10 ranks above Manu and is 8th
from the top.

www.careers360.com Back to Index 169


The rank of Shrey from the top = 8
The rank of Shrey from the bottom = 10 + 7 = 17 ( Since Shrey is 10 ranks above Manu)
Total number of people in a row = Rank from top + Rank from bottom – 1
Total number of people in a row = 8 + 17 – 1 = 24

Hence, the third option is correct.

12-A
Given:

PEN is written as QFO.

Like,

P + 1 = Q; E + 1 = F; N + 1 = O

The word formed is QFO.

Similarly, for the word CAT

C + 1 = D; A + 1 = B; T + 1 = U

The word formed is DBU in that code language.

Hence, the first option is correct.

www.careers360.com Back to Index 170


LEGAL REASONING

13 -C:
Duress involves the use of force or threat of physical harm to compel someone to enter
into a contract against their will. It goes beyond economic pressure or persuasive
tactics.

14-C
The Exclusionary Rule mandates that evidence obtained through illegal or
unconstitutional means, such as a warrantless search, is generally inadmissible in court.

15-B:
In a pure comparative negligence system, the plaintiff's recovery is reduced by the
percentage of their own fault. Even if the plaintiff is partially at fault, they can still
recover damages.

16-B
Qualified privilege protects certain communications made in good faith and with a
legitimate interest, such as statements made in legal proceedings or by employers to
employees.

17-C
Character evidence generally cannot be used to show a defendant's propensity to
commit a crime. Exceptions exist, but introducing a prior criminal record solely for
propensity is often prohibited.

18-C
The principle of State Responsibility holds states accountable for wrongful acts that
breach their international obligations. Allowing another state to use territory may trigger
responsibility if harm results.

19-D
Voluntary intoxication is a defense in which the defendant claims that their state of
intoxication at the time of the offense prevented them from forming the necessary intent.

www.careers360.com Back to Index 171


20-A
Design defect occurs when a product's design makes it unreasonably dangerous. If the
danger is foreseeable, and the product is used as intended, it may give rise to a design
defect claim.

21-D
No, as MCK is a statutory authority, An injury caused by an activity within the
statutory authority of an agency cannot give rise to a tort claim. As it is a general
defence. Hence option d is correct.

22-B
Rescission is a remedy that allows a party to cancel the contract and return to the status
quo before the agreement, typically sought when there are fraudulent
misrepresentations.

23-A
Content-based restrictions on speech are subject to strict scrutiny, requiring the
government to show a compelling interest and demonstrate that the restriction is
narrowly tailored to achieve that interest.

24-D
Adverse possession allows an individual to acquire legal ownership of land by openly
occupying it without the owner's permission for a specified period, fulfilling certain
requirements.

ANALYTICAL REASONING
25-D
Speed =2 m/sec.

Converting m/sec to km/hr= = 7.2 km/hr.

26-D
Distance = (240 x 5) = 1200 km.
Speed = 1200/(5/3) km/hr.

www.careers360.com Back to Index 172


27-A

28-A
work done by A in 1 day = 1/ 18
Number of days A work = 2+4= 6 therefore, total work done by A =6x1/18 = 1/3
The remaining 2/3 work is done by B in 4 days and hence complete work done
by B will be
4x(3/2) = 6 days.

29-D
Let the leak can drain all the water of the tank in y hours
Part of the tank filled by the pipe in 1 hr = 1/2
Part of the tank emptied by the leak in 1 hr = 1/y

www.careers360.com Back to Index 173


1/2−1/y=3/8⇒1/y=1/2−3/8=1/8⇒y=8
i.e., the leak can drain all the water of the tank in 8 hours

30-B
H: W H: W
1: 3 3: 1

2 gallon 3 gallon
1/2, 3/2 9/4, 3/4
Ratio = ½ + 9/4: 3/2 + ¾
11/4: 9/4
11:9

31-B
We have ∠ A = ∠ C = 60° (opposite angle of parallelogram are equal)

Let, ∠ CDB = x

In triangle CDB we have

CD = BC (side of rhombus are equal)

So, ∠ CDB = ∠ DBC = x

therefore, ∠ CDB + ∠ DBC + 60° = 180°

2x = 120°

x = 60°

∠ CDB = 60°

32-C
Given, the volume of cube = 216 cubic cm

let the side of the cube be 'a' cm

Volume of cube = a3 = 216

a = 6 cm

Length of resulting cuboid, l = 18 cm.

www.careers360.com Back to Index 174


Breadth of resulting cuboid, b = 6 cm.

Height of resulting cuboid, h = 6 cm.

therefore, the total surface area of the cuboid

33-C

34-D
Let salaries be 4x & 7x
So, (4x + 25)/(7x + 25) = 3/5
= > x = 50
4*50 : 7*50
200 :350
4:7

35-B
Given:
36 : 6 :: 441 : ?
√36 = 6 (6 is the square root of 36.)
Similarly, √441 = 21 (21 is the square root of 441.)

Hence, the second option is correct.

www.careers360.com Back to Index 175


36-C
Given:
KK_LL_K_L_LL

Divide the series into letter clusters – KK _LL_ / K_ L_LL


Let's check each option –
First option: LKML; KKLLLK / KMLLLL (No pattern)
Second option: LKKK; KKLLLK / KKLKLL (No pattern)
Third option: LLKL; KKLLLL / KKLLLL (KKLLLL is repeated)
Fourth option: LKKL; KKLLLK / KKLLLL (No pattern)

So, the series becomes→KKLLLLKKLLLL. Hence, the third option is correct.

READING COMPREHENSION

37-A

Shrub: A small woody plant typically with multiple stems and a shorter height than a
tree.

Rind: The tough outer layer or bark of a fruit vegetable or tree.

Peel: The outer skin or covering of a fruit or vegetable often removed before eating or
cooking.

Bark refers to the tough protective outer sheath of the trunk branches and twigs of a
tree or woody shrub. Thus the outer protective layer of a tree is called bark.

38-C
The error lies in the third option. The error in this option is the incorrect use of the
indefinite article "a" before "honest." The correct form should be "an honest" because
"honest" begins with a vowel sound.

So the correct sentence should be: "My mother is an honest and well-known woman in
the society."

39-B
The correct order of the sentences to form a meaningful and coherent paragraph is:
CDBA
C. which comes but rarely in history

D. when we step out from the old to the new when an age ends

www.careers360.com Back to Index 176


B. and when the soul of a nation

A. long suppressed

The paragraph is describing a significant moment in the history of a nation when a


long-suppressed soul or spirit finds expression as an old age or era comes to an end
giving way to the new. This moment is portrayed as a rare occurrence in history
signifying a significant transformation or change in the nation's identity or direction.

40-B
The sentence that contains no spelling errors is: "Manisha got overwhelmed by the
tremendous offer by a company to work in collaboration with internet personnel."

● "Tremendous" is the correct spelling of the word meaning "very large or


great."

● "Personnel" is the correct spelling for the word referring to the staff or
employees of a company or organization.

● "Collaboration" is the correct spelling for the act of working together or


cooperating with others.

The other options contain various spelling errors.

41-D

The idiom "In a soup" is used to describe a situation where someone is facing trouble
or difficulty not specifically a financial issue. "In trouble" conveys the idea that
someone is facing difficulties challenges or problems.

42-B
"Praised" means to express approval admiration or compliments for someone's actions
qualities or achievements. "Rebuked" means to criticise or reprimand someone for
their actions or behaviour.

● Acclaimed: It means to be praised or celebrated.

● Defended: It means to protect or support.

www.careers360.com Back to Index 177


● Assaulted: It means to attack or physically harm.

43-B
The error in this sentence is the use of "was always been" in the past tense. The
correct form should be "He has always been a grumpy man." This is a present perfect
tense which is used to describe an action or situation that started in the past and
continues into the present.

The other options have different errors such as missing commas but they don't have the
same grammatical issue as the one identified in the selected sentence.

So the correct sentence should be: "He has always been a grumpy man so I never
liked him and avoided his company."

44-C
"Teetotaller" is a one-word substitute that specifically describes a person who does
not drink alcohol making it the most appropriate choice.

● Vegetarian: A person who does not consume meat or animal products.


● Celibate: A person who abstains from sexual activity typically for religious or
personal reasons.
● Abstain: It is a verb that means to refrain from doing something.

45-C

The original passive sentence is in the present simple tense. In the active voice the
tense remains the same The verb "loves" is used to maintain the present simple tense
and convey the ongoing action of loving. In active voice the subject of the passive
sentence becomes the object of the sentence. The structure of the active voice
sentence follows: "Subject (S) + Verb (V) + Object (O)."

46-D
The correct spelling is "pronunciation." It refers to the way in which a word is spoken
with emphasis on the correct sounds and articulation. The incorrect spelling of "ou"
instead of "u" is a common mistake.

The meaning of other options are as follows:

www.careers360.com Back to Index 178


● Believe: It means to accept something as true or to have faith or confidence in
something or someone.

● Audible: It means capable of being heard or loud enough to be heard.

● Throat: It refers to the passage in the neck that leads to the stomach and the
lungs used for swallowing and breathing.

47-B
This is the correct antonym for "confronted." "Avoided" means to stay away from or
prevent contact or interaction with something while "confronted" means to face or deal
with it directly.

● Resisted: It implies actively opposing something.

● Challenged: It is related to questioning or testing something.

● Encountered: It means to come across or meet something.

48-D
The idiom "Familiarity breeds contempt" means that as we become more familiar
with someone or something we may notice their flaws or shortcomings decreasing our
liking or respect for them. This idiom accurately captures the idea presented in the given
group of words making it the most suitable choice.

The meanings of the other options are as follows:

● "Save for a rainy day" means to save some of your money in case any problem
arises in the future.
● "Spill the beans" means to tell people secret information.
● "Elephant in the room" is an obvious truth that is deliberately ignored by
everyone in a situation.

The other options are not suitable choices that can substitute the given group of words.

www.careers360.com Back to Index 179


GENERAL KNOWLEDGE

49-A
Article 84(b) of the Constitution of India provides that the minimum qualifications for
becoming a candidate for the Lok Sabha election shall be 25 years, and he or she
should be a citizen of India. The minimum age eligibility for the Lower House is 25
years. To qualify for the Rajya Sabha, a candidate needs to be 30 years of age or older.
Provision exists for a candidate to the Legislative Assembly under Article 173(b) of the
Constitution.

50-D
On October 2, 1869, Mohandas Karamchand Gandhi was born in Porbandar, Gujarat, in
western India. The occasion is now known as Gandhi Jayanti. His mother's name was
Putlibai, and his father's name was Karamchand Gandhi. A lawyer, politician, social
activist, and author, Mohandas Karamchand Gandhi led the nationalist movement that
opposed British rule in India.

51-C
The size of the Central Council of Ministers will be decided by the Prime Minister based
on the demands of the time and the circumstances. On the advice of the Prime Minister,
the President of India decides the tenure and composition of the Central Council of
Ministers under Article 75(1). The president has discretion over who serves on the
council.

52-A
Under the National Food Security Mission (NFSM-CC), the Government of India
approved the Crop Development Program on Cotton, Jute, and Sugarcane to boost
production and productivity. Through state governments, the Indian government is
putting various crop development programs into action. As part of these programs,
states and UTs offer financial assistance to farmers so they can use cutting-edge
production methods and essential inputs like seeds and micronutrients to increase crop
production and productivity.

53-A
The term "synaptic gap" or "synaptic cleft" refers to the small space or gap that exists
between one neuron's synaptic terminal and the dendrites or cell body of another
neuron. It is a necessary component of the synapse, which is the junction where two
neurons connect with one another.

www.careers360.com Back to Index 180


54-D
DNA, or deoxyribonucleic acid, is a complex molecule that holds the genetic information
required for all known living species to grow, develop, function, and reproduce. It is
made up of a distinct sequence of nucleotide bases (adenine, thymine, cytosine, and
guanine) that create a double helix. James Watson and Francis Crick discovered the
structure of DNA in 1953. The sugar molecule in DNA is deoxyribose, a five-carbon
sugar with one oxygen atom missing when compared to ribose, the sugar present in
RNA (ribonucleic acid).

55-C
Article 93 to Article 96 of the Indian constitution deals with the powers and functions of
speaker and deputy speaker of Lok Sabha. As per the convention the speaker of Lok
Sabha is from the ruling party and the deputy speaker is from opposition. The speaker
of Lok Sabha is responsible for orderly functioning of the lower house, conducting
regular proceeding and maining law and order in the house. As per the constitution the
speaker of Lok Sabha can resign by addressing his resignation to the deputy speaker of
Lok Sabha.

56-C
Glycolysis is a metabolic process that occurs in the cytoplasm of living cells. It is the
process of converting glucose into pyruvate (pyruvic acid). It is the biological process
that breaks down glucose to release energy. Glucose is converted to pyruvate, and
energy in the form of ATP is released. The process happens in both aerobic and
anaerobic organisms and occurs without the participation of extra oxygen.

In aerobic glycolysis, the end product is pyruvate (pyruvic acid), while in anaerobic
glycolysis, the end product is lactate (lactic acid). The end product of glycosis is two
molecules of pyruvic acid, along with two molecules of ATP and NADH.

57-D
The Rajput monarch Maharana Pratap of Mewar and the Mughal emperor Akbar
engaged in combat in Haldighati. On June 18, 1576, a battle took place in the Haldighati
Pass, which is now in the state of Rajasthan. The bravery and resistance shown by
Maharana Pratap and his Rajput soldiers against the strong Mughal armies made this
battle one of the most well-known and pivotal clashes in Indian history.

58-C
Banks were nationalised with the aim of expansion of banking services to the unserved
and underserved areas, and to bring financial inclusion and consolidation in India. July
19, 1969 was the date when Indira Gandhi, then Prime Minister of India, declared the
nationalization of 14 banks whose deposits exceeded Rs 50 crore. Banks guarantee the

www.careers360.com Back to Index 181


security of our money and in return provide interest. They lend money to people,
institutions, and others to establish businesses and for various commercial and personal
transactions.

59-C
Lysosomes contain a range of hydrolytic enzymes, which are enzymes that break down
molecules via hydrolysis, a chemical reaction in which water is added. These enzymes
are responsible for degrading diverse biological components such as proteins, lipids,
carbohydrates, and nucleic acids. The breakdown of these compounds within
lysosomes aids in the recycling of cellular components, the elimination of waste
products, and the maintenance of cellular homeostasis. Proteases, lipases,
glycosidases, nucleases, phosphatases, and sulfatases are some of the enzymes found
in lysosomes.

60-D
On July 9, 1875, the Bombay Stock Exchange (BSE), also known as the Mumbai Stock
Exchange, was founded. It is one of the oldest stock exchanges in Asia and is important
to the Indian financial market. The BSE is a trading venue for a variety of financial
assets, including stocks, bonds, derivatives, and other securities. It is based in Mumbai,
Maharashtra, India.

www.careers360.com Back to Index 182


MOCK TEST - 6
____________________________________________________________________________

LOGICAL REASONING
Q1.Direction: In the following question below, some statements are followed by
some conclusions. Taking the given statements to be true, even if they seem to
be at variance from commonly known facts, read all the conclusions and then
decide which of the given conclusions logically follows from the given
statements.

Statements:

Some bags are hot.

All hot things are cakes.

Conclusions:

I. All cakes are bags.

II. All bags are cakes.

A) Conclusion I follows
B) Conclusion II follows
C) Neither I nor II follows
D) Both I and II follows

Q2. Direction: In the following question, two statements are given followed by two
conclusions I and II. You have to consider the two statements to be true even if
they seem to be at variance from commonly known facts. You have to decide
which of the given conclusions, if any, follow from the given statements.

Statements:

I. Sohan is a good sportsman.

II. Sportsmen are healthy.

Conclusions:

I. All healthy people are sportsmen.

II. Sohan is healthy.

www.careers360.com Answer 183


A) Conclusion I follows
B) Conclusion II follows
C) Neither conclusion I nor conclusion II follows
D) Both conclusion I and conclusion II follows

Q3. Assertion: The human heart has four chambers.

Reason: The heart's four chambers include two atria and two ventricles, each
with specific functions.

A) Both the assertion and reason are true, and the reason is a correct explanation of the
assertion.

B) Both the assertion and reason are true, but the reason is not a correct explanation of
the assertion.

C) The assertion is true, but the reason is false.

D) The assertion is false, but the reason is true.

Q4.Assertion: The process of osmosis involves the movement of water molecules


from a region of higher solute concentration to a region of lower solute
concentration.

Reason: Osmosis is a passive transport process driven by a concentration


gradient.

A) Both the assertion and reason are true, and the reason is a correct explanation of the
assertion.

B) Both the assertion and reason are true, but the reason is not a correct explanation of
the assertion.

C) The assertion is true, but the reason is false.

D) The assertion is false, but the reason is true.

Q5..According to the given statements, Which of the following options is correct.

Statement 1: The capital of Japan is Tokyo.

Statement 2: The capital of South Korea is Seoul.

A) Statement 1 is correct, but Statement 2 is incorrect.

www.careers360.com Answer 184


B) Statement 2 is correct, but Statement 1 is incorrect.

C) Statement 1 is incorrect.

D) Statement 2 is incorrect.

Q6.According to the given statements, Which of the following options is correct

Statement 1:Sodium is a non-metal.

Statement 2:Chlorine is a metal.

A) Statement 1 is correct, but Statement 2 is incorrect.

B) Statement 2 is correct, but Statement 1 is incorrect.

C) Statement 1 is incorrect.

D) Statement 2 is incorrect.

Q7. Direction: In the following question, a statement is given, followed by two


arguments, I and II. You have to consider the statement to be true, even if it
seems to be at variance from commonly known facts. You have to decide which
of the given arguments, if any, follows from the given statement.

Statement:
"Should animal testing be banned for cosmetics?"
Arguments:
I. Yes, it is inhumane and alternative testing methods are available.
II. No, it ensures product safety and compliance with regulations.

A) Only Argument I follows.


B) Only Argument II follows.
C) Both Arguments I and II follow.
D) Neither Argument I nor Argument II follows.

Q8. Direction: In the following question, a statement is given, followed by two


arguments, I and II. You have to consider the statement to be true, even if it
seems to be at variance from commonly known facts. You have to decide which
of the given arguments, if any, follows from the given statement.

Statement:

www.careers360.com Answer 185


"Should the legal drinking age be lowered to 18?"

Arguments:

I. Yes, it aligns with the age of adulthood and reduces the appeal of underage
drinking.

II. No, it can lead to increased alcohol-related problems among younger


individuals.

A) Only Argument I follows.

B) Only Argument II follows.

C) Both Arguments I and II follow.

D) Neither Argument I nor Argument II follows.

Q9. Direction: In the following question, select the odd one from the given
alternatives.

A) RTV
B) HJL
C) KMP
D) BDF

Q10. Direction: In the following question, select the odd word/ letter/ number from
the given alternatives.

A) ACDF
B) TUOP
C) HIVW
D) FGKL

Q11. Direction: A boy and a girl are playing in a park. The only daughter of the
maternal grandfather of the girl is the sister of the boy's father. How is the boy
related to the girl?

A) Father
B) Grandfather
C) Son
D) Cousin

www.careers360.com Answer 186


Q12. Direction: Rakesh is six ranks ahead of Tanvir in a class of 42 students. If
Tanvir's rank is eighteenth from the last, then what is Rakesh's rank from the
start?

A) 14th
B) 15th
C) 19th
D) 17th

LEGAL REASONING
Q13: In a property dispute, two neighbors claim ownership of a strip of land
between their properties. Neighbor A has been maintaining and using the land
openly for over 20 years, while Neighbor B only recently discovered the dispute.
Which legal doctrine might apply, and who is likely to have a stronger claim?

A. Doctrine of Eminent Domain


B. Doctrine of Adverse Possession
C. Doctrine of Escheat
D. Doctrine of Necessity

Q14:A family inherits a historic mansion, but due to financial difficulties, they
decide to convert it into a bed-and-breakfast without checking local zoning laws.
Later, they face legal issues because the zoning does not permit commercial use.
What legal concept is relevant, and how might it affect the family?

A. Doctrine of Fixture
B. Doctrine of Eminent Domain
C. Doctrine of Variance
D. Doctrine of Zoning Nonconformity

Q15:A landowner grants an easement to a utility company for the installation and
maintenance of power lines. Subsequently, the landowner decides to build a
fence across the easement, obstructing access for the utility company. What legal
principle might the utility company invoke to challenge the obstruction?

A. Doctrine of Implied Easement


B. Doctrine of License
C. Doctrine of Merger
D. Doctrine of Termination

www.careers360.com Answer 187


Q16.What is the purpose of the preamble in a constitution?

A) Provide a summary of legal cases


B) State the goals and objectives of the constitution
C) Outline the procedural rules for courts
D) Define the hierarchy of laws

Q17.Which branch of law deals with disputes between individuals or


organizations?

A) Criminal Law
B) Civil Law
C) Constitutional Law
D) Administrative Law

Q18. What does the term "habeas corpus" refer to?

A) Right to bear arms


B) Protection against double jeopardy
C) Right to a fair trial
D) Protection against unlawful detention

Q19.In a criminal trial, what does the prosecution bear the burden of proving?

A) Defendant's innocence
B) Defendant's guilt
C) Defendant's mental state
D) Defendant's alibi

Q20.What is the primary function of an appellate court?

A) Conduct trials
B) Review decisions from lower courts
C) Set legal precedents
D) Enforce laws

www.careers360.com Answer 188


Q21.Which amendment protects against unreasonable searches and seizures?

A) First Amendment
B) Second Amendment
C) Fourth Amendment
D) Fifth Amendment

Q22.What is the doctrine of precedent in legal terminology?

A) Legal immunity
B) Binding judicial decisions
C) Parliamentary privilege
D) Executive privilege

Q23.What is the age of majority in most legal systems?

A) 16
B) 18
C) 21
D) 25

Q24.What is the purpose of a confidentiality agreement in legal contracts?

A) Disclose sensitive information


B) Protect against defamation
C) Restrict competition
D) Keep information confidential

ANALYTICAL REASONING
Q25. If a person walks at 14 km/hr instead of 10 km/hr, he would have walked 20
km more. The actual distance travelled by him is:

A] 50 km
B] 56 km
C] 70 km

www.careers360.com Answer 189


D] 80 km

Q26. A train can travel 50 \% faster than a car. Both start from point A at the same
time and reach point B away from A at the same time. On the way, however, the
train lost about 12.5 minutes while stopping at the stations. The speed of the car
is:

A] 100 kmph

B] 120kmph

C] 210kmph

D] 200kmph

Q27. 2.A boat goes a certain distance at 30 km/hr and comes back the same
distance at 60 km/hr. What is the average speed (in km/hr) for the total journey?

A] 45
B] 40
C] 35
D] 55

Q28. Ram completes 60% of a task in 15 days and then takes the help of Rahim
and Rachel. Rahim is 50% as efficient as Ram is and Rachel is 50% as efficient as
Rahim is. In how many more days will they complete the work?

A] 40/7
B] 20/7
C] 30/7
D] 10/7

Q29. Three taps A, B and C can fill a tank in 12, 15 and 20 hours respectively. If A
is open all the time and B and C are open for one hour each alternately, the tank
will be full in:

A] 7 hour

www.careers360.com Answer 190


B] 5 hour
C] 3 hour
D] 4 hour

Q30. Two solutions of 90% and 97% purity are mixed resulting in 21 litres of
mixture of 94 % purity. How much is the quantity of the first solution in the
resulting mixture?

A] 15 litres
B] 12 litres
C] 9 litres
D] 6 litres

Q31. Length of one side of a parallelogram is 10 cm, then the length of its
adjacent side, when perimeter of parallelogram given as 50 cm is

A] 15cm

B] 12cm

C] 10cm

D] 16cm

Q32.A medicine capsule is in the shape of a cylinder of diameter 1 cm with two


hemispheres stuck to each of its ends. The length of the entire capsule is 2 cm.
The capacity (in cm3) of the capsule is

A] 0.31
B] 1.31
C] 2.31
D] 4.05

Q33. If the height of a bucket in the shape of frustum of a cone is 16 cm and the
diameters of its two circular ends are 40 cm and 16 cm then its slant height is

A] 20
B] 15
C] 25
D] 30

www.careers360.com Answer 191


Q34. The speed of three cars is in the ratio of 2 : 3 : 4. What is the ratio among the
times taken by these cars to travel the same distance?

A] 6:4:3
B] 6:5:4
C] 1:2:3
D] 3:4:6

Q35.Directions: In the given figure, how many papers are also hard?

A) 14
B) 23
C) 37
D) 32

Q36.Directions: The weights of the 4 boxes are 90, 40, 80, and 50 kilograms.
Which of the following cannot be the total weight, in kilograms, of any
combination of these boxes, and in a combination a box can be used only once?

A) 200
B) 260
C) 180
D) 170

READING COMPREHENSION

Q37. Directions: In the following question, a sentence is given with a blank that is
to be filled in with an appropriate word. Four alternatives are suggested; choose
the correct alternative out of them as your answer.
The bank decided to _____ them the money as they were reliable clients.

www.careers360.com Answer 192


A) let

B) lend

C) borrow

D) lend

Q38. Directions: In the given question, a sentence is given with a blank to be filled
in with an appropriate word. Four alternatives are suggested for each question.
Choose the correct alternative out of the four alternatives.

The flood damaged the books so much that it was impossible to __________them.

A) retrieve

B) retrace

C) retract

D) retrieve

Q39.Directions: In the given question, a sentence is given with a blank to be filled


in with an appropriate word. Four alternatives are suggested for each question.
Choose the correct alternative out of the four alternatives.

She was aware of what was going ______her father's mind.

A) through

B) on

C) in

D) through

Q40. Directions: In the given question, a sentence is given with a blank to be filled
in with an appropriate word. Four alternatives are suggested for each question.
Choose the correct alternative out of the four alternatives.

Most parents_________love and affection on their children.

A) pour

B) pore

www.careers360.com Answer 193


C) poor

D) pour

Q41.Directions: In the given question, a sentence is given with a blank to be filled


in with an appropriate word. Four alternatives are suggested for each question.
Choose the correct alternative out of the four alternatives.

Then railway trains_______ the most popular and the cheapest means of
transport.

A) become

B) became

C) becomes

D) became

Q42. Directions: In the given question, a sentence is given with a blank to be filled
in with an appropriate word. Four alternatives are suggested for each question.
Choose the correct alternative out of the four alternatives.

If left unattended, even a small cut can turn into a _______.

A) soar

B) sour

C) sore

D) sore

Q43.Directions: In the given question, a sentence is given with a blank to be filled


in with an appropriate word. Four alternatives are suggested for each question.
Choose the correct alternative out of the four alternatives.

My neighbour is very _______for he believes that nothing good will happen to


him.

A) reasonable

B) forward-looking

C) pessimistic

www.careers360.com Answer 194


D) pessimistic

Q44.Directions: In the following question, a sentence is given with a blank that is


to be filled in with an appropriate word. Four alternatives are suggested; choose
the correct alternative out of them as your answer.
Do not ____ your own personal views upon the reader.

A) expose

B) interpose

C) express

D) impose

Q45. Directions: In the following question, a sentence is given with a blank that is
to be filled in with an appropriate word. Four alternatives are suggested; choose
the correct alternative out of them as your answer.
She ___ him angrily because he made the floor dirty with his muddy shoes.

A) scold

B) scolds

C) scolded

D) scolded

Q46.Directions: In the given question, a sentence is given with a blank to be filled


in with an appropriate word. Four alternatives are suggested for each question.
Choose the correct alternative out of the four alternatives.

She dwells too much _______her past.

A) from

B) on

C) in

D) on

Q47.Select the option that expresses the given sentence in passive voice.

We compelled the teacher to finish the class sooner.

www.careers360.com Answer 195


A) The teacher was compelled to finish the class sooner.

B) The teacher finished the class compelled by us.

C) Finishing the class sooner was what the teacher was compelled to do.

D) The teacher finished the class sooner because she was compelled.

Q48.Sentences of a paragraph are given below in jumbled order. Arrange the


sentences in the correct order to form a meaningful and coherent paragraph.
A. It is not a walker's paradise. So walking is not a fashionable pastime here.
B.London is known for its hectic face, frenzied movements, high decibels and
dust kicked by the speeding vehicles.
C. On the contrary, life in the countryside is laid back and easy-paced. Unless it is
raining, people set out for a walk.
D. For these reasons, Max never went out for a walk, nor did anyone ask him to
accompany.

A) ADCB

B) BADC

C) DCAB

D) CADB

GENERAL KNOWLEDGE

Q48. ’Public health, sanitation conservancy and solid waste management' is


inserted into the Schedule of the Constitution by which of the following
Constitutional Amendments?

A) 42nd Constitutional Amendment.


B) 44th Constitutional Amendment
C) 73rd Constitutional Amendment
D) 74th Constitutional Amendment

Q49. According to Article 74 of the Constitution, which of the following


statements relating to the advice tendered by Ministers to the President would be
correct?

A) It shall not be inquired into in any court.

www.careers360.com Answer 196


B) It can be inquired into in the Supreme Court.
C) It can be inquired into in all the courts.
D) It cannot be inquired into in the High Courts.

Q50. The 'Lake Tahoe' recently seen in the news is located in?

A) United States
B) Japan
C) India
D) Brazil

Q51. Which of the following is a feature of a tropical monsoon climate?

A) High average temperatures year-round


B) Consistent rainfall throughout the year
C) Dry summers and wet winters
D) Extreme temperature variations between day and night

Q52.The Investor Education and Protection Fund Authority (IEPFA) was


established by which of the following?

A) RBI
B) SEBI
C) NITI
D) None

Q53. Who was the first Viceroy of India?

A) Lord Canning

B) Lord Curzon

C) Lord Wavell

D) Lord Mountbatten

Q54.Who was given the title of "The Ambassador of Hindu-Muslim Unity" for
being the architect and mastermind of the historic Lucknow Pact?

A) Sir Syed Ahmad Khan

www.careers360.com Answer 197


B) Fazl-ul-Haq

C) Syed Mohammad Sharfuddin Quadri

D) Muhammad Ali Jinnah

Q55. The 'Narmada Water Dispute tribunal' was constituted to resolve the water
sharing between

A) Gujarat and Rajasthan

B) Gujarat and Maharashtra

C) Gujarat, Maharashtra, Rajasthan and Madhya Pradesh

D) Gujarat, Daman and Diu

Q56. Who gave the slogan "Back to the Vedas (Vedo ki aur lauto)"?

A) Guru Nanak Dev Ji

B) Dayananda Saraswati

C) Swami Vivekananda

D) Raja Ram Mohan Roy

Q57. The Jammu and Kashmir state legislative Assembly has a tenure of

A) six years

B) five years

C) four years

D) seven years

Q58. Fish liver oil is rich in

A) Vitamin A

B) Vitamin C

C) Vitamin D

D) Vitamin E

www.careers360.com Answer 198


Q59. Which one of the following is not a true fish?

A) Shark

B) Starfish

C) Eel

D) Seahorse

Q60. Who appoints the members of the State Public Service Commission (PSC)?

A) The Chief Minister

B) The Governor

C) The Chief Justice

D) The Vice President

www.careers360.com Answer 199


ANSWER KEY
____________________________________________________________________________

1 2 3 4 5 6 7 8 9 10
C B A A B C C C C A
11 12 13 14 15 16 17 18 19 20
D C B D A B B D B B
21 22 23 24 25 26 27 28 29 30
C B B D A B B A A C
31 32 33 34 35 36 37 38 39 40
A B A A B A B A A A
41 42 43 44 45 46 47 48 49 50
B C C D C B A B D A
51 52 53 54 55 56 57 58 59 60
C D A C C B B A B B

www.careers360.com Solutions 200


SOLUTIONS
____________________________________________________________________________________

LOGICAL REASONING
1-C
Based on the given statements, the Venn diagram is as follows –

Let's analyze the conclusions –


Conclusion (I): All cakes are bags – From the Venn diagram, the circle of bags does not
completely overlap the circle of cakes. This means that some cakes are bags. So, this
conclusion does not follow.
Conclusion (II): All bags are cakes – From the Venn diagram, the circle of cakes does
not completely overlap the circle of bags. This means that some bags are not cakes.
So, this conclusion does not follow.

So, neither I nor II follows. Hence, the third option is correct.

2-B
Given:
Statements:
I. Sohan is a good sportsman.
II. Sportsmen are healthy.
The Venn diagram is–

www.careers360.com Back to Index 201


Let's analyze the conclusions–
Conclusion (I): All healthy people are sportsmen.
From the Venn diagram, some healthy people are not sportsmen as they lie outside the
circle that represents sportsmen. So, this conclusion does not follow.
Conclusion (II): Sohan is healthy.
From the Venn diagram, the circle that represents sohan lies completely inside the circle
that represents healthy. So, this conclusion follows.

Thus, only conclusion II follows. Hence, the second option is correct.

3-A
The human heart has four chambers: two atria, which receive blood, and two ventricles,
which pump blood to different parts of the body. Each chamber serves a specific
function in the circulation of blood.

4-A
Osmosis is a passive process where water molecules move from areas of lower solute
concentration to areas of higher solute concentration, driven by the concentration
gradient.

5-B
Tokyo is indeed the capital of Japan, but Seoul is the capital of South Korea.

6-C
Sodium is a metal, and chlorine is a non-metal.

7-C
Both Argument I and Argument II follow. Argument I supports the idea of banning animal

www.careers360.com Back to Index 202


testing due to ethical concerns and the availability of alternative methods. Argument II
points out a potential benefit of animal testing, which is ensuring product safety and
compliance with regulations.

8-C
Both Argument I and Argument II follow. Argument I supports the idea of lowering the
drinking age, citing alignment with the age of adulthood and a potential reduction in the
appeal of underage drinking. Argument II raises concerns about potential negative
consequences, such as increased alcohol-related problems among younger individuals.

9-C

Let's check the options –

First option: RTV; R + 2 = T; T + 2 = V

Second option: HJL; H + 2 = J; J + 2 = L

Third option: KMP; K + 2 = M; M + 3 = P

Fourth option: BDF; B + 2 = D; D + 2 = F

The difference between the letters of the third option is different from the other options.
So, the third option is different from the others.

Hence, the third option is correct.

10-A
Let's check the options —

First option: ACDF; A + 1 = B, D + 1 = E

www.careers360.com Back to Index 203


Second option: TUOP; T + 1 = U, O + 1 = P

Third option: HIVW; H + 1 = I, V + 1 = W

Fourth option: FGKL; F + 1 = G, K + 1 = L

All the options are the same except the first option.

Hence, the first option is correct.

11-D
As per the given instructions, the family tree will be drawn as shown below –

So, from the above family tree, the boy is the cousin of that girl. Hence, the fourth option
is correct.

12-C
Given:
Tanvir is 6 positions behind Rakesh in a class of 42 students. Tanvir is ranked 18th from
the bottom.Rakesh's position from last is (18 + 6) = 24.
The rank of Rakesh from the start is 42 – 24 + 1 = 19.

Hence, the third option is correct.

www.careers360.com Back to Index 204


LEGAL REASONING

13- B
The Doctrine of Adverse Possession allows an individual to gain ownership of land by
openly occupying and using it for a specified period. In this case, Neighbor A, who has
been maintaining the land openly for over 20 years, is likely to have a stronger claim
based on adverse possession.

14-D
The Doctrine of Zoning Nonconformity allows existing structures to continue operating
even if they violate current zoning laws. The family's use of the mansion as a
bed-and-breakfast may be protected under this doctrine, but they may face restrictions
on expansion or modifications.

15-A
The Doctrine of Implied Easement arises when an easement is reasonably necessary
for the continued use of the dominant estate. If the landowner obstructs access to the
utility company's power lines, they may invoke this doctrine to challenge the obstruction
and maintain access.

16-B
The preamble of a constitution articulates the fundamental values and goals that the
constitution seeks to achieve.

17-B
Civil law pertains to private disputes between individuals or entities, often involving
issues like contracts or property.

18-D
Habeas corpus is a legal action that protects an individual's right to be free from
unlawful detention.

19-B
The prosecution must prove the guilt of the defendant beyond a reasonable doubt in a
criminal trial.

20-B
Appellate courts review decisions made by lower courts to ensure they were correctly
applied and interpreted.

www.careers360.com Back to Index 205


21-C
The Fourth Amendment protects individuals from unreasonable searches and seizures
by the government.

22-B
The doctrine of precedent means that a court should follow the decisions of higher
courts in similar cases.

23-B
The age of majority is the age at which a person is legally considered an adult and is
typically set at 18 in many jurisdictions.

24-D
A confidentiality agreement is designed to prevent the unauthorized disclosure of
sensitive information.

ANALYTICAL REASONING
25-A
Let the actual distance travelled be
Then,

26-B
Let the speed of the car be x \mathrm{kmph}.

Then, speed of the train

www.careers360.com Back to Index 206


27-B

28-A
Ram completes 60% of the task in 15 days.
i.e., he completes 4% of the task in a day.
Rahim is 50% as efficient as Ram is.
Therefore, Rahim will complete 2% of the task in a day.
Rachel is 50% as efficient as Rahim is
Therefore, Rachel will complete 1% of the task in a day.
Together, Ram, Rahim and Rachel will complete 4 + 2 + 1 = 7% of the work in a day.
They have another 40% of the task to be completed.
Therefore, they will take 40/7 more days to complete the task.

29-A
Part filled by pipe A in 1 hour = 1/12
Part filled by pipe B in 1 hour = 1/15
Part filled by pipe C in 1 hour = 1/20
In first hour, A and B is open
In second hour, A and C is open
then this pattern goes on till the tank fills
Part filled by pipe A and pipe B in 1 hour = 1/12+1/15=9/60=3/20
Part filled by pipe A and pipe C in 1 hour = 1/12+1/20=8/60=2/15
Part filled in 2 hour = 3/20+2/15=17/60
Part filled in 6 hour =17/60×3=17/20 Remaining part = (1−17/20)=3/20

www.careers360.com Back to Index 207


Now, 6 hours are over and only 3/20 part needed to be filled. At this 7th hour, A and B is
open Time taken by pipe A and B to fill this 3/20 part = 1 hour
Total time taken = 6 hour + 1 hour = 7 hour

30-C
90 47

94 = 3:4

3 4

First solution = 3/7 x 21 = 9L

31-A
From the concept, the opposite side of a parallelogram is equal.

Let ABCd is parallelogram, AB = 10 cm and AB and BC are adjacent side

AB = CD and AD = BC (opposite sides)

AB+BC+CD+DA = 50

10+BC+10+DA = 50

BC+DA+20=30

BC=15

www.careers360.com Back to Index 208


BC = DA = 15 cm

32-B

Radius of the capsule = cm = 0.5 cm

Let the length of cylindrical part be x cm

33-A

34-A
Ratio of time = 1/2 : 1/3 : ¼
= 6 : 4: 3

35-B
The shaded part in the diagram given below represents papers that are also hard.

www.careers360.com Back to Index 209


Papers that are also hard→14 + 9 = 23

23 papers are also hard. Hence, the second option is correct.

36-A
Given:
The weights of 4 boxes are 90, 40, 80, and 50 kilograms.

The total weight of all possible combinations of boxes is as follows –


90 + 40 = 130; 90 + 80 = 170; 90 + 50 = 140; 40 + 80 = 120;
40 + 50 = 90; 80 + 50 = 130; 90 + 40 + 80 = 210;
90 + 40 + 50 = 180; 90 + 80 + 50 = 220; 40 + 80 + 50 = 170;
40 + 80 + 50 + 90 = 260

There is no total weight of 200 kilograms in any combination. Hence, the first option is
correct.

READIN G COMPREHENSION

37-B
The second option is the correct answer: lend.

let: to allow someone to use or occupy something, often temporarily.

lend: to give something to someone with the expectation of getting it back, typically
money or an item.

borrow: to take something from someone with the intention of returning it, typically
money or an item.

lease: to rent or hire something, such as property or equipment, for a specified period in
exchange for payment.

Therefore, the corrected sentence is "The bank decided to lend them the money as
they were reliable clients."

38-A
The first option is the correct choice because "retrieve" means to get or bring
something back from somewhere, as per the context of the question.

The meanings of the other options are as follows:

● Retrace means to get or bring something back from somewhere.

www.careers360.com Back to Index 210


● Retract means to go back over the same route that one has just taken.
● Retreat means an act of moving back or withdrawing.

Therefore, the correct sentence is, "The flood damaged the books so much that it was
impossible to retrieve them".

39-A
The correct choice is the first option, "through".

The preposition "through" correctly expresses the intended meaning of the sentence
that she was aware of what her father was thinking.

Therefore, the complete sentence is, "She was aware of what was going through her
father's mind."

40-A
The correct choice is the first option, "pour".

The word "pour" is the correct choice as it means to flow something in large amounts,
and it helps convey the intended meaning that parents give large amounts of love and
affection to their children.

The meanings of the other options are as follows:

● "Pore" means a minute opening on a surface.


● "Poor" means someone who lacks money in order to live at a basic standard.
● "Pool" means a small area of still water.

Therefore, the correct sentence is, "Most parents pour love and affection on their
children."

41-B
The correct choice is the second option, "became".

The verb "became", which is the second form of the verb "begin", which means begins
to be, is the correct form of the verb that should be used in the sentence, as it helps
maintain the tense consistency and also maintains the subject-verb agreement in the
sentence.

Therefore, the correct sentence is, "Then railway trains became the most popular and
the cheapest means of transport."

42-C
The correct choice is the third option, "sore".

www.careers360.com Back to Index 211


"Sore" means a raw and painful place on one's body. It is the correct choice, as it helps
convey the intended meaning of the sentence that even a small cut can turn painful if
left unattended.

The meanings of the other options are as follows:

● "Soar" means to increase rapidly above the usual level.


● "Sour" means having an acidic taste like lemon or vinegar.
● "Ore" is a naturally occurring solid material from which a material is extracted.

Therefore, the correct sentence is, If left unattended, even a small cut can turn into a
sore."

43-C
The correct choice is the third option, "pessimistic".

A pessimistic person is a person who always expects that the worst will happen or
looks at the negative aspects of things. This is the correct word choice for this sentence,
which helps in conveying the intended meaning that the neighbour is the type of person
who believes the worst will happen to him.

The meanings of the other options are as follows:

● "Reasonable" means fair and sensible.


● "Forward-looking" is a person who plans for the future.
● An optimistic person is a person who expects good outcomes in his life.

Therefore, the correct sentence is, "My neighbour is very pessimistic for he believes
that nothing good will happen to him."

44-D
The fourth option is the correct answer: impose.

Expose: To reveal or make something visible or known, often something hidden or


concealed.

Interpose: To place or insert something between two things or people; to intervene in a


conversation or situation.

Express: To convey or communicate feelings, thoughts, or information; to show or


make something known clearly.

Impose: To establish or apply a rule, law, tax, or obligation on someone; to force or


place a burden on someone.

www.careers360.com Back to Index 212


Therefore, the correct answer is "Do not impose your own personal views upon the
reader."

45-C
The third option is the correct answer: scolded.

The sentence is set in the past tense. Hence, the missing word must be in the past
simple tense as well.

Therefore, the correct answer is "She scolded him angrily because he made the floor
dirty with his muddy shoes."

46-B
The correct choice is the second option, "on".

"On" is the correct preposition, as it helps in indicating that the subject (she) focuses
too much on her past.

Therefore, the correct sentence is, "She dwells too much on her past."

47-A

The correct option that expresses the given sentence in the passive voice is: The
teacher was compelled to finish the class sooner.

In the passive voice, the structure is generally: Subject (Receiver of the action)+to be
verb (am, is, are, was, were, etc.)+{past participle of the main verb}} + optional: by +
Agent (doer of the action). Passive voice is the voice in which the object gets the
preference.

The active voice sentence. We compelled the teacher to finish the class sooner is
transformed into the passive voice by making the teacher the subject of the sentence
and using the appropriate passive voice structure.

48-B

The first sentence should be B since it gives general information about London. Option
2 can be selected directly to save time. Sentence A is also supporting sentence B
therefore, it should come next. Sentence D mentions the consequence of the previous 2
sentences. Thus, it should be next. Sentence C shows a contrasting view to the
previous sentences. Therefore, it should be at the end.

www.careers360.com Back to Index 213


Therefore, the correct order is BADC.

GENERAL KNOWLEDGE
48-D
The 12th Schedule of the Indian Constitution deals with the provisions that specify the
powers, authority and responsibilities of Municipalities. This schedule was added by the
74th Amendment Act of 1992. It has 18 subjects.

49-A
There shall be a Council of Ministers with the Prime Minister at the head to aid and
advise the President, who shall act by such advice in his functions. The question of
whether any and, if so, what advice was tendered by Ministers to the President shall not
be inquired into in any court.

50-A
Lake Tahoe is the largest alpine lake in North America, and the second deepest lake in
the US, with Crater Lake in Oregon being the deepest in the US.

51-C
A tropical monsoon climate is characterized by dry winters and wet summers due to the
seasonal reversal of winds.

52-D
The Investor Education and Protection Fund Authority (IEPFA) was established by the
Ministry of Corporate Affairs to manage the Investor Education and Protection Fund. It
focuses on refunding shares, unclaimed dividends, matured deposits, and debentures
to investors, among other responsibilities

53-A
The correct option is Lord Canning.

Lord Canning was a British diplomat who held the positions of Governor-General and
first Viceroy of India during the 1857 Indian uprising and when the East India
Company's power was transferred to Queen Victoria's throne in 1858 following the
suppression of the uprising. Lord Dalhousie drafted the Hindu Widows' Remarriage Act
1856 before the rebellion and Canning approved it.

www.careers360.com Back to Index 214


54-D
The correct answer is Muhammad Ali Jinnah

In December 1916, the historic Lucknow Pact was signed. The Muslim League and the
Indian National Congress had an agreement. Ambika Charan Mazumdar served as
Indian National Congress president during the Lucknow Pact. Muhammad Ali Jinnah,
whom Sarojini Naidu referred to as the Ambassador of Hindu-Muslim Unity, was
instrumental in bringing about the agreement between the two groups.

55-C
The correct answer is Gujarat, Maharashtra, Rajasthan and Madhya Pradesh.

To settle disagreements over water sharing amongst the Indian states that make up the
Narmada River basin, the Narmada Water Dispute Tribunal was established. The
Narmada River is one of the major rivers in central India, and it flows through several
states. The tribunal was established to adjudicate the allocation of Narmada river waters
among the riparian states, which include Gujarat, Maharashtra, Rajasthan, and Madhya
Pradesh.

56-B
The answer is Dayanand Saraswati.

Swami Dayananda Saraswati was born in Gujarat. He was a social reformer, and he
formed an organisation called Arya Samaj. Bombay served as the headquarters of this
organisation, but later it was shifted to Lahore. He considered the Vedas to be infallible
and the sacrosanct book of Hindu literature. He also gave the slogan "Go Back to the
Vedas".

57-B
The answer is five years.

After the passage of the Jammu and Kashmir Reorganisation Act, 2019, the state of
Jammu and Kashmir was bifurcated into the Union Territory of Jammu & Kashmir and
Ladakh, and Article 370 of the Indian constitution stands abrogated.

Earlier, the legislative assembly of the state had a tenure of six years but is now
reduced to five years. Now, in Jammu & Kashmir, the total number of assembly seats is
90, of which 43 are marked for Jammu and 47 are for Kashmir.

58-A
The correct option is vitamin A.

www.careers360.com Back to Index 215


Fish liver oil, such as cod liver oil, is high in vitamin A and other vital elements. Vitamin
A is a fat-soluble vitamin that is essential for many body activities such as eyesight,
immune system health, reproduction, and cellular communication. Because of its high
vitamin A concentration, fish liver oil is an excellent source of this crucial nutrient.

59-B
The correct option is starfish.

Despite its name, a sea star is not a real fish. Echinoderms are marine invertebrates
that include starfish (sea stars), sea urchins, sand dollars and sea cucumbers. A starfish
does not have the characteristics of a fish.

Fish are aquatic vertebrates classified as Osteichthyes or Chondrichthyes. They are


distinguished by the presence of fins, gills and a streamlined body designed for
swimming in water.

60-B
The correct answer is The Governor

The Governor of the State appoints the Chairman and members of the State Public
Service Commission. The Chairman and members of the Joint Public Service
Commission are chosen by the Indian President. All of the tasks carried out by the
UPSC at the federal level are completed by State Public Service Commissions. A state
PSC is therefore responsible for the following: Exams are held there in preparation for
the appointment of state workers.

www.careers360.com Back to Index 216


MOCK TEST - 7
____________________________________________________________________________

LOGICAL REASONING
Q1.Direction: In the following question below, some statements are followed by
some conclusions. Taking the given statements to be true, even if they seem to
be at variance from commonly known facts, read all the conclusions and then
decide which of the given conclusions logically follows from the given
statements.
Statements:
I. All dogs are bullocks.
II. All bullocks are goats.
Conclusions:
I. All dogs are goats.
II. All goats are dogs.

A) Conclusion I follows
B) Conclusion II follows
C) Neither conclusion I nor conclusion II follows
D) Both conclusion I and conclusion II follows

Q2.Direction: In the following question below, some statements are followed by


some conclusions. Taking the given statements to be true, even if they seem to
be at variance from commonly known facts, read all the conclusions and then
decide which of the given conclusions logically follows from the given
statements.

Statements:
1. All young girls are modern.
2. No modern men are superstitious.
Conclusions:
I. No girls are superstitious.
II. No young men are superstitious.

A) Conclusion I follows
B) Conclusion II follows
C) Neither conclusion I nor conclusion II follows
D) Both conclusion I and conclusion II follows

www.careers360.com Answer 217


Q3.Assertion: Earth's magnetic field is generated by the motion of molten iron in
its outer core.
Reason: The motion of molten iron creates electric currents, which generate a
magnetic field through the geodynamo effect.

A) Both the assertion and reason are true, and the reason is a correct explanation of the
assertion.
B) Both the assertion and reason are true, but the reason is not a correct explanation of
the assertion.
C) The assertion is true, but the reason is false.
D) The assertion is false, but the reason is true.

Q4.Assertion:DNA replication is a semi-conservative process.


Reason:In DNA replication, each newly formed DNA strand contains one original
strand and one newly synthesized strand.

A) Both the assertion and reason are true, and the reason is a correct explanation of the
assertion.
B) Both the assertion and reason are true, but the reason is not a correct explanation of
the assertion.
C) The assertion is true, but the reason is false.
D) The assertion is false, but the reason is true.

Q5.According to the given statements, Which of the following options is correct.

Statement 1:
Sound travels faster in water than in air.
Statement 2:
Sound travels faster in a vacuum than in water.

A) Statement 1 is correct, but Statement 2 is incorrect.


B) Statement 2 is correct, but Statement 1 is incorrect.
C) Statement 1 is incorrect.
D) Statement 2 is incorrect.

Q6.According to the given statements, Which of the following options is correct


Statement 1:
The largest planet in our solar system is Earth.

www.careers360.com Answer 218


Statement 2:
The smallest planet in our solar system is Jupiter.

A) Statement 1 is correct, but Statement 2 is incorrect.


B) Statement 2 is correct, but Statement 1 is incorrect.
C) Statement 1 is incorrect.
D) Statement 2 is incorrect.

Q7. Direction: In the following question, a statement is given, followed by two


arguments, I and II. You have to consider the statement to be true, even if it
seems to be at variance from commonly known facts. You have to decide which
of the given arguments, if any, follows from the given statement.

Statement:
"Should schools switch to digital textbooks instead of printed ones?"
Arguments:
I. Yes, digital textbooks are more environmentally friendly and cost-effective.
II. No, not all students have access to digital devices, and print is a reliable
medium.

A) Only Argument I follows.


B) Only Argument II follows.
C) Both I and II follows
D) Neither Argument I nor Argument II follows.

Q8. Direction: In the following question, a statement is given, followed by two


arguments, I and II. You have to consider the statement to be true, even if it
seems to be at variance from commonly known facts. You have to decide which
of the given arguments, if any, follows from the given statement.

Statement:
"Should the government provide universal basic income to all citizens?"
Arguments:
I. Yes, it can alleviate poverty and ensure a minimum standard of living.
II. No, it discourages people from seeking employment and can lead to economic
instability.

A) Only Argument I follows.


B) Only Argument II follows.
C) Both I and II follows

www.careers360.com Answer 219


D) Neither Argument I nor Argument II follows.

Q9.Direction: In the following question, select the odd one from the given
alternatives.

A) BEHK
B) JMPS
C) PSVY
D) EHKM

Q10. Direction: In the following question, select the odd one from the given
alternatives.

A) AD
B) PT
C) EH
D) JM

Q11. Direction: A man and a woman are sitting in a room. The man's
mother-in-law and the woman's mother-in-law are mother and daughter,
respectively. How is the man related to the woman?

A) Father
B) Father-in-law
C) Grandfather
D) Brother

Q12. Direction: Aman, Rohit, Suresh, Danish, and Alok are arranged in
descending order of weight. Aman is third from the top. Alok is between Danish
and Aman, while Danish is not at the top. Who is at the top?

A) Aman
B) Rohit
C) Suresh
D) Cannot be determined

www.careers360.com Answer 220


LEGAL REASONING

Q13: In a criminal trial, what is the standard of proof required for a conviction?

A) Beyond a reasonable doubt


B) Balance of probabilities
C) Clear and convincing evidence
D) Preponderance of the evidence

Q14.What is the "Miranda warning" designed to inform individuals of?

A) The right to remain silent


B) The right to an attorney
C) The right to a fair trial
D) All of the above

Q15.Under which legal doctrine can a person use force, even deadly force, to
protect their home from an intruder?

A) Self-defense
B) Castle doctrine
C) Stand your ground
D) Defense of property

Q16.What is the purpose of a subpoena in legal proceedings?

A) To request documents
B) To compel testimony
C) Both A and B
D) None of the above

Q17.What is the term for a legal dispute resolution process in which a neutral
third party helps the parties reach a settlement?

A) Mediation
B) Arbitration
C) Litigation

www.careers360.com Answer 221


D) Negotiation

Q18.What legal principle allows a court to disregard precedent and make


decisions based on fairness and justice in unique cases?

A) Stare decisis
B) Equity
C) Res ipsa loquitur
D) Habeas corpus

Q19.In contract law, what term refers to a statement that is crucial to the
formation of a contract and induces the other party to enter into the agreement?

A) Representation
B) Warranty
C) Misrepresentation
D) Material fact

Q20.Under what legal doctrine can a person be held criminally liable for assisting
or encouraging another person to commit a crime?

A) Conspiracy
B) Accomplice liability
C) Mens rea
D) Duress

Q21.What legal concept allows a defendant to be tried again for the same offense
after an earlier trial ended without a verdict?

A) Double jeopardy
B) Due process
C) Ex post facto
D) Habeas corpus

Q22.Which legal doctrine allows a government to take private property for public
use with just compensation to the property owner?

A) Eminent domain

www.careers360.com Answer 222


B) Escheat
C) Adverse possession
D) Nuisance

Q23.In a defamation case, which defense allows a defendant to escape liability by


proving that the statement was true?

A) Privilege
B) Absolute immunity
C) Substantial truth
D) Fair comment
Q24.:What legal doctrine allows a court to assume a party's guilt if they refuse to
testify in a criminal trial?

A) Hearsay
B) Adverse inference
C) Self-incrimination
D) Exclusionary rule

ANALYTICAL REASONING

Q25. Excluding stoppages, the speed of a bus is and including


stoppages, it is . For how many minutes does the bus stop per hour?

A] 9

B] 10

C] 12

D] 20

Q26. In a flight of an aircraft was slowed down due to bad weather. Its
average speed for the trip was

reduced by and the time of flight increased by 30 minutes. The


duration of the flight is:

A] 1 hour

B] 5 hour

www.careers360.com Answer 223


C] 6 hour

D] 7 hour

Q27. A boat goes a certain distance at 30 km/hr and comes back the same
distance at 60 km/hr. What is the average speed (in km/hr) for the total journey?

A] 45
B] 40
C] 35
D] 50

Q28. X can do a piece of work in 20 days working 7 hours a day. The work is
started by X and on the second day one man whose capacity to do the work is
twice that of X, joined. On the third day another man whose capacity is thrice that
of X, joined and the process continues till the work is completed. In how many
days will the work be completed, if everyone works for four hours a day?
A] 5 days
B] 8 days
C] 6 days
D] 4 days

Q29.One pipe can fill a tank three times as fast as another pipe. If together the
two pipes can fill the tank in 36 minutes, then the slower pipe alone will be able to
fill the tank in:

A] 81 min.
B] 108 min.
C] 144 min.
D] 140 min.

Q30. There are two solutions of sulphuric acid (acid + water) with concentration
of 50% and 80% respectively. They are mixed in a certain ratio to get a 62%
sulphuric acid solution. This solution is mixed with 6 litres of water to get
back 50% solution. How much of the 80% solution has been used in the entire
process?

A] 15 litres
B] 12 Litres
C] 10 Litres

www.careers360.com Answer 224


D] None of these

Q31. The measure of sum of interior angle of a regular heptagon is

A] 900°

B] 300°

C] 600°

D] 400°

Q32. The slant height of a bucket is 45 cm and the diameter of its top and bottom
are 56 cm and 14 cm respectively. The curved surface area (in cm2) of the bucket
is

A] 4025
B] 5045
C] 4950
D] 1020

Q33. Find the surface area of a sphere in cm2 whose volume is 4851 cm3.

A] 1300
B] 1366
C] 1370
D] 1386

Q34. The ratio of two numbers is 2:5. Their LCM is 120. Determine the second
number.

A] 24
B] 45
C] 60
D] 85

www.careers360.com Answer 225


Q35.Directions: Which of the following diagrams represents the relationship among
Omnivores, Bear, and Deer?

A)

B)

C)

D)

www.careers360.com Answer 226


Q36.Directions: Which of the following diagrams represents the relationship
between Furniture, Chair, and Table?

A)

B)

C)

D)

READING COMPREHENSION

Q37. Directions: In the given question, a sentence is given with a blank to be filled
in with an appropriate word. Four alternatives are suggested for each question.
Choose the correct alternative out of the four alternatives.

He is getting married _________ Maya.

www.careers360.com Answer 227


A) only

B) by

C) with

D) to

Q38.Directions: In the given question, a sentence is given with a blank to be filled


in with an appropriate word. Four alternatives are suggested for each question.
Choose the correct alternative out of the four alternatives.

Medical doctors should try to ________ as many patients as possible.

A) heel

B) heal

C) kill

D) hate

Q39.Direction: In the given question, a sentence is given with a blank to be filled


in with an appropriate word. Four alternatives are suggested for each question.
Choose the correct alternative out of the four alternatives.

Can you tell the difference _________ butter and margarine?

A) over

B) with

C) between

D) among

Q40.Directions: In the given question, a sentence is given with a blank to be filled


in with an appropriate word. Four alternatives are suggested for each question.
Choose the correct alternative out of the four alternatives.

Mr. Sen is absent today. He ________ sick.

A) has to be

B) could have been

www.careers360.com Answer 228


C) might be

D) must have been

Q41. Directions: In the following question, a sentence is given with a blank that is
to be filled in with an appropriate word. Four alternatives are suggested; Choose
the correct alternative out of them as your answer.

I always fall ______ old friends in times of need.

A) over

B) through

C) back on

D) off

Q42.Directions: In the given question, a sentence is given with a blank to be filled


in with an appropriate word. Four alternatives are suggested for each question.
Choose the correct alternative out of the four alternatives.

Will all of you ______ up for prayer?

A) raise

B) rise

C) raze

D) race

Q43.Directions: In the given question, a sentence is given with a blank to be filled


in with an appropriate word. Four alternatives are suggested for each question.
Choose the correct alternative out of the four alternatives.

The art of cooking _______ in ancient India.

A) is perfected

B) will perfect

C) perfected

D) was perfected

www.careers360.com Answer 229


Q44.Directions: In the given question, a sentence is given with a blank to be filled
in with an appropriate word. Four alternatives are suggested for each question.
Choose the correct alternative out of the four alternatives.

A number of refugees _________ been turned back at the border.

A) are

B) has

C) is

D) have

Q45.Directions: In the given question, a sentence is given with a blank to be filled


in with an appropriate word. Four alternatives are suggested for each question.
Choose the correct alternative out of the four alternatives.

The new government took __________ last year.

A) after

B) over

C) upon

D) out

Q46.Directions: In the given question, a sentence is given with a blank to be filled


in with an appropriate word. Four alternatives are suggested for each question.
Choose the correct alternative out of the four alternatives.

Mohan's career has taken some _____ twists and turns.

A) interesting

B) interactive

C) intuitive

D) incentive

Q47. Directions: In the given question, a sentence is given with a blank to be filled
in with an appropriate word. Four alternatives are suggested for each question.
Choose the correct alternative out of the four alternatives.

www.careers360.com Answer 230


Sheila gained an advantage ____ me.

A) from

B) on

C) over

D) upon

Q48.Directions: In the given question, a sentence is given with a blank to be filled


in with an appropriate word. Four alternatives are suggested for each question.
Choose the correct alternative out of the four alternatives.

She was remarkably _______ in singing and dancing.

A) conductive

B) fluctuating

C) cooperative

D) accomplished

GENERAL KNOWLEDGE

Q49.In which article of the Indian Constitution have the directive principles of the
state been described?

A) Article 26-46
B) Article 31-50
C) Article 36-51
D) Article 38-52

Q50."Central Bureau of Intelligence and Investigation" is listed in the ________list


given in the Seventh Schedule in the Constitution of India.

A) union
B) state
C) global
D) concurrent

www.careers360.com Answer 231


Q51.Adamsia (Sea anemone), Pennatula (Sea – pen) are examples of which
Phylum?

A) Coelenterata
B) Aschelminthes
C) Annelida
D) Ctenophera

Q52.First Leader of opposition in Loksabha was __________.

A) B. R. Ambedkar
B) A. K. Gopalan
C) S Radhakrishnan
D) Vallabhbhai Patel

Q53.The speaker of the Lok Sabha has to address his/her letter of resignation to

A) Prime Minister of India


B) President of India
C) Deputy speaker of lok sabha
D) Minister of parliamentary affairs

Q54.Fundamental duties are mentioned in which of the following parts of the


Indian Constitution?

A) Part II
B) Part III
C) Part V
D) Part IV A

Q55. Which of the following is justiciable?

A) Fundamental Duties
B) Directive Principles of State Policy
C) Fundamental Rights
D) None of these

Q56. Which of the following is not a fundamental duty?

A) To abide by the Constitution and respect the National Flag


B) To promote harmony and brotherhood
C) To uphold and protect the sovereignty
D) Abolition of titles except military and academic

www.careers360.com Answer 232


Q57. ____________ is separated from India by a narrow channel of Sea formed by
Palk Strait and the Gulf of Mannar.

A) Sri Lanka

B) Myanmar

C) Bangladesh

D) Pakistan

Q58. The land mass of India has an area of ______________ million square km.

A)1.28

B)2.28

C)3.28

D)4.28

Q59.Right to Privacy comes under

A) Article 19

B) Article 20

C) Article 21

D) Article 18

Q60.Palk Strait separates India with which country?

A) Pakistan

B) Bangladesh

C) Indonesia

D) Sri Lanka

www.careers360.com Answer 233


ANSWER KEY
____________________________________________________________________________

1 2 3 4 5 6 7 8 9 10
A C A A B D A A D B
11 12 13 14 15 16 17 18 19 20
B D A D B B A B D B
21 22 23 24 25 26 27 28 29 30
B A C B B A B A C C
31 32 33 34 35 36 37 38 39 40
A C D C A A D B C C
41 42 43 44 45 46 47 48 49 50
C B D D B A C D C A
51 52 53 54 55 56 57 58 59 60
A B C D C D A C C D

www.careers360.com Solutions 234


SOLUTIONS
____________________________________________________________________________

LOGICAL REASONING

1-A
Statements:
I. All dogs are bullocks.
II. All bullocks are goats.
The Venn diagram is

Let's analyze the conclusions


Conclusion (I): All dogs are goats.
From the Venn diagram, we can see that the circle that represents dogs lies completely
inside the circle that represents goats. This means that all the dogs are goats. So, this
conclusion follows.
Conclusion (II): All goats are dogs.
From the Venn diagram, we can see that some goats lie outside the circle that
represents dogs. This means that some goats are not dogs. So, this conclusion does
not follow.

Thus, only conclusion I follows. Hence, the first option is correct.

2-C
Given:
Statements:
1. All young girls are modern.

www.careers360.com Back to Index 235


2. No modern men are superstitious.
The venn diagram is:

Let's analyze the conclusions


Conclusion (I): No girls are superstitious.
The statements only mention young girls. None of the statements contain any
information about girls. So, This conclusion does not follow.
Conclusion (II): No young men are superstitious.
The statements only mention modern men. None of the statements contain any
information about young men. So, This conclusion does not follow.

3-A
Earth's magnetic field is indeed generated by the motion of molten iron in its outer core,
which creates electric currents and leads to the geodynamo effect, producing the
magnetic field.

4-A
DNA replication is a semi-conservative process because each new DNA molecule
formed has one strand from the original DNA molecule and one newly synthesized
strand. This is crucial for genetic inheritance.

5-B
Sound does not travel in a vacuum, so it is faster in water compared to a vacuum.

6-D
Earth is not the largest planet in our solar system, and Jupiter is not the smallest.

7-A
Argument I supports the idea of switching to digital textbooks for environmental and
cost-saving reasons. Argument II points out potential issues with digital textbooks, such
as accessibility and the reliability of print.

www.careers360.com Back to Index 236


8-A
Argument I supports universal basic income for its potential to alleviate poverty and
ensure a minimum standard of living. Argument II raises concerns but doesn't directly
refute the benefits mentioned in Argument I.

9-D
Let's check the options –

First option: BEHK → B + 3 = E, E + 3 = H, H + 3 = K

Second option: JMPS → J + 3 = M, M + 3 = P, P + 3 = S

Third option: PSVY → P + 3 = S, S + 3 = V, V + 3 = Y

Fourth option: EHKM → E + 3 = H, H + 3 = K, K + 3 = N

Therefore, the fourth option is different from the other three. So, it's an odd one out.

Hence, the fourth option is correct.

10-B
Let's check the options —

First option: AD → A + 3 = D

Second option: PT → P + 3 = S

Third option: EH → E + 3 = H

Fourth option: JM → J + 3 = M

Therefore, the second option is different from the other three. So, it's an odd one out.

Hence, the second option is correct.

www.careers360.com Back to Index 237


11-B
First, we need to draw a family tree.

Now, we see that –


The woman's mother-in-law is the daughter of the man's mother-in-law.
The man is the husband of the woman's mother-in-law.

So, from the above, the man is the father-in-law of the woman. Hence, the second
option is correct.

12-D
Given:
Aman is third from the top. Alok is between Danish and Aman, while Danish is not at the
top.

The weights from the given question are listed in descending order –

Rohit/Suresh

Suresh/Rohit

Aman

www.careers360.com Back to Index 238


Alok

Danish

It's unclear who is at the top, although it might be either Rohit or Suresh.

So, the answer cannot be determined. Hence, the fourth option is correct.

LEGAL REASONING

13-A
In criminal cases, the prosecution must prove the defendant's guilt beyond a reasonable
doubt, the highest standard of proof.

14-D
The Miranda warning informs individuals of their rights to remain silent and to have an
attorney present during police questioning to ensure a fair trial.

15-B
The Castle doctrine allows individuals to use force, including deadly force, to protect
their home from intruders without the duty to retreat.

16-B
A subpoena can be used to compel the production of documents or the attendance of a
witness to testify in a legal proceeding.

17-A
Mediation involves a neutral third party facilitating communication between parties to
help them reach a mutually acceptable resolution.

18-B
Equity allows courts to consider principles of fairness and justice in cases where strict
application of the law may lead to an unjust outcome.

19- D
A material fact is a crucial piece of information that, if known, might have influenced a
party's decision to enter into a contract.

20-B

www.careers360.com Back to Index 239


Accomplice liability holds individuals accountable for aiding, abetting, or encouraging
the commission of a crime.

21-B
Due process ensures fairness in legal proceedings and may allow retrial under certain
circumstances, such as a mistrial.

22-A
Eminent domain gives the government the power to take private property for public use,
provided the owner is justly compensated.

23-C
The defense of substantial truth in defamation cases acknowledges that minor
inaccuracies won't negate the overall truth of the statement.

24-B
Adverse inference allows a court to draw a negative conclusion about a party who
refuses to testify, considering it as evidence against them.

ANALYTICAL REASONING

25-B
Due to stoppages, it covers less.

Time taken to cover

26-A
Let the duration of the flight be x hours.

www.careers360.com Back to Index 240


27-B

28-A
Since X takes 20 days working 7 hours a day to complete the work, the number of
day-hours required to complete this work would be 140 day-hours. Like in the two
problems above, this is going to be constant throughout. So, W = 140 day-hours.
Amount of work done in the 1st day by X = 1day × 4 hours = 4 day-hours
2nd day, X does again 4 day-hours of work. The second person is twice as efficient as X
so he will do 8 day-hours of work. Total work done on second day = 8 + 4 = 12
day-hours. Amount of work completed after two days = 12 + 4 = 16 day-hours.
3rd day, X does 4 day-hours of work. Second Person does 8 day-hours of work. Third
person who is thrice as efficient as X does 12 day-hours of work. Total work done on 3rd
day = 4 + 8 + 12 = 24 day-hours Amount of work completed after 3 days = 16 + 24 = 40
day-hours
Similarly on 4th day the amount of work done would be 4 + 8 + 12 + 16 = 40 day-hours
Work done on the 5th day = 4 + 8 + 12 + 16 + 20 = 60 day-hours
Total work done after 5 days = 4 + 12 + 24 + 40 + 60 = 140 day-hours = W. So it takes 5
days to complete the work.

29-C
Let the slower pipe alone fill the tank in x minutes.

Then, faster pipe will fill it in minutes.

www.careers360.com Back to Index 241


30-C

50 80

62 = 3:2

18 12
3x/2x+6 = 1/1 = 3x = 2x+6
x=6
Quantity of 80 % solution = 6X2 = 12 L

31-A
Sum of the interior angles of a convex polygon of n sides = (2n – 4)90°.

For heptagon, n = 7

So sum of angle is (2x7-4)90° = 10 x 90° = 900°

32-C
Here, l = 45 cm, R = 28 cm and r = 7 cm.

33-D
Let the radius of the sphere be r cm.

Then,

www.careers360.com Back to Index 242


Thus, the radius of the sphere is 10.5 cm.

34-C
2 : 5 = 120
2x : 5x = 120
10x = 120
x = 12
Second number is 60

35-A
Deer only eats grass. This means that deer is a herbivore and not an omnivore. But, Bear
is both carnivore and herbivore, i.e. Bear is an omnivore. So, all bears are omnivores
while no deer is omnivore. Deer and bears are not related to each other.
So, the Venn diagram is –

Hence, the first option is correct.

www.careers360.com Back to Index 243


36-A
We know that both chairs and tables are furniture. So, their diagrams will lie inside that
of furniture. But, chairs and tables are not related to each other. So, their diagrams will
not have a common area.
The Venn diagram will be –

Hence, the first option is correct.

READING COMPREHENSION

37-D

The preposition "to" is the correct choice in this sentence because it is used to indicate
the person to whom someone is getting married. In this context, "to" is the appropriate
preposition to connect the subject (he) with the person he is marrying (Maya).

The meanings of the other options are as follows:

● "Only" is an adverb and doesn't convey the intended meaning of who he is


marrying.
● "By" and "with" are not the appropriate prepositions for expressing the idea of
marriage in this context.

Therefore, the correct sentence is, "He is getting married to Maya."

38-B

The word "heal" is the correct choice in this sentence because it means to cure or treat
patients, helping them recover from their illnesses or injuries. It's the appropriate word
for what medical doctors are supposed to do.

www.careers360.com Back to Index 244


The meanings of the other options are as follows:

● "Heel" means the back part of the foot or to follow someone closely, which is
unrelated to medical treatment.
● "Hate" means to feel strong hostility or intense dislike.
● "Kill" means to cause death, which is the opposite of what doctors are expected
to do.

Therefore, the correct sentence is, "Medical doctors should try to heal as many patients
as possible."

39-C

The preposition "between" is the correct choice in this sentence because it is used to
indicate the distinction or comparison between two items, in this case, butter and
margarine. When you want to highlight the contrast or distinguish one thing from
another, "between" is the appropriate preposition.

The meanings of the other options are as follows:

● "Over" typically implies covering something or dealing with a range of issues, not
making a comparison.
● "With" is used to indicate an association, not a distinction or difference between
two items.
● "Among" is used to refer to a relationship within a group of three or more items,
not a comparison between two specific items.

Therefore, the correct sentence is, "Can you tell the difference between butter and
margarine?"

40-C

"Might" is the simple past form of "may," which is used to express the possibility that
the given condition can be true, although not very likely. "Might be" is used here to
indicate the possibility that Mr. Sen can be sick, and that is why he is absent.

Therefore, the correct sentence is, "Mr. Sen is absent today. He might be sick."

41-C

Explanation: The phrase "fall back on" is a common expression that means to rely on
or turn to something or someone for help or support in difficult times. In this context, it is

www.careers360.com Back to Index 245


used to indicate that the speaker relies on old friends when they are in need of
assistance or support.

The meanings of other options are as follows:

● Over: Across or above; indicating movement or position from one side to


another.

● Through: Moving from one side to the other; indicating completion, passage, or
penetration.

● Off: Away from a surface or position; indicating separation or cessation.

So, the sentence would be: "I always fall back on old friends in times of need."

42-B

The word "rise" is the appropriate choice in this sentence because it means to move
from a lower to a higher position, which is appropriate in the context of standing up for
prayer.

The meanings of the other options are as follows:

● "Race" means to compete in a contest of speed, which is not relevant here.


● "Raise" means to lift or elevate something, but it's not suitable for asking people
to stand up.
● "Raze" means to completely destroy, which is not related to the act of standing
up for prayer.

Therefore, the correct sentence is, "Will all of you rise up for prayer?"

43-D

The phrase "was perfected" is the appropriate choice in this sentence because it
indicates that the art of cooking reached a state of perfection in the past, specifically in
ancient India. This form of the verb "perfected" in the past tense is appropriate to
describe an action that was completed at a specific time in the past.

The meanings of the other options are as follows:

● "Is perfected" is in the present tense and doesn't correctly convey the past
action.

www.careers360.com Back to Index 246


● "Perfected" is not the correct choice as it does not fit in the sentence
grammatically.
● "Will perfect" is in the future tense, which is not suitable for describing an action
in ancient India.

Therefore, the correct sentence is, "The art of cooking was perfected in ancient India."

44-D

The auxiliary verb "have" is the correct choice in this sentence because "a number of
refugees" is a plural subject, and when the subject is plural, you use the plural form of
the verb, which is "have," as it shows possession.

The meanings of the other options are as follows

● "has" is the singular form of the verb and doesn't match the plural subject.
● "is" is also singular and doesn't match the plural subject.
● "are" is plural, but it's not used with "number of" to refer to a quantity of
something.

Therefore, the correct sentence is, "A number of refugees have been turned back at the
border."

45-B

The word "over" is the correct choice in this sentence because it indicates the
assumption of control or responsibility. When a new government "takes over," it means it
assumes authority and leadership, which is the intended meaning in this context.

The meanings of the other options are as follows:

● "After" does not convey the same sense of assuming control or responsibility;
it's used to indicate a sequence in time.
● "Upon" is not the appropriate word for describing the assumption of government
control; it's used more for formal or legal actions.
● "Out" is unrelated to the idea of a new government taking control.

Therefore, the correct sentence is, "The new government took over last year."

46-A

www.careers360.com Back to Index 247


The word "interesting" is the correct choice in this sentence because it conveys the
idea that Mohan's career has experienced various changes or developments that
capture one's attention or curiosity. It suggests that the twists and turns in his career are
intriguing or engaging.

The meanings of the other options are as follows:

● "Intuitive" refers to something that is based on instinct or gut feeling, which


doesn't appropriately describe career changes.
● "Interactive" refers to actions or processes involving communication or
interaction between people or systems, which is not relevant here.
● "Incentive" refers to motivation or stimulus to take action, which doesn't
describe the nature of the career changes.

Therefore, the correct sentence is, "Mohan's career has taken some interesting twists
and turns."

47-C

The preposition "over" is the correct choice in this sentence, as it helps to indicate that
Sheila gained an advantage in a way that surpasses or exceeds the advantage held by
the person being compared (in this case, the speaker).

● "From" implies receiving something, but it doesn't convey the idea of gaining an
advantage over someone.
● "On" is used to indicate a physical location or topic of discussion, and it doesn't
convey the intended meaning.
● "Upon" is a less common preposition for expressing this kind of relationship or
advantage and is not as suitable as "over" in this context.

Therefore, the correct sentence is, "Sheila gained an advantage over me."

48-D

The word "accomplished" is the correct choice in this sentence because it means
highly skilled or proficient in a particular area. When describing someone as
"accomplished in singing and dancing," it suggests that the person is very skilled and
capable in these arts.

The meanings of the other options are as follows:

● "Conductive" refers to the ability to conduct electricity or heat and is not


relevant to singing and dancing.

www.careers360.com Back to Index 248


● "Cooperative" means willing to work together with others, which is not the
intended meaning in this context.
● "Fluctuating" means changing or varying, which is not appropriate to describe
someone's skills in singing and dancing.

Therefore, the correct sentence is, "She was remarkably accomplished in singing and
dancing."

GENERAL KNOWLEDGE

49-C
The Indian Constitution Part IV contains Articles 36–51 that deal with Directive
Principles of State Policy (DPSP). They originated in the Spanish Constitution, and India
adopted them from the Irish Constitution. The DPSP must be taken into consideration
by the state when creating policies and passing legislation.

50-A
The central government has the authority to establish and manage investigative
agencies such as the Central Bureau of Investigation (CBI), but the details and
administration of such agency may not be specifically specified under the Seventh
Schedule. Because central agencies such as the CBI can investigate specific crimes
across states based on special regulations and agreements, they are considered to be
on the Union List. The Union List includes topics like as defence, foreign affairs, money,
atomic energy, and telecommunications.

51-A
Cnidaria, also known as Coelenterata, is a phylum of aquatic invertebrates that
includes a varied collection of organisms distinguished by their specialised stinging cells
known as cnidocytes. These animals have a body structure with radial symmetry and a
sac like or cylindrical form. Cnidarians live in both saltwater and freshwater habitats.
Corals, sea anemones, and sea pens are all members of the Anthozoa class. They are
frequently sessile (non-moving) and can form colonies, contributing significantly to coral
reef habitat and ecological support. Cnidocytes are used by cnidarians to acquire prey
and to defend themselves.

52-B
The communist party of India (CPI) apart from the congress, had representation in the
inaugural Lok Sabha. In 1952, A. K. Gopalan, an Indian communist, served as the head
of the opposition. With the enactment of the 1977 salary and allowances of leaders of
opposition in parliament act, the leader of the opposition role gained official recognition.
This position is held by the individual of the party with the highest number of opposition
seats or at least ten percent of all seats in the house.

www.careers360.com Back to Index 249


53-C
Article 93 to Article 96 of the Indian constitution deals with the powers and functions of
speaker and deputy speaker of Lok Sabha. As per the convention the speaker of Lok
Sabha is from the ruling party and the deputy speaker is from opposition. The speaker
of Lok Sabha is responsible for orderly functioning of the lower house, conducting
regular proceeding and maining law and order in the house. As per the constitution the
speaker of Lok Sabha can resign by addressing his resignation to the deputy speaker of
Lok Sabha.

54-D
The correct answer is Part IV A.

Part IV A of the Indian Constitution contains Article 51A, which addresses the
Fundamental Duties. The 42nd Constitutional Amendment Act of 1976 added the
fundamental duties to Part IV A of our constitution. Currently, article 51 A of the
Constitution lists eleven obligations. Fundamental duties were added to the Indian
Constitution on the recommendations of the Swaran Singh Committee.

55-C
The correct answer is Fundamental Rights.

When a right is justiciable, there is a legal basis for seeking a judicial or quasi-judicial
remedy when a violation has occurred. Fundamental Rights are justiciable because
anyone who believes their rights have been violated has the right to take legal action in
the High Court or Supreme Court. Any Fundamental Rights may be enforced by the
Supreme Court or High Courts issuing directives, orders, or writs.

56-D
The correct answer is the abolition of titles except military and academic.

11 fundamental obligations that are both statutory and legally enforceable under Article
51-A of our Constitution. Titles are eliminated in Article 18, which aims to lessen social
inequality in India. By abiding by the rules, such as the ban on Indian citizens obtaining
any titles from other countries, the abolition of the title is achieved. A few non-title
awards are exempt from Article 18, and its rules are given out by the country. An award
known as a "Bharat Ratna" is given to individuals who have made outstanding
contributions to any field.

www.careers360.com Back to Index 250


57-A

The correct answer is Sri Lanka.

Sri Lanka and India are separated by a small body of water known as the Palk Strait or
the Gulf of Mannar. Sri Lanka is cut off from India by the Gulf of Mannar and the Palk
Strait. The Jaffna peninsula in Sri Lanka and the Indian state of Tamil Nadu are
separated by the Palk Strait. It is named after Madras Governor Robert Palk and
measures 137 km in length with a minimum depth of 9.1 metres. The Pamban Island,
Adam's Bridge and the Gulf of Mannar form its southern and western boundaries,
respectively. The southwestern section of the strait is known as Palk Bay. Tamil Nadu's
Vaigai River empties into the Palk Strait.

58-C
The correct answer is 3.28.

India's land area is 3.28 million square kilometres. India is the seventh-largest country
on earth in terms of area and the largest one in terms of population. India has a total
land boundary of about 15,200 km, of which India shares its land boundary with
Bangladesh.

59-C

The correct option is Article 21

The right to privacy is a basic one that the Indian Supreme Court has regarded as
being implicit in the rights to life and personal liberty given by Article 21 of the
Constitution. This interpretation was established in the landmark 2017 decision of
Justice K. S. Puttaswamy (Retd.) vs. Union of India. The right to privacy is a
fundamental right that safeguards individuals from unwarranted intrusion into their
private lives, homes, families, and communication.

60-D
The correct option is Sri Lanka.

The Palk Strait separates Sri Lanka's Northern Province, particularly the Jaffna District,
from the Indian state of Tamil Nadu. It connects Palk Bay in the southwest to the Bay of
Bengal in the northeast. This strait is relatively shallow, measuring less than 330 feet in
depth, with a width ranging from 40 to 85 miles. Its total length extends for
approximately 85 miles.

www.careers360.com Back to Index 251


MOCK TEST - 8
______________________________________________________________________

LOGICAL REASONING
Q1. Direction: In the following question below, some statements are followed by
some conclusions. Taking the given statements to be true, even if they seem to
be at variance from commonly known facts, read all the conclusions and then
decide which of the given conclusions logically follows from the given
statements.
Statements:
1. All young girls are modern.
2. No modern men are superstitious.
Conclusions:
I. No girls are superstitious.
II. No young men are superstitious.

A) Conclusion I follows/
B) Conclusion II follows
C) Neither conclusion I nor conclusion II follows
D) Both conclusion I and conclusion II follows

Q2.Direction: In the following question, two statements are given followed by two
conclusions I and II. You have to consider the two statements to be true even if
they seem to be at variance from commonly known facts. You have to decide
which of the given conclusions, if any, follow from the given statements.

Statements:

No man is a donkey.

Rahul is a man.

Conclusions:

I. Rahul is not a donkey.

II. All men are not Rahul.

www.careers360.com Answer 252


A) Either conclusion I or conclusion II follow.
B) Neither Conclusion I nor II follow
C) Only conclusion I follow
D) Only conclusion II follow.

Q3.Assertion: The boiling point of a liquid decreases at higher altitudes.

Reason: At higher altitudes, the atmospheric pressure is lower, which lowers the
boiling point of liquids.

A) Both the assertion and reason are true, and the reason is a correct explanation of the
assertion.

B) Both the assertion and reason are true, but the reason is not a correct explanation of
the assertion.

C) The assertion is true, but the reason is false.

D) The assertion is false, but the reason is true.

Q4.Assertion: A prism can separate white light into its constituent colors through
dispersion.

Reason: Different colors of light travel at different speeds when passing through
a prism, causing them to spread apart.

A) Both the assertion and reason are true, and the reason is a correct explanation of the
assertion.

B) Both the assertion and reason are true, but the reason is not a correct explanation of
the assertion.

C) The assertion is true, but the reason is false.

D) The assertion is false, but the reason is true.

Q5.According to the given statements, Which of the following options is correct.

Statement 1:The chemical symbol for potassium is Po.

Statement 2:The chemical symbol for lead is Pb.

A) Statement 1 is correct, but Statement 2 is incorrect.

B) Statement 1 is incorrect.

www.careers360.com Answer 253


C) Statement 2 is incorrect.

D) Statement 2 is correct, but Statement 1 is incorrect.

Q6.According to the given statements, Which of the following options is correct

Statement 1: Sound travels faster in a vacuum than in air.

Statement 2: Sound does not travel in a vacuum.

A) Statement 1 is correct, but Statement 2 is incorrect.

B) Statement 1 is incorrect.

C) Statement 2 is incorrect.

D) Statement 2 is correct, but Statement 1 is incorrect.

Q7. Direction: In the following question, a statement is given, followed by two


arguments, I and II. You have to consider the statement to be true, even if it
seems to be at variance from commonly known facts. You have to decide which
of the given arguments, if any, follows from the given statement.

Statement:

"Should schools have mandatory drug testing for students?"

Arguments:

I. Yes, it can deter drug use among students and create a safer learning
environment.

II. No, it infringes on students' privacy and may not be effective in curbing drug
use.

A) Only Argument I follows.

B) Only Argument II follows.

C) Both I and II follows

D) Neither Argument I nor Argument II follows.

www.careers360.com Answer 254


Q8. Direction: In the following question, a statement is given, followed by two
arguments, I and II. You have to consider the statement to be true, even if it
seems to be at variance from commonly known facts. You have to decide which
of the given arguments, if any, follows from the given statement.

Statement:
"Should genetically modified organisms (GMOs) be used in agriculture?"
Arguments:
I. Yes, GMOs can increase crop yield and reduce the need for pesticides.
II. No, GMOs may have long-term health and environmental risks.

A) Only Argument I follows.


B) Only Argument II follows.
C) Both I and II follows
D) Neither Argument I nor Argument II follows.

Q9. Direction: In the following question, select the odd letter pair from the given
alternatives.

A) PU
B) SW
C) CG
D) KO

Q10. Direction: In the following question, select the odd letter cluster from the
given alternatives.

A) GJM
B) PSV
C) MPR
D) KNQ

Q11.Direction: Pointing to a photograph of a lady, Vishal says "She is the


sister-in-law of my grandfather's (paternal) daughter". How is the lady in the
photograph related to Vishal?

A) Mother
B) Aunt
C) Sister
D) Cannot be determined

www.careers360.com Answer 255


Q12. Direction: In a row, Sanjay is in 11th position from the top and 13th from the
bottom. How many people are there in the row?

A) 24
B) 23
C) 25
D) 26

LEGAL REASONING
Q13. Question: According to the Constitution of India, who has the power to
proclaim an emergency in case of a threat to the security of India?

A) President of India
B) Prime Minister of India
C) Chief Justice of India
D) Governor of the state

Q14. De jure means:

A) Relating to law
B) Relating to fact
C) Concerning law and fact
D) None of these

Q15. Question: Under which constitutional provision does the President of India
have the power to pardon or suspend a sentence of punishment?
A) Article 72
B) Article 32
C) Article 356
D) Article 44
Q16. Question: What constitutional amendment introduced the Goods and
Services Tax (GST) in India, replacing a complex system of indirect taxes?

A) 101st Amendment
B) 122nd Amendment
C) 73rd Amendment
D) 86th Amendment

www.careers360.com Answer 256


Q17. Question: Which international legal principle allows a state to take action in
self-defense against an armed attack?

A) Jus cogens
B) Universal jurisdiction
C) Collective security
D) Article 51 of the UN Charter

Q18.In the context of international humanitarian law, what treaty regulates the
treatment of prisoners of war during armed conflicts?

A) Geneva Conventions
B) Rome Statute
C) Hague Convention
D) Vienna Convention

Q19.Under the principle of diplomatic immunity, which of the following actions is


protected for diplomats?

A) Criminal activities
B) Violation of traffic laws
C) Espionage
D) None of the above

Q20/Which international court has jurisdiction over cases involving alleged


violations of human rights committed by states or individuals?

A) International Court of Justice (ICJ)


B) International Criminal Court (ICC)
C) European Court of Human Rights
D) Permanent Court of Arbitration

Q21.In a hypothetical country, a child is born to non-citizen parents within its


territory. What principle determines the child's citizenship status?

A) Jus sanguinis
B) Jus soli
C) Dual citizenship
D) Naturalization

www.careers360.com Answer 257


Q22.In some countries, citizenship can be acquired by residing in the country for
a specified period, demonstrating good conduct, and fulfilling other
requirements. What is this process called?

A) Jus sanguinis
B) Jus soli
C) Denaturalization
D) Naturalization

Q23.What is the term for the situation where an individual holds citizenship in
more than one country?

A) Stateless
B) Expatriate
C) Dual citizenship
D) Denaturalization

Q24.In a certain country, citizenship is automatically lost if a person voluntarily


acquires citizenship in another country. What concept does this represent?

A) Renunciation
B) Expatriation
C) Denaturalization
D) Loss by dual citizenship

ANALYTICAL REASONING

Q25.A man completes a journey in 10 hours. He travels the first half of the
journey at the rate of 21 km/hr and second half at the rate of 24 km/hr. Find the
total journey in km.

A] 220 km

B] 224 km

C] 200 km

D] 240 km

www.careers360.com Answer 258


Q26.The ratio between the speeds of two trains is 7 : 8. If the second train runs
400 km in 4 hours, then the speed of the first train is:

A] 12.5 km/hr

B] 27 km/hr

C] 20 km/hr

D] 87.5 km/hr

Q27. A train is running at a speed of 116 km/h. The distance covered by the train
in meters in 18 seconds is:

A] 580 metres
B] 480 metres
C] 280 metres
D] 380 metres

Q28. P, Q and R can do a work in 20, 30 and 60 days respectively. How many days
does it need to complete the work if P does the work and he is assisted by Q and
R on every third day?

A] 15 days
B] 10 days
C] 25 days
D] 45 days

Q29.A large tanker can be filled by two pipes A and B in 60 minutes and 40
minutes respectively. How many minutes will it take to fill the tanker from an
empty state if B is used for half the time and A and B fill it together for the other
half?

A] 15 min
B] 20 min
C] 30 min
D] 40 min

www.careers360.com Answer 259


Q30. In what ratio should water be mixed with soda costing Rs. 12 per litre so as
to make a profit of 25% be selling the diluted liquid at Rs. 13.75 per litre?

A] 10 : 1
B] 11 : 1
C] 1 : 11
D] 12 : 1

Q31. A quadrilateral is a parallelogram if


A] both pair of opposite angles are equal
B] the diagonals bisect each other
C] both pairs of opposite sides are equal
D] All of the above

Q32.The internal and external diameters of a hollow hemispherical vessel are 20


cm and 28 cm respectively. Find the cost of painting the vessel all over at 35
paise per cm2.

A] Rs 756.80
B] Rs 766.80
C] Rs 750.80
D] Rs 740.80

Q33 The surface areas of two spheres are in the ratio 25 : 16. The ratio of their
volumes is

A]

B]
C] 120/7
D] 135/11

Q34.A, B and C play cricket. A's runs are to B's runs and B's runs are to C's as
3:2. They get altogether 342 runs. How many runs did A make?
A] 162
B] 108

www.careers360.com Answer 260


C] 72
D] None of these
Q35.Directions: Which of the following diagrams represents the relationship
between Vegetables, Onion, and Eatables?

A)

B)

C)

D)

Q36.Directions: Among four books, Book 1 is twice as heavy as Book 2. Book 3's
weight is half of Book 2's weight. Book 4 is 60 grams more as compared to Book
2, but 60 grams less as compared to Book 1. Which book is the heaviest?

A) Book 1

www.careers360.com Answer 261


B) Book 2
C) Book 3
D) Book 4

READING COMPREHENSION

Q37. Select the option that expresses the given sentence in active voice.

By whom was the coffee made?

A) Who made the coffee?

B) Who had made the coffee?

C) Who has made the coffee?

D) Who makes the coffee?

Q38. Select the most appropriate ANTONYM of the underlined words in the given
sentence.
Developed countries are at a disadvantage to have access to modern science and
technologies.

A) powered

B) privileged

C) useless

D) unhappy

Q39. In the following sentence, four words are underlined, out of which one word
is incorrectly spelt. Identify the INCORRECTLY spelt word.
Seema was a timid girl. She did not talk to people who were not familiar to her.
She was an introvert since adolosense.

A) introvert

B)timid

C) adolosense

D) familiar

www.careers360.com Answer 262


Q40.Sentences of a paragraph are given below in jumbled order.Arrange the
sentences in the correct order to form a meaningful and coherent paragraph.
A.According to Homer, Achilles was brought up by his mother at Phthia with his
inseparable companion Patroclus.
B.Achilles was the bravest, handsomest and greatest warrior of the army of
Agamemnon in the Trojan War.
C.Later non-Homeric tales suggest that Patroclus was Achilles' kinsman or lover.
D.Achilles, in Greek mythology, is the son of the mortal Peleus, king of the
Myrmidons, and the Nereid Thetis.

A) ABCD

B) DBAC

C) DABC

D) CABD

Q41.Select the most appropriate idiom/phrase that can substitute the underlined
segment in the given sentence.
Vennela made a big fuss about a small problem.

A) Get a taste of your own medicine

B) Barking up the wrong tree

C) Add insult to injury

D) A storm in a teacup

Q42.Describe how you will tell your parents that Mahesh and his team were
helping the fire and rescue team in a passive voice.

A) The fire and rescue team helped by Mahesh and his team.

B) The fire and rescue team was being helped by Mahesh and his team.

C) The fire and rescue team being help by Mahesh and his team.

D) The fire and rescue team being help by Mahesh and his team.

Q43. Select the grammatically correct statement from among the given options.
A. It is a nice party. I am enjoying the party.
B. It is a party. I am enjoying an party.

www.careers360.com Answer 263


C. It is party. I am enjoying a party.
D. It is the party. I am enjoying an party.

A) D

B) C

C) B

D) A

Q44. Select the option that can be used as a one-word substitute for the given
group of words.
A book or set of books giving information about all areas of knowledge

A) Encyclopedia

B) Dictionary

C) Volume

D) Anthology

Q45.Select the most appropriate synonym of the given word.


Peruse

A) Check

B) Draw

C) Pursue

D) Narrate

Q46.Sentences of a paragraph are given below in jumbled order. Arrange the


sentences in the correct order to form a meaningful and coherent paragraph.
A. We often make all things around us the way we want them.
B. Even during our pilgrimages, we have begun to look for whatever makes our
heart happy, gives comfort to our body and peace to the mind.
C. Our mind is resourceful enough to find shortcuts in simple and easy ways.
D. It is as if external solutions will fulfil our needs, and we do not want to make
any special efforts even in our spiritual search.

A) CABD

www.careers360.com Answer 264


B) ADBC

C) ABDC

D) DABC

Q47. Select the most appropriate ANTONYM of the underlined word in the given
sentence.

The Emperor was considered a wicked man.

A) Virtuous

B) Savage

C) Fierce

D) Vile

Q48. Sentences of a paragraph are given below in jumbled order. Arrange the
sentences correctly to form a meaningful and coherent paragraph.
A. It's because the 'practical people' in society boast of having a practical
approach to life.
B. It is very likely for youngsters to seek advice from their elders.
C. Emphasising upon the times that prevailed, the youth very much get their
queries answered systematically.
D. In a similar context even the elders are, in most likelihood, expected to give
their advice as a word of guidance.

A) BDCA

B) CDAB

C) ACDB

D) DCAB

GENERAL KNOWLEDGE

Q49.Which Constitutional Amendment Act deals with the disqualification of MPs


and MLAs?

A) 42nd Amendment Act

www.careers360.com Answer 265


B) 52nd Amendment Act

C) 62nd Amendment Act

D) 32nd Amendment Act

Q50.Which of the following schedules contains special provisions for the


administration of tribal areas in the four northeastern states of Assam,
Meghalaya, Tripura, and Mizoram?

A) First Schedule

B) Second Schedule

C) Third Schedule

D) Sixth Schedule

Q51. The largest irrigation canal in India is called the

A) Yamuna Canal

B) Sirhind Canal

C) Indira Gandhi Canal

D) Upper Bari Doab Canal

Q52. The plateau that has both West and East-flowing drainage systems is

A) Malwa

B) Chota Nagpur

C) Ranchi

D) Hazaribagh

Q53. A tropical deciduous plant special to the Deccan Plateau is

A) teak

B) shisham

C) sandalwood

www.careers360.com Answer 266


D) sal

Q54. The Himalayas are also known by the name:

A) ancient mountains

B) fold mountains

C) valley mountains

D) Indus mountains

Q55. Which river originates from the Amarkantak plateau?

A) Narmada river

B) Son river

C) Betwa river

D) Godavari river

Q56.The outermost range of the Himalayas is called

A) Kali

B) Shivalik

C) Dehradun

D) Kumaon

Q57. A continuous chain of mountains that rise abruptly, more or less parallel to
the coastline of India is:

A) Aravalli

B) Satpura

C) Eastern Ghats

D) Western Ghats

Q58. India shares the longest international border with which country?

A) Bangladesh

www.careers360.com Answer 267


B) China

C) Nepal

D) Bhutan

Q59.The term "LAF" in the context of RBI stands for:

A) Lender Adjustment Facility


B) Liquidity Adjustment Facility
C) Loan Allocation Framework
D) Liability Assessment Fund

Q60. What is the term for a large, bowl-shaped depression in the Earth's surface,
typically caused by the collapse of a volcano?

A) Plateau
B) Caldera
C) Abyssal plain
D) Cuesta

www.careers360.com Answer 268


ANSWER KEY
______________________________________________________________________

1 2 3 4 5 6 7 8 9 10
C C A A D C B A A C
11 12 13 14 15 16 17 18 19 20
D B A A A B D A D B
21 22 23 24 25 26 27 28 29 30
B D C B B D A A C C
31 32 33 34 35 36 37 38 39 40
A A A A B A A B C B
41 42 43 44 45 46 47 48 49 50
D B D A A A A A B D
51 52 53 54 55 56 57 58 59 60
C A C B A B D A B B

www.careers360.com Solutions 269


SOLUTIONS
______________________________________________________________________

LOGICAL REASONING
1-C
Given:
Statements:
1. All young girls are modern.
2. No modern men are superstitious.
The venn diagram is:

Let's analyze the conclusions


Conclusion (I): No girls are superstitious.
The statements only mention young girls. None of the statements contain any
information about girls. So, This conclusion does not follow.
Conclusion (II): No young men are superstitious.
The statements only mention modern men. None of the statements contain any
information about young men. So, This conclusion does not follow.

So, neither Conclusion I nor Conclusion II follows. Hence, the third option is correct.

2-C
Given:
Statements:
No man is a donkey.
Rahul is a man.
The Venn diagram is:

www.careers360.com Back to Index 270


Let's analyze the conclusions–
Conclusion (I): Rahul is not a donkey.
From the Venn diagram, we can see that the circles that represent Rahul and the
donkey do not overlap. So, Rahul is not a donkey. This conclusion follows.
Conclusion (II): All men are not Rahul.
This conclusion does not follow because there is a possibility that Rahul is the name of
all the men.

So, the only conclusion I follow. Hence, the third option is correct.

3-A
The boiling point of a liquid is influenced by atmospheric pressure. At higher altitudes,
where atmospheric pressure is lower, liquids boil at lower temperatures.

4-A
A prism can separate white light into its constituent colors through dispersion because
different colors of light refract by different amounts when passing through the prism due
to their different wavelengths.

5-D
The chemical symbol for potassium is K, and the chemical symbol for lead is Pb.

6-C
Sound does not travel in a vacuum, so Statement 2 is correct. However, Statement 1 is
incorrect; sound does not travel in a vacuum at all.

7-B
Argument I supports mandatory drug testing for its potential deterrent effect, while
Argument II raises privacy and effectiveness concerns, indicating reasons against it.

8-A
Argument I supports the use of GMOs for their potential benefits in increasing crop

www.careers360.com Back to Index 271


yield and reducing pesticide usage. Argument II raises concerns about long-term health
and environmental risks but does not negate the benefits mentioned in Argument I.

9-A
Let's check the options —

First option: PU; P + 5 = U (The first option is different from the other three)

Second option: SW; S + 4 = W

Third option: CG; C + 4 = G

Fourth option: KO; K + 4 = O

Hence, the first option is correct.

10-C
Let's check the options —

First option: GJM; G + 3 = J; J + 3 = M

Second option: PSV; P + 3 = S; S + 3 = V

Third option: MPR; M + 3 = P; P + 2 = R (The third option is different from the other
three)

Fourth option: KNQ; K + 3 = N; N + 3 = Q

Hence, the third option is correct.

www.careers360.com Back to Index 272


11-D
The family tree is as follows –

In the above diagram + denotes male and – denotes female.


The above family tree shows two possibilities where in one, the lady can be the mother
of Vishal, and in other, the lady can be the Aunt of Vishal.

So, from the above, the exact relationship between the lady and Vishal cannot be
determined. Hence, the fourth option is correct.

12-B
Given:

Sanjay is 11th from the top and 13th from the bottom in a row.

So, the total number of members in that row:

(11 + 13) – 1 = 24 – 1 = 23

Hence, the second option is correct.

LEGAL REASONING

13-A
The President of India has the authority to proclaim a state of emergency under Article
352 of the Indian Constitution in case of a threat to the security of India.

14-A
De jure means Relating to law. Hence option a is correct.

www.careers360.com Back to Index 273


15-A
Article 72 of the Indian Constitution grants the President the power to grant pardons,
reprieves, respites, or remissions of punishment or to suspend, remit, or commute the
sentence.

16-B
The Goods and Services Tax (GST) was introduced in India through the 122nd
Amendment to the Constitution, which came into effect in 2017.
Certainly, here are four challenging multiple-choice questions based on international law
along with answers and explanations:

17-D
Article 51 of the UN Charter recognizes the inherent right of individual or collective
self-defense in the case of armed attacks.

18-A
The Geneva Conventions establish the legal framework for the treatment of prisoners of
war and civilians during armed conflicts.

19-D
While diplomats enjoy immunity for their official acts, diplomatic immunity does not
protect criminal activities, espionage, or gross violations of laws.

20-B
The International Criminal Court (ICC) has jurisdiction over cases involving crimes such
as genocide, war crimes, and crimes against humanity.

21-B
Jus soli is the principle that grants citizenship based on the location of birth, irrespective
of the citizenship status of the parents.

22-D
Naturalization is the legal process by which a person not born in a country can become
a citizen through fulfilling certain criteria.

23-C Dual citizenship refers to the status of an individual who is a citizen of two or more
countries simultaneously.

24-B Expatriation occurs when an individual voluntarily acquires citizenship in another


country, leading to the automatic loss of citizenship in their original country.

www.careers360.com Back to Index 274


ANALYTICAL REASONING

25-B

26-D
Let the speed of two trains be

27-A

28-A
Amount of work P can do in 1 day = 1/20

Amount of work Q can do in 1 day = 1/30

Amount of work R can do in 1 day = 1/60

P is working alone and every third day Q and R is helping him

Work completed in every three days = 2 × (1/20) + (1/20 + 1/30 + 1/60) = 1/5

www.careers360.com Back to Index 275


So work completed in 15 days = 5 × 1/5 = 1

hence, the work will be done in 15 days

29-C

Part filled by (A+B) in 1 minute


Suppose the tank is filled in minutes.

Then,

30-C
S.P = 13.75
P = 25 %
S.P = C.P x 125/100
13.75 = C.P x 125/100
C.P = 11 Rs.

12 0

11 = 11:1

11 1

W: S = 1:11

www.careers360.com Back to Index 276


31-A
Types of Quadrilaterals - (Part 1) -
Parallelogram : A quadrilateral in which both pairs of opposite sides are parallel is called
a parallelogram.
A summary of the properties of a parallelogram is:
Both pairs of opposite sides are parallel (AB || CD and AD || BC).
Both pairs of opposite sides are equal in length (AB = CD and AD = BC).
Both pairs of opposite angles are equal.
Both diagonals bisect each other (i.e. they cut each other in half).

32-A

Outer radius of the vessel, R = 14 cm.

Inner radius of the vessel, r = 10 cm.

www.careers360.com Back to Index 277


33-A

34-A
A:B = 3:2 = 9:6
B : C = 3 : 2 = 6 : 4 ( making B equal)
So, A : B : C = 9 : 6 : 4
the runs made by A = (9/19) * 342 = 162

35-B
We know that all onions are vegetables. This means its circle will lie inside that of
vegetables. Also, we can eat both onions and vegetables. So, they are eatables and their
circle will lie inside that of eatables.

The correct Venn diagram is –

Hence, the second option is correct.

www.careers360.com Back to Index 278


36-A
Let W1 be the weight of Book 1, W2​be the weight of Book 2, W3 be the weight of Book 3
and W4 be the weight of Book 4.

Book 1 is twice as heavy as Book 2. So, W1 > W2.


Book 3's weight is half of Book 2's weight. So, W2 > W3.
Book 4 is 60 grams more than Book 2, but 60 grams less than Book 1. So, W1 > W4 >
W2.
Combining all these, we get W1 > W4 > W2 > W3.

Book 1 is the heaviest of all. Hence, the first option is correct.

READING COMPREHENSION

37-A
The sentence in active voice is:

"Who made the coffee?"

Explanation: The original sentence is in the passive voice, with the subject (the coffee)
coming after the action (was made).Passive voice is the voice in which something is
being done upon the object whereas in the active voice, the subject is the doer.Here,
(who) is the subject and it performed the action (made).

38-B
The most appropriate antonym of the underlined words "at a disadvantage" is
"privileged."

Explanation: "At a disadvantage" means being in an unfavourable or less


advantageous position. "Privileged" is the opposite, indicating that someone or
something has special advantages or benefits.

Correct sentence: "Developed countries are privileged to have access to modern


science and technologies."

39-C
The incorrectly spelt word in the sentence is "adolosense." The correct spelling should
be "adolescence."

Explanation: Adolescence is the period of life between childhood and adulthood,


typically occurring during the teenage years. It is characterised by significant physical,
psychological, and emotional changes.

www.careers360.com Back to Index 279


The meaning of other options are as follows:

● Introvert: An introvert is a person who is more focused on his own thoughts and
feelings rather than seeking external stimulation.
● Timid: "Timid" means lacking in self-confidence, shy, or easily frightened.
● Familiar: "Familiar" means well-known or acquainted with something or
someone.

40-B
The correct order is DBAC.

Explanation:

1. Sentence D introduces Achilles and provides information about his parentage.


2. Sentence B describes Achilles' qualities and his role in the Trojan War.
3. Sentence A follows logically, mentioning Achilles' upbringing and his companion
Patroclus.
4. Sentence C introduces additional information about the relationship between
Achilles and Patroclus.

So, the correct sentence would be: "Achilles, in Greek mythology, is the son of the
mortal Peleus, king of the Myrmidons, and the Nereid Thetis. Achilles was the bravest,
handsomest, and greatest warrior of the army of Agamemnon in the Trojan War.
According to Homer, Achilles was brought up by his mother at Phthia with his
inseparable companion Patroclus. Later non-Homeric tales suggest that Patroclus was
Achilles' kinsman or lover."

41-D
The most appropriate idiom that can substitute the underlined segment in the given
sentence is "A storm in a teacup."

Explanation: This idiom means making a great deal of fuss or commotion over a trivial
or minor issue. It is the most suitable choice because it directly conveys the idea of
creating a fuss over a small problem, which is the context of the sentence.

The meaning of other options are as follows:

● Get a taste of your own medicine: This idiom means to experience the same
negative treatment or situation that one has previously inflicted on others.
● Barking up the wrong tree: This idiom means to pursue a mistaken or
misguided course of action.
● Add insult to injury: This idiom means to make a bad situation worse.

www.careers360.com Back to Index 280


42-B
The correct passive voice sentence to describe is: "The fire and rescue team was
being helped by Mahesh and his team."

Explanation: In passive voice construction, the subject (the fire and rescue team in this
case) receives the action, and the agent (Mahesh and his team) is indicated using "by."

The other options contain grammatical errors and are not in proper passive voice form.

43-D
The grammatically correct statement is A. It is a nice party. I am enjoying the party.

Explanation: The reason for this choice is that it uses proper articles and prepositions.
"A" is the indefinite article used before "nice party," and "the" is the definite article
used before "party."

The use of "an" in options B and D is incorrect because it should be "a" before "party"
because "party" begins with a consonant sound.

Option C is also incorrect because it lacks the article "a" before "party."

44-A
The one-word substitute for the given group of words is Encyclopedia.

Explanation: An encyclopedia is a comprehensive reference work that contains


information on a wide range of topics or knowledge areas. It consists of articles, often
arranged alphabetically, covering various subjects.

● Dictionary: A dictionary is a reference book that provides definitions, meanings,


pronunciations, and often usage examples of words in a particular language.
● Volume: A volume is a book or part of a book that is part of a larger work, such
as a series or collection
● Anthology: An anthology is a collection of selected writings or literary works,
often by different authors, on a specific theme, genre, or subject.

45-A
The most appropriate synonym for the word "Peruse" is Check.

Explanation: To check means to inspect or examine something to ensure it is correct,


in order, or up to standard. While it may not always involve reading as closely as
"peruse," in some contexts, it can refer to examining written or printed material.

www.careers360.com Back to Index 281


Pursue: To pursue means to follow or chase something or someone, often to achieve a
particular objective.

The meaning of other options are as follows:

● Draw: To draw means to create a picture, image, or representation of something


using a pen, pencil, or other artistic medium.
● Narrate: To narrate means to tell or recount a story or a series of events, often in
a spoken or written form.

46-A

The correct choice is the First option: "CABD"

C. Our mind is resourceful enough to find shortcuts in simple and easy ways.
A. We often make all things around us the way we want them.

B. Even during our pilgrimages, we have begun to look for whatever makes our heart
happy, gives comfort to our body, and peace to the mind.

D. It is as if external solutions will fulfil our needs, and we do not want to make any
special efforts even in our spiritual search.

The reason for this order is that (Sentence C) introduces the idea that the mind looks for
shortcuts and easy solutions. (Sentence A) expands on the notion that we shape our
environment according to our desires and needs, reinforcing the idea of seeking comfort
and ease. (Sentence B) introduces the concept of seeking happiness, comfort, and
peace even during pilgrimages, which is an example of the behaviour discussed in the
previous sentences. Finally (Sentence D) concludes the paragraph by stating that
people often seek external solutions and avoid making special efforts, even in spiritual
matters, which ties back to the theme of seeking comfort and shortcuts introduced
earlier in the paragraph.

47-A
The most appropriate antonym of the underlined word is "Virtuous."

Explanation: Virtuous means having high moral standards, being good, righteous, and
behaving in a morally upright and ethical way. It is the opposite of "wicked," which
implies morally evil or corrupt.

The meaning of other options are as follows:

● Savage: Savage refers to something or someone brutal, cruel, or uncivilized.

www.careers360.com Back to Index 282


● Fierce: Fierce means showing a ferocious or aggressive nature, often in the
context of strength or intensity.
● Vile: Vile means extremely unpleasant, morally reprehensible, or wicked.

48-A

The correct choice is the First option: 'BDCA'

B. It is very likely for youngsters to seek advice from their elders.

D. In a similar context even the elders are, in most likelihood, expected to give their
advice as a word of guidance.

C. Emphasising upon the times that prevailed, the youth very much get their queries
answered systematically.

A. It's because the 'practical people' in society boast of having a practical approach to
life.

This is the correct order because (Sentence B) introduces the idea that young people
often seek advice from their elders, setting the context for the paragraph. (Sentence D)
explains that elders are also expected to provide advice in similar situations,
establishing a reciprocal relationship. (Sentence C) provides additional information
about the nature of the interaction, emphasising that young people receive systematic
answers to their questions. Finally (Sentence A) concludes the paragraph by introducing
the concept of "practical people" and how they perceive their approach to life. It ties
back to the initial idea of seeking advice and guidance from elders.

GENERAL KNOWLEDGE

49-B
The correct option is the 52nd Amendment Act.

The Tenth Schedule of the Indian Constitution, also referred to as the "Anti-Defection
Law", primarily governs the disqualification of Members of Parliament (MPs) and
Members of Legislative Assemblies (MLAs) in India. The Fifty-second Amendment Act
of 1985 added the Tenth Schedule to the Constitution. This law states the conditions
under which an MP or MLA may be removed from office, the processes and the
decision-makers in charge of such actions.

www.careers360.com Back to Index 283


50-D
The correct answer is the Sixth Schedule

In the four northeastern states of Assam, Meghalaya, Tripura, and Mizoram, there are
particular rules for the governance of tribal territories included in the sixth schedule.
Article dealing with it are 244(2) and 275(1). This schedule and these articles deal with
autonomous districts, district councils, and regional councils.

51-C
The correct answer is the Indira Gandhi Canal.

The Indira Gandhi Canal is the longest canal system in India. It starts at Harike Barrage
and flows parallel to the India-Pakistan border. The primary aim of the project was to
provide irrigation facilities in Rajasthan. Harike Barrage is formed at the confluence of
the rivers Sutlej and Beas in the state of Punjab, and the Indira Gandhi Canal passes
through the states of Punjab, Rajasthan and Haryana.

52-A
The correct answer is Malwa

The Malwa Plateau geologically refers to the volcanic highland north of the Vindhya
Range. It is connected to both the west and east drainage systems. The Mahi and
Chambal Rivers drain the plateau's centre, while the Betwa River and the headwaters of
the Dhasan and Ken Rivers drain the plateau's east.

53-C
The correct answer is sandalwood.

Tropical deciduous woods blanket the Deccan Plateau, and common plants found there
include sal, teak, shisham, and other plants. The unique tropical deciduous shrub
known as sandalwood is native to the Deccan Plateau, particularly, the districts of
Mysore in Karnataka and Coimbatore in Tamil Nadu.

54-B
The correct answer is fold mountains.

The Himalayas are known as "young fold mountains" because they developed only a
few million years ago, as a result of continental drift, where the Indo-Australian and
Eurasian plates collide.
Evidence that the Himalayas are young-fold mountains includes the deep gorges,
V-shaped valleys, U-turn river courses, parallel mountain ranges, steep slopes and
more.

www.careers360.com Back to Index 284


55-A
The correct answer is Narmada River

The Narmada River originates from the Amarkantak Plateau in Madhya Pradesh.

Other options:

● The Betwa River originates from the Vindhya range, near Narmadapuram in
Madhya Pradesh.
● The Godavari originates near Nashik in Maharashtra, in the Western Ghats of
central India.
● River Son originates from the Amarkantak Hills in Chattisgarh.

56-B
The correct answer is Sivalik

The Shivalik Himalaya, also known as the Shivalik Hills or Outer Himalayas, is the
southernmost and outermost range of the Himalayan mountain system. These hills are
lower in elevation, compared to the Greater Himalayas to the north. They are composed
of sedimentary rocks, including sandstone and shale. The Shivalik Hills were formed as
a result of the tectonic collision between the Indian and Eurasian plates. Its elevations
range from 600 metres (2,000 feet) to 1,500 metres (4,900 feet) above sea level.

57-D
The correct answer is the Western Ghats.The Western Ghats are a continuous chain
of mountains along the Arabian Sea coastline. It is also known as the Sahyadri
Mountain Range and traverses across the states of Gujarat, Maharashtra, Goa,
Karnataka, Kerala, and Tamil Nadu.

58-A
The correct answer is Bangladesh.India's land border is about 15,200 km. The
neighbouring countries of India, in decreasing order of border length, are Bangladesh >
China > Pakistan > Nepal > Myanmar > Bhutan >Afghanistan. India's neighbouring
island countries are Sri Lanka and the Maldives.

59-B
Liquidity Adjustment Facility. LAF is a framework used by the RBI to manage short-term
liquidity in the financial system.

60-B
A caldera is a large, bowl-shaped depression that forms when a volcano collapses after
a massive eruption or when a magma chamber is emptied.

www.careers360.com Back to Index 285


MOCK TEST - 9
____________________________________________________________________________

LOGICAL REASONING
Q1.Direction: In the following question, two statements are given, followed by two
conclusions, I and II. You have to consider the statements to be true even if they
are seen to be at variance from commonly known facts. You have to decide which
of the given conclusions, if any, follows from the given statements.
Statements:
1. All dawns are day.
2. No day is night.
Conclusions:
I. No night is day.
II. Some dawns are night.

A) Only conclusion I follows


B) Only conclusion II follows
C) Both I and II follow
D) Neither I nor II follows

Q2. Direction: In the following question two statements are given, followed by two
conclusions I and II. You have to consider the statements to be true even if they
are seen to be at variance from commonly known facts. You have to decide which
of the given conclusions, if any follows from the given statements.

Statements:

I. All flowers are leaves.

II. Some flowers are plants.

Conclusions:

I. Some leaves are plants.

II. Some plants are flowers.

A) Only conclusion I follows


B) Only conclusion II follows

www.careers360.com Answer 286


C) Both I and II follow
D) Neither I nor II follows

Q3. Assertion:The process of nuclear fusion powers the Sun and other stars.

Reason:Nuclear fusion occurs when atomic nuclei combine, releasing a


tremendous amount of energy.

A) Both the assertion and reason are true, and the reason is a correct explanation of the
assertion.

B) Both the assertion and reason are true, but the reason is not a correct explanation of
the assertion.

C) The assertion is true, but the reason is false.

D) The assertion is false, but the reason is true.

Q4.Assertion:The primary function of red blood cells is to transport oxygen to


body tissues.

Reason:Red blood cells contain hemoglobin, a molecule that binds to oxygen and
carries it in the bloodstream.

A) Both the assertion and reason are true, and the reason is a correct explanation of the
assertion.

B) Both the assertion and reason are true, but the reason is not a correct explanation of
the assertion.

C) The assertion is true, but the reason is false.

D) The assertion is false, but the reason is true.

Q5.According to the given statements, Which of the following options is correct.

Statement 1:The Amazon River is the longest river in the world.

Statement 2:The Nile River is the second-longest river in the world.

A) Statement 1 is correct, but Statement 2 is incorrect.

B) Statement 2 is incorrect.

C) Statement 1 is incorrect.

www.careers360.com Answer 287


D) Statement 2 is correct, but Statement 1 is also correct.

Q6.According to the given statements, Which of the following options is correct.

Statement 1:Neon is a noble gas.

Statement 2: Helium is a halogen.

A) Statement 1 is correct, but Statement 2 is incorrect.

B) Statement 2 is incorrect.

C) Statement 1 is incorrect.

D) Statement 2 is correct, but Statement 1 is also correct.

Q7. Direction: In the following question, a statement is given, followed by two


arguments, I and II. You have to consider the statement to be true, even if it
seems to be at variance from commonly known facts. You have to decide which
of the given arguments, if any, follows from the given statement.

Statement:

"Should the death penalty be abolished?"

Arguments:

I. Yes, it is inhumane and irreversible, leading to the possibility of executing


innocent individuals.

II. No, the death penalty serves as a strong deterrent against serious crimes.

A) Only Argument I follows.

B) Only Argument II follows.

C) Both I and II follows

Q8. Direction: In the following question, a statement is given, followed by two


arguments, I and II. You have to consider the statement to be true, even if it
seems to be at variance from commonly known facts. You have to decide which
of the given arguments, if any, follows from the given statement.

Statement:

www.careers360.com Answer 288


"Should the voting age be lowered to 16?"

Arguments:

I. Yes, 16-year-olds are affected by political decisions and should have a say in
the democratic process.

II. No, 16-year-olds may not have the maturity and understanding to make
informed political decisions.

A) Only Argument I follows.

B) Only Argument II follows.

C) Both I and II follows

D) Neither Argument I nor Argument II follows.

Q9. Direction: In the following question, four number pairs are given. The number
on the left side of (–) is related to the number on the right side of (–) with some
logic, rule, or relationship. Three are similar on the basis of the same logic, rule,
or relationship. Select the odd one out of the given alternatives.

A) 20 – 30
B) 35 – 45
C) 45 – 55
D) 25 – 30

Q10. Direction: In the following question, four number pairs are given. The
number on the left side of (–) is related to the number on the right side of (–) with
some logic, rule, or relationship. Three are similar on the basis of the same logic,
rule, or relationship. Select the odd one out of the given alternatives.

A) (1, 11, 1111)


B) (2, 22, 4444)
C) (4, 44, 4444)
D) (8, 88, 8888)

Q11. Direction: Introducing a girl, Raju says, "She is the daughter of my


grandfather's son's daughter". How is the girl related to Raju?

A) Cousin

www.careers360.com Answer 289


B) Wife
C) Sister
D) Niece

Q12.Direction: The weight of P is twice that of Q. The weight of Q is half that of R.


The weight of R is 3 times of T. The weight of T is half that of S. The weight of T is
greater than the weight of how many people among P, Q, R, and S?

A) 1
B) 0
C) 3
D) 4

LEGAL REASONING
Q13. Question: What is the highest court in India?

A) Supreme Court
B) High Court
C) District Court
D) Session Court
Q14. Question: Who is known as the "Father of the Indian Constitution"?

A) Jawaharlal Nehru
B) B.R. Ambedkar
C) Sardar Patel
D) Mahatma Gandhi

15. Question: What does the term "Habeas Corpus" mean?

A) Right to Equality
B) Right to Life and Personal Liberty
C) Right to Freedom of Religion
D) Right against Exploitation

Q16. Question: Which fundamental right guarantees protection against arrest and
detention in certain cases?

A) Right to Freedom
B) Right to Equality

www.careers360.com Answer 290


C) Right to Life and Personal Liberty
D) Right to Education

Q17 . Question:What is the minimum voting age in India?

A) 18 years
B) 21 years
C) 25 years
D) 16 years

Q18. Question: What is the term for intentional false communication that harms a
person's reputation?

A) Libel
B) Slander
C) Defamation
D) Invasion of Privacy

Q19.Which law regulates contracts in India?

A) Indian Penal Code


B) Indian Contract Act
C) Companies Act
D) Consumer Protection Act
Q20.What is the punishment for the offense of theft under the Indian Penal Code?

A) Fine
B) Imprisonment
C) Both A and B
D) Community Service

Q21.Which constitutional amendment introduced the Goods and Services Tax


(GST) in India?

A) 101st Amendment
B) 122nd Amendment
C) 91st Amendment
D) 75th Amendment

www.careers360.com Answer 291


Q22.What is the term for a written order issued by a higher court to a lower court
or tribunal to enforce a right or to correct an error?

A) Subpoena
B) Mandamus
C) Injunction
D) Writ of Certiorari

Q23.Which international organization is responsible for the peaceful settlement of


disputes between countries?

A) United Nations
B) International Monetary Fund
C) World Health Organization
D) World Trade Organization

Q24.What does "Caveat Emptor" mean in contract law?

A) Let the seller beware


B) Let the buyer beware
C) Buyer's remorse
D) Seller's regret

ANALYTICAL REASONING

Q25. A man on tour travels first 160 km at 64 km/hr and the next 160 km at 80
km/hr. The average speed for the first 320 km of the tour is:

A] 71.11 km/hr

B] 35 km/hr

C] 30.55 km/hr

D] 32.50 km/hr

Q26. A car travelling with of its actual speed covers 42 km in 1 hr 40 min 48


sec. Find the actual speed of the car.

A] 35 km/hr

B] 40 km/hr

www.careers360.com Answer 292


C] 50 km/hr

D] 25 km /hr

Q27. A boat can travel with a speed of 13 km/hr in still water. If the speed of the
stream is 4 km/hr, find the time taken by the boat to go 68 km downstream.

A] 4 hours
B] 5 hours
C] 3 hours
D] 6 hours

Q28. A alone can do a piece of work in 6 days and B alone in 8 days. A and B
undertook to do it for Rs. 3200. With the help of C, they completed the work in 3
days. How much is to be paid to C?

A] Rs. 375
B] Rs. 400
C] Rs. 600
D] Rs. 800

Q29. A tap can fill a tank in 6 hours. After half the tank is filled, three more similar
taps are opened. What is the total time taken to fill the tank completely?

A] 3 hrs 15 min
B] 4 hrs 15 min
C] 3 hrs 55 min
D] 3 hrs 45 min

Q30.Ram prepares solutions of alcohol in water according to customer’s needs.


This morning Ram has prepared 27 litres of a 12% alcohol solution and kept it
ready in a 27 litre delivery container to be shipped to the customer. Just before
delivery, he finds out that the customer had asked for 27 litres of 21% alcohol
solution. To prepare what the customer wants, Ram replaces a portion of 12%
solution by 39% solution. How many litres of 12% solution are replaced?
A] 5
B] 9
C] 10
D] 12

www.careers360.com Answer 293


Q31. Four angles of a quadrilateral are in the ratio 1:2:3:4. The angles are

A] 30°. 60°, 90°, 120°

B] 36°. 72°, 108°, 144°

C] 40°. 80°, 120°, 160°

D] None of these

Q32. Find the volume of a triangular pyramid whose base area is 144 cm2 and the
height is 18 cm. (in cm3)

A] 564
B] 864
C] 664
D] 520

Q33.The volume of a square pyramid is 480 cubic cm. If the height of the pyramid
is 10 cm, what are the dimensions of the base of the pyramid?

A] 18 cm by 18 cm
B] 11 cm by 11 cm
C] 12 cm by 12 cm
D] 16 cm by 16 cm

Q34. The ratio of marks obtained by vinod and Basu is 6:5. If the combined
average of their percentage is 68.75 and their sum of the marks is 275, find the
total marks for which exam was conducted.

A] 150
B] 200
C]400
D] None of these.

Q35.Directions: If S denotes multiplied by, V denotes subtracted from, M denotes


added to and L denotes divided by, then,
7 V 24 M 77 L 11 S 5 = ?

A) 21

www.careers360.com Answer 294


B) 18
C) 35
D) 27

Q36.Directions: Correct the following equation by interchanging the two signs


and two numbers.
7 × 6 + 5 – 4 = 33

A) –, ×, and 4, 5
B) ×, +, and 4, 5
C) +, –, and 5, 6
D) ×, –, and 5, 6

READING COMPREHENSION

Q37. Select the most appropriate ANTONYM of the underlined word.

He was compelled by everyone to join the group.

A) Demur

B) Obscured

C) Confessed

D) Induced

Q38.Select the most appropriate meaning of the given idiom.

To kick the bucket

A) To be ill

B) Be sad

C) Be happy

D) To die

Q39.Identify and correct the INCORRECTLY spelt word in the given sentence.

The Collector paid floaral tributes to the statue of Mahatma Gandhi after hoisting
the flag.

www.careers360.com Answer 295


A) floaral

B) statue

C) tributes

D) hoisting

Q40. Select the most appropriate ANTONYM of the underlined word.

Do you know the old adage 'the show must go on’?

A) Misconstruction

B) Rebuke

C) Dare

D) Addiction

Q41. Select the option that expresses the given sentence in passive voice. The
little boy called up his best friend on his birthday.

A) His best friend has been called up by the little boy on his birthday.

B) His best friend was called up by the little boy on his birthday.

C) His best friend is called up by the little boy on his birthday.

D) His best friend was called up by the little boy on

Q42.Sentences of a paragraph are given below in jumbled order. Arrange the


sentences in the correct order to form a meaningful and coherent paragraph.

A. Through a subtle analysis, Woolf raises certain concerns regarding


discrimination against women in a male-dominated society.
B. It's also against the need for freedom of expression in women, and the right to
human dignity and equality.
C. In 'Shakespeare's Sister', Virginia Woolf explores the plight of women in
society in England during the 15th and 16th centuries.
D. It is against the denial of education to the girl-child and violence against
women.

A) BACD

www.careers360.com Answer 296


B) DCBA

C) ADBC

D) CADB

Q43. Select the most appropriate option to fill in the blank.


She reminds me always that I_______ to be a little more careful.

A) had

B) have

C) must

D) has

Q44.Select the option that can be used as a one-word substitute for the given
group of words.
A person who hates and avoids other people

A) Masochist

B) Pervert

C) Hermit

D) Misanthrope

Q45.Select the most appropriate ANTONYM of the word given in brackets to fill in
the blank.
Mr. Khanna is quite___________ (callous) towards the victims of the earthquake.

A) rationalistic

B) unsympathetic

C) sympathetic

D) nondiplomatic

Q46.Select the option that expresses the given sentence in passive voice.

Who stole my tickets?

www.careers360.com Answer 297


A) My tickets was stolen by whom?

B) My tickets got stolen by who?

C) By whom were my tickets stolen?

D) My tickets were stolen by who?

Q47.Sentences of a paragraph are given below. While the first and the last
sentences (S1 and S6) are in the correct order, the sentences in between are
jumbled up and named P, Q, R, and S. Arrange the sentences in the correct order
to form a meaningful and coherent paragraph.
(S1) Newspapers, especially of the popular variety, aim to do two things: to
inform and to entertain.
(P) Entertainment may be principally found in the centre pages of the newspaper
in some of the feature articles and in some local news stories.
(Q) The principal aim of the short news report on the front pages is to inform.
(R) Most newspapers are published daily, and the news reporters must make
sure that they have up-to-the-minute news.
(S) In their news reports, they will aim to create a sense of immediacy, to convey
the impression that they have 'rushed' the news to the reader and 'that the
information contained in the very latest.'
(S6) Reporters also wish to create a sense of authenticity, that is, that their news
is accurate and genuine.

A) Q, S, R, P

B) R, P, S, Q

C) S, R, P, Q

D) P, Q, R, S

Q48. Select the most appropriate ANTONYM of the word given in brackets to fill in
the blank.
The gradual __________ (curtail) in the expenditures made them tense.

A) indent

B) innate

C) increase

D) insinuate

www.careers360.com Answer 298


GENERAL KNOWLEDGE
Q49. Which actor received the Academy Award (Oscar) for Best Actor
in 2021?

A) Brad Pitt
B) Leonardo DiCaprio
C) Anthony Hopkins
D) Joaquin Phoenix

Q50. The Pulitzer Prize is awarded in which field?

A) Literature and Journalism


B) Medicine and Science
C) Engineering and Technology
D) Performing Arts

Q51. Who is the author of the book "Captain Cool: The MS Dhoni Story," which
provides insights into the life of the former Indian cricket captain?

A) Virender Sehwag
B) Suresh Raina
C) Michael Hussey
D) Gulu Ezekiel

Q52. Which author wrote the book "Gandhi: An Autobiography - The Story of My
Experiments with Truth"?

A) Rabindranath Tagore
B) Jawaharlal Nehru
C) Sardar Patel
D) Mahatma Gandhi

Q53. What is the term for a large, bowl-shaped depression in the Earth's surface,
typically caused by the collapse of a volcano?

A) Plateau
B) Caldera
C) Abyssal plain
D) Cuesta

www.careers360.com Answer 299


Q54.Atomic number of which of the following elements is greater than that of
fluorine?

A) Sodium
B) Beryllium
C) Nitrogen
D) Boron

Q55.By which Constitutional Amendment has the right to property been scrapped
as a Fundamental Right?

A) 42nd
B) 43rd
C) 44th
D) 45th

Q56. Ozone layer is present in

A) trophosphere

B) ionosphere

C) stratosphere

D) exosphere

Q57. Which causes rainfall during winters in the north-western part of India?

A) Western disturbances

B) Cyclonic depression

C) Southwest monsoon

D) Retreating monsoon

Q58. The daily weather map of India is prepared and printed at

A) Kolkata

B) Mumbai

C) New Delhi

www.careers360.com Answer 300


D) Pune

Q59. Tsangpo is the other name in the Tibet for

A) Kosi

B) Gandak

C) Brahmaputra

D) Ganga

Q60. The sideways erosion that widens the river valley is called

A) lateral erosion

B) vertical erosion

C) side erosion

D) mean erosion

www.careers360.com Answer 301


ANSWER KEY
____________________________________________________________________________________

1 2 3 4 5 6 7 8 9 10
A C A A D A A A D B
11 12 13 14 15 16 17 18 19 20
D B A B B C A C B C
21 22 23 24 25 26 27 28 29 30
B B A B A A A B D B
31 32 33 34 35 36 37 38 39 40
B B C B B C D D A A
41 42 43 44 45 46 47 48 49 50
B D B D C C D C C A
51 52 53 54 55 56 57 58 59 60
D D B A C C A D C A

www.careers360.com Solutions 302


SOLUTIONS
____________________________________________________________________________________

LOGICAL REASONING

1-A

Now, let's analyze the conclusions –


Conclusion (I): No night is day – From the diagram, we can see that circles representing
night and day do not overlap which means there can not be any positive relation
between them. So, this conclusion follows.
Conclusion (II): Some dawns are night – From the diagram, we can see that circles that
represent dawn and night do not overlap. which means there can not be any positive
relation between them. So, this conclusion does not follow.

So, from the above, the only conclusion I follow. Hence, the first option is correct.

2-C
According to the given statements, the Venn diagram will be –

www.careers360.com Back to Index 303


Now, let's analyse the conclusions –
Conclusion (I): Some leaves are plants – From the diagram, we can see that some part
of the circles that represent plants and leaves overlap each other which means that the
given conclusion can be drawn from the statements. So, this conclusion follows.
Conclusion (II): Some plants are flowers – From the diagram, we can see that some
part of the circles that represent plants and flowers overlap each other. which means
that the given conclusion can be drawn from the statements. So, this conclusion follows.
So, this conclusion follows.

So, both Conclusion I and II follow. Hence, the third option is correct.

3-A
Nuclear fusion is the process by which stars like the Sun produce energy. It occurs
when atomic nuclei combine to form a heavier nucleus, releasing a tremendous amount
of energy in the process.

4-A
The primary function of red blood cells is to transport oxygen to body tissues, and they
do so by containing hemoglobin, which binds to oxygen and carries it throughout the
body.

5-D
Both statements are true. The Nile River is indeed the longest river in the world, and the
Amazon River is the second-longest.

6-A
Neon is a noble gas, but helium is not a halogen.

7-A
Argument I supports the abolition of the death penalty due to ethical and practical
concerns, while Argument II defends the death penalty as a deterrent against crimes.
Argument I provides a stronger case against the statement.

8-A
Argument I supports lowering the voting age based on the idea that 16-year-olds are
affected by political decisions. Argument II raises concerns about the maturity and
understanding of 16-year-olds but doesn't directly refute the reasons provided in
Argument I.

www.careers360.com Back to Index 304


9-D
Let's check the options —

First option: 20 – 30; 20 + 10 = 30

Second option: 35 – 45; 35 + 10 = 45

Third option: 45 – 55; 45 + 10 = 55

Fourth option: 25 – 30; 25 + 5 = 30; This option is different from the other options.

Hence, the fourth option is correct.

10-B
Let's check the options —

First option: (1, 11, 1111); 1 × 11 = 11; 1 × 1111 = 1111

Second option: (2, 22, 4444); 2 × 11 = 22; 2 × 1111 = 2222 is not equal to 4444; This
option is different from the other options.

Third option: (4, 44, 4444); 4 × 11 = 44; 4 × 1111 = 4444

Fourth option: (8, 88, 8888); 8 × 11 = 88; 8 × 1111 = 8888

Hence, the second option is correct.

11-D
First, we need to draw a family tree.

www.careers360.com Back to Index 305


Now, from the above diagram–
Raju's grandfather's son's daughter is Raju's sister.
So, that girl is the daughter of Raju's sister and Raju is her uncle.
Therefore, the girl is Raju's niece. Hence, the fourth option is correct.

12-B
Given:

The weight of P is twice that of Q, i.e., P = 2Q

The weight of Q is half that of R, i.e., R = 2Q

The weight of R is 3 times of T, i.e., R = 3T

The weight of T is half that of S, i.e., S = 2T

By concluding all the given information, we have:

P=R>S>Q>T

Therefore, it is clear that the weight of T is not more than any of P, Q, R, or S.

Hence, the second option is correct.

LEGAL REASONING
13-A
The Supreme Court is the highest court in India, with the power of judicial review and
the final authority on constitutional matters.

14-B
Dr. B.R. Ambedkar chaired the drafting committee of the Indian Constitution and is often
referred to as its architect.

15-B
Habeas Corpus is a legal action that protects an individual's right to be free from
unlawful detention or imprisonment.

16-C
This right ensures protection against arbitrary arrest and detention.

17-A
The legal voting age in India is 18 years.

www.careers360.com Back to Index 306


18-C
Defamation includes both oral (slander) and written (libel) false statements that harm a
person's reputation.

19-B
The Indian Contract Act governs contracts in India.

20-C
Theft is punishable by both imprisonment and/or a fine under the Indian Penal Code.

21-B
The 122nd Amendment of the Constitution introduced the Goods and Services Tax
(GST) in India.

22-B
Mandamus is a writ issued by a higher court to compel a lower court or public authority
to perform a duty.

23-A
The United Nations plays a key role in the peaceful settlement of disputes between
countries.

24-B
Caveat Emptor is a Latin term that means the buyer is responsible for checking the
quality and suitability of goods before purchase.

ANALYTICAL REASONING
25-A

26-A

Time taken =

Let the actual speed be

www.careers360.com Back to Index 307


Then,

27-A
Speed downstream =

Time taken to travel

28-B

29-D
Time taken by one tap to fill half of the tank =3 \mathrm{hrs}.

So, total time taken =

30-B
Direct solution
(27-x) X 12 + 39 x = 27 x 21
324 – 12x + 39x = 567

www.careers360.com Back to Index 308


27x = 233
X= 9L

31-B

Let ABCD be the given quadrilateral and let

Since the sum of the angles of a quadrilateral is 360°

32-B
Given,

Base area = 144 cm2

Height = 18 cm.

The volume of a triangular pyramid = (1/3) x (Area of Base) x (height)

33-C
Given:

Volume = 480 cubic cm.

height = 10 cm

the volume of a square pyramid formula = ,

Let base area = a x a

(since, base is a square)

⇒ 480 = 1/3 a2 h

⇒ 480 = 1/3 x a2 x 10

www.careers360.com Back to Index 309


⇒ 48 = 1/3 x a2

⇒ 480 x 3 = a2

⇒ 144 = a2

⇒ a = 12 cm

34-B
let vinod marks be 6x and basu is 5x. Therefore the sum = 11x
but the sum of marks is given as 275 = 11x
we get x = 25, so vinod marks = 150, Basu = 125
combined average = (150 +125)/2 = 137.5
If the total marks of exam is 100 their combined avg. Of their % is 68.75.
Therefore combined average % of 137.5 then total marks = (137.5/68.75)*100 = 200

35-B
Given:
7 V 24 M 77 L 11 S 5 = ?
On replacing the letters with the assigned mathematical signs, the equation becomes –
= 7 – 24 + 77 ÷ 11 × 5
= 7 – 24 + 7 × 5
= 7 – 24 + 35
= 42 – 24
= 18

So, 18 is the required answer to the given equation. Hence, the second option is correct.

36-C
Given:
7 × 6 + 5 – 4 = 33
Let's check the options –
First option: –, ×, and 4, 5
⇒ 7 – 6 + 4 × 5 = 33
On solving the L.H.S. of the equation –
=7–6+4×5
= 7 – 6 + 20
= 21 ≠ 33
Second option: ×, +, and 4, 5
⇒ 7 + 6 × 4 – 5 = 33
On solving the L.H.S. of the equation –
=7+6×4–5

www.careers360.com Back to Index 310


= 7 + 24 – 5
= 26 ≠ 33
Third option: +, –, and 5, 6
⇒ 7 × 5 – 6 + 4 = 33
On solving the L.H.S. of the equation –
=7×5–6+4
= 35 – 6 + 4
= 33
Fourth option: ×, –, and 5, 6
⇒ 7 – 5 + 6 × 4 = 33
On solving the L.H.S. of the equation –
=7–5+6×4
= 7 – 5 + 24
= 26 ≠ 33

Only the third option satisfies the given equation. Hence, the third option is correct.

READING COMPREHENSION

37-D
The correct choice is the fourth option, "induced".

"Induced" means 'to succeed in persuading or leading (someone) to do something'


which is opposite to what compelled means. Compelled means 'force or oblige
(someone) to do something.'

The meanings of the other options are as follows:

● Demur means to show reluctance.


● Obscured means to keep from being seen.
● Confess means to admit something.

38-D
The correct choice is the fourth option, "to die".

The idiom "To kick the bucket" means to die or to stop living. This makes the fourth
option the appropriate choice.

www.careers360.com Back to Index 311


39-A
The correct choice is the first option i.e. "floaral."

The correct spelling is "floral," not "floaral." Floral relates to flowers or the use of
flowers, which is accurate in the context of paying floral tributes.

The meanings of other options are as follows:

Statue: A statue is a three-dimensional sculpture, often representing a person, an


animal, an object, or an abstract form.

Tributes: Tributes are expressions of admiration, respect, gratitude, or honour paid to


someone or something.

Hoisting: Hoisting refers to the action of raising or lifting something, often using ropes,
pulleys, cranes, or other mechanisms.

40-A
The correct answer is the first option i.e. "misconstruction."

"Misconstruction" means 'the action of misconstructing words or actions, or


misinterpretation' which is opposite to what the adage means. It means 'a proverb or
short statement expressing a general truth.'

The meanings of other options are as follows:

● Rebuke: It means to express sharp disapproval or criticism.

● Dare: This is the word being considered, and it means to have the courage to do
something or to take risks.

● Addiction: It refers to a compulsive and harmful need for a substance or activity.

41-B
The correct answer is the second option i.e. "His best friend was called up by the
little boy on his birthday."

The reason for this answer is that the passive voice construction typically involves using
a form of the verb "to be" along with the past participle of the main verb. In this case,
"was called" is the correct passive construction, indicating that the action (calling up)
was performed by the little boy on his friend.

www.careers360.com Back to Index 312


42-D
The correct order is the fourth option i.e. "CADB."

The first sentence should be C since it gives general information about 'Shakespeare's
Sister'. Sentence A also supports sentence C by explaining what it aims to do,
therefore, it should come next. Sentence D provides examples of the previous two
sentences. Thus, it should be next. Sentence B concludes the paragraph by
summarising the broader theme and ideas.

Therefore, the correct order is CADB.

43-B
The correct answer is the second option i.e. "have."

The sentence is in present perfect tense and the auxiliary verb "have" is used in such
sentences.

Therefore, the complete sentence is "She reminds me always that I have to be a little
more careful."

44-D
The correct answer is the fourth option i.e. "Misanthrope."

"Misanthrope" means 'a person who dislikes humankind and avoids human society'
which means is the same as the group of words provided in the question.

The meanings of other options are as follows:

● Masochist: This term refers to a person who derives pleasure from their own
pain or humiliation.

● Pervert: This term generally refers to someone whose behaviour is sexually


abnormal or deviant.

● Hermit: A hermit is someone who lives in seclusion, often for religious or


spiritual reasons.

www.careers360.com Back to Index 313


45-C
The correct answer is the third option i.e. "sympathetic."

"Sympathetic" means 'feeling, showing, or expressing sympathy' which is opposite to


what callous means. It means 'showing or having an insensitive and cruel disregard for
others.'

The meanings of other options are as follows:

● Rationalistic: This term relates to a philosophical or religious perspective


emphasizing reason or logic.

● Unsympathetic: It expresses a lack of sympathy or compassion.

● Nondiplomatic: not related to, or concerned with diplomats or diplomacy or not


employing tact and conciliation.

Therefore, the complete sentence is "Mr Khanna is quite sympathetic towards the
victims of the earthquake."

46-C
The correct answer is the third option i.e. "By whom were my tickets stolen?"

To convert the sentence to passive voice, we need to identify the doer of the action and
place it after the verb. In this case, "Who" becomes "By whom" in the passive
construction. The correct passive structure is: "By + Subject (doer of the action) +
Verb (in the corresponding tense) + Object."

In passive voice constructions, the transformation involves using a form of the verb "to
be" (am, is, are, was, were, etc.) and the past participle of the main verb. So, the past
simple tense of the verb "stole" changes to "were stolen."

47-D
The correct order is the fourth option i.e. P, Q, R, S.

Sentence (P) comes first after S1 because it expands on the aspect of entertainment.
After that, sentence (Q) is the most appropriate because it introduces the primary
purpose of front-page news reports. Sentence (R) comes next as it provides context to
the previous sentence. Sentence (S) is at the end because it continues to describe the
approach of news reporters.

www.careers360.com Back to Index 314


48-C
The correct answer is the third option i.e. "increase."

"Curtail" means to reduce or cut back, while "increase" means to make or become
larger or more extensive.

The meanings of other options are as follows:

● Indent: It means to notch or cut the edge of something, often used in the context
of writing or formatting.

● Innate: It means existing from birth, inherent, or natural.

● Insinuate: It means to suggest or imply something indirectly.

Therefore, the sentence is "The gradual increase in expenditures made them tense."

GENERAL KNOWLEDGE

49-C
In the 2021 Academy Awards, Anthony Hopkins won the Best Actor award for his role
in "The Father."

50-A
The Pulitzer Prize is awarded for achievements in journalism, literature, and musical
composition.

51-D
Gulu Ezekiel. "Captain Cool: The MS Dhoni Story" is written by Gulu Ezekiel.

52-D
Mahatma Gandhi. This book is an autobiography written by Mahatma Gandhi himself,
detailing his life, philosophy, and his experiments with truth.

53-B
A caldera is a large, bowl-shaped depression that forms when a volcano collapses after
a massive eruption or when a magma chamber is emptied.

54-A
Among the elements listed, sodium has a greater atomic number than fluorine. The

www.careers360.com Back to Index 315


following are the atomic numbers: The atomic number of sodium is 11. Beryllium is the
fourth element in the periodic table. Boron is the fifth element in the periodic table.
Nitrogen is the seventh element in the periodic table. Fluorine is the ninth element in the
periodic table. As a result, sodium has a greater atomic number than fluorine (atomic
number 9).

55-C
With the passage of the 44th Constitutional Amendment in 1978, which changed Article
300A to make property rights simple legal rights, property rights were removed from the
list of fundamental freedoms. Property ownership was originally regarded as a
fundamental right that the Constitution guaranteed. The Constitution, however, made it
clear that the government could seize property in order to advance the welfare of the
public.

56-C
The correct answer is the stratosphere

The ozone layer is located in the stratosphere and plays a crucial role in protecting
Earth from harmful ultraviolet (UV) rays. It absorbs approximately 97-99% of the
incoming solar radiation. Ozone found at ground level is considered a pollutant and is
often referred to as bad ozone, whereas ozone present in the stratosphere is known as
good ozone.

57-A
The correct answer is Western disturbances.

Western disturbances are extratropical weather phenomena that typically affect the
Indian subcontinent, especially its northern and northwest parts. They are storms that
originate in the Caspian or Mediterranean Seas and provide non-monsoonal rainfall to
northwest India during the winter season. They are categorised as extratropical
cyclones.

58-D
The correct answer is Pune.

The National Data Centre of India Meteorological Department (IMD) in Pune


prepares and prints the daily weather map of India. The India Meteorological
Department (IMD) is an agency of the Government of India's Ministry of Earth Sciences.
It is the primary agency of meteorological monitoring, forecasts and seismology.

www.careers360.com Back to Index 316


59-C
The answer is Brahmaputra.

The Brahmaputra River is 2900 km long and originates from the Himalayas. It flows
through Tibet, India, and Bangladesh before draining into the Bay of Bengal. It is known
as the Brahmaputra or Luit in Assamese, Yarlung Tsangpo in Tibetan, the Siang or
Dihang River in Arunachali and the Jamuna River in Bengali.

60-A
The answer is lateral erosion.

The river becomes more profound due to vertical erosion, while lateral erosion erodes
the banks and forms a cliff. In lateral erosion, the river erodes its banks and widens the
river channel. It is also known as sideways erosion. A V-shaped river valley may be
found in the upper course of the river, formed by both vertical and lateral erosion
combined.

www.careers360.com Back to Index 317


MOCK TEST - 10
____________________________________________________________________________

LOGICAL REASONING
Q71. Direction: In the following question two statements are given, followed by
two conclusions I and II. You have to consider the statements to be true even if
they are seen to be at variance from commonly known facts. You have to decide
which of the given conclusions, if any follows from the given statements.
Statements:
I. All clouds are fogs.
II. All fogs are white.
Conclusions:
I. Some white are clouds.
II. Some fogs are clouds.

A) Only conclusion I follows


B) Only conclusion II follows
C) Both I and II follow
D) Neither I nor II follows

Q72.Direction: In the following question below, some statements are followed by


some conclusions. Taking the given statements to be true, even if they seem to
be at variance from commonly known facts, read all the conclusions and then
decide which of the given conclusions logically follows from the given
statements.

Statements:

I. Some white are yellow.

www.careers360.com Answer 318


II. All yellow are blue.

Conclusions:

I. Some blue are white.

II. Some yellow are white.

A) OnlyI
B) Only II
C) Neither I nor II
D) Both I and II

Q73. Assertion: Ice floats on water.


Reason: Ice is less dense than liquid water.

A) Both the assertion and reason are true, and the reason is a correct explanation of the
assertion.

B) Both the assertion and reason are true, but the reason is not a correct explanation of
the assertion.

C) The assertion is true, but the reason is false.

D) The assertion is false, but the reason is true.

Q74.Assertion: The Earth experiences seasons due to its axial tilt.

Reason: The axial tilt causes different parts of the Earth to receive varying
amounts of sunlight throughout the year.

A) Both the assertion and reason are true, and the reason is a correct explanation of
the assertion.

www.careers360.com Answer 319


B) Both the assertion and reason are true, but the reason is not a correct explanation of
the assertion.

C The assertion is true, but the reason is false.

D) The assertion is false, but the reason is true.

Q75.According to the given statements, Which of the following options is correct.

Statement 1:The chemical symbol for copper is Co.

Statement 2:The chemical symbol for cobalt is Cu.

A) Statement 1 is correct, but Statement 2 is incorrect.

B) Statement 2 is incorrect.

C) Statement 1 is incorrect.

D) Statement 2 is correct, but Statement 1 is also correct.

Q76.According to the given statements, Which of the following options is correct


Statement 1: An arachnid is an insect.

Statement 2: A spider is an arachnid.

A) Statement 1 is correct, but Statement 2 is incorrect.

B) Statement 2 is incorrect.

C) Statement 1 is incorrect.

D) Statement 2 is correct, but Statement 1 is also correct.

Q77. Direction: In the following question, a statement is given, followed by two


arguments, I and II. You have to consider the statement to be true, even if it
seems to be at variance from commonly known facts. You have to decide which
of the given arguments, if any, follows from the given statement.

www.careers360.com Answer 320


Statement:
"Should public schools have mandatory religious education?"
Arguments:
I. Yes, it promotes understanding and tolerance among students of different religions.
II. No, it can lead to religious discrimination and should be a personal choice.

A) Only Argument I follows.


B) Only Argument II follows.
C) Both I and II follows
D) Neither Argument I nor Argument II follows.

Q78. Direction: In the following question, a statement is given, followed by two


arguments, I and II. You have to consider the statement to be true, even if it
seems to be at variance from commonly known facts. You have to decide which
of the given arguments, if any, follows from the given statement.

Statement:
"Should the government regulate the sale of sugary drinks and snacks?"
Arguments:
I. Yes, it can help combat the rising rates of obesity and related health issues.
II. No, it interferes with personal choice and the free market.

A) Only Argument I follows.


B) Only Argument II follows.
C) Both I and II follows
D) Neither Argument I nor Argument II follows.

www.careers360.com Answer 321


Q79. Direction: In the following question, find the odd letter from the given
alternatives.

A) QJ
B) SH
C) LN
D) UF

Q80.Direction: In the following question, find the odd letter from the given
alternatives.

A) NT
B) WC
C) JP
D) GE

Q81.Direction: Introducing a man, Amar says, "His wife is the only daughter of my
maternal grandfather". How is the man related to Amar?

A) Father
B) Grandfather
C) Son
D)Grandson

Q82. Direction: In a row of 63 cars, black car is 32nd from the right end. What is
its position from left end?
A) 32
B) 31
C) 33
D) 34

www.careers360.com Answer 322


LEGAL REASONING
Q13. Question: Which constitutional amendment in India added the Right to
Education as a fundamental right?

A) 82nd Amendment
B) 86th Amendment
C) 92nd Amendment
D) 94th Amendment

Q14. Question: What is the legal term for a written statement submitted in a court
proceeding, typically by one of the parties?

A) Affidavit
B) Pleading
C) Subpoena
D) Warrant

Q15. Question: In criminal law, what is the term for the act of intentionally helping
another person commit a crime?

A) Conspiracy
B) Accomplice
C) Aiding and abetting
D) Solicitation
Q16. Question: Which international treaty addresses the protection of intellectual
property rights?

A) Paris Convention
B) Geneva Convention
C) Kyoto Protocol
D) Hague Convention

Q17.What legal doctrine allows a person to use another person's property without
permission for a certain period, as long as specific conditions are met?

A) Adverse Possession
B) Eminent Domain
C) Trespass
D) Nuisance18.Question: Which type of law governs disputes between private
parties, including individuals and corporations?

www.careers360.com Answer 323


A) Criminal Law
B) Civil Law
C) Administrative Law
D) Constitutional Law

19. Question: What is the primary purpose of the "Doctrine of Precedent" in


common law systems?

A) To ensure consistency in judicial decisions


B) To encourage creative interpretation of laws
C) To prioritize legislative intent over precedent
D) To eliminate the need for appeals
Q20.Which international court is responsible for settling legal disputes between
states and giving advisory opinions on legal questions referred by the United
Nations?

A) International Court of Justice (ICJ)


B) International Criminal Court (ICC)
C) International Tribunal for the Law of the Sea (ITLOS)
D) International Criminal Tribunal for the Former Yugoslavia (ICTY)

Q21. Question: In contract law, what is the term for a situation where both parties
have not yet fulfilled their contractual obligations?

A) Breach of contract
B) Frustration of contract
C) Executed contract
D) Executory contract

Q22.Which legal principle assumes innocence until proven guilty and places the
burden of proof on the prosecution?

A) Presumption of innocence
B) Beyond a reasonable doubt
C) Habeas corpus
D) Ex post facto
Q23.What legal term is used to describe a situation where a contract is
terminated, and the parties are restored to their original positions?

www.careers360.com Answer 324


A) Rescission
B) Novation
C) Accord and Satisfaction
D) Estoppel

Q24.Under which law is the protection of personal data and information


addressed in India?

A) Information Technology Act


B) Right to Privacy Act
C) Personal Data Protection Bill
D) Cybersecurity and Data Protection Act

ANALYTICAL REASONING

Q25. In covering a distance of 30 km, Abhay takes 2 hours more than Sameer. If
Abhay doubles his speed, then he would take 1 hour less than Sameer. Abhay's
speed is:

A] 5 kmph

B] 10 kmph

C] 12 kmph

D] 8 kmph

Q26.Robert is travelling on his cycle and has calculated to reach point A at 2 P.M.
if he travels at 10 kmph, he will reach there at 12 noon if he travels at 15 kmph. At
what speed must he travel to reach A at 1 P.M.?

A] 8 kmph

B] 12 kmph

C] 10 kmph

D] 14 8 kmph

www.careers360.com Answer 325


Q27. A man's speed with the current is 15 km/hr and the speed of the current is
2.5 km/hr. The man's speed against the current is:

A] 12.5 km/hr
B] 5.5 km/hr
C] 6.5 km/hr
D] 10 km/hr

Q28. A, B and C can do a piece of work in 20, 30 and 60 days respectively. In how
many days can A do the work if he is assisted by B and C on every third day?

A] 12 days
B] 15 days
C] 10 days
D] 25 days

Q29. Three taps A, B and C can fill a tank in 12, 15 and 20 hours respectively. If A
is open all the time and B and C are open for one hour each alternately, the tank
will be full in:

A] 6 hours

B] 7 hours

C] 10 hours

D] 8 hours

Q30. There are two containers: the first contains 500ml of alcohol, while the
second contains 500 ml of water. Three cups of alcohol from the first
container is removed and is mixed well in the second container. Then three
cups of this mixture is removed and is mixed in the first container. Let ‘A’
denote the proportion of water in the first container and ‘B’ denote the proportion
of alcohol in the second container. Then A,

A] A > B
B] A < B
C] A = B
D] Cannot be determined

www.careers360.com Answer 326


Q31. Three angles of a quadrilateral are 110°, 80°, 95°. The measure of the fourth
angle is

A] 55°

B] 85°

C] 75°

D] 45°

Q32.The curved surface area of a cylindrical pillar is 264 m2 and its volume is 924
m3. Find the ratio of its diameter to its height.

A] 3 : 7
B] 7 : 3
C] 7 : 6
D] 2 : 3

Q33. A cistern of capacity 8000 litres measures externally 3.3 m by 2.6 m by 1.1 m
and its walls are 5 cm thick. The thickness of the bottom is:

A] 90 cm
B] 70 cm
C] 1 dm
D] 5m

Q34. The marks scored by a student in three subjects are in the ratio of 4 : 5 : 6. If
the candidate scored an overall aggregate of 60% of the sum of the maximum
marks and the maximum marks in all three subjects is the same, in how many
subjects did he score more than 60%?

A] 1
B] 2
C] 3
D] None of the subjects
Q35.Directions: The following equations have been solved based on a certain
pattern. Find the correct answer for the unsolved equation on that basis.
If 2 * 3 * 5 = 523, 6 * 4 * 1 = 164, 8 * 2 * 4 = 482, then 6 * 8 * 2 = ?

A) 826

www.careers360.com Answer 327


B) 268
C) 286
D) 628

Q36.Directions: In the following question, select the related word from the given
alternatives.
Line : Square :: Arc : ?

A) Ring
B) Sphere
C) Circle
D) Ball

READING COMPREHENSION

Q37.Directions: In the following question, a sentence is given with a blank that is


to be filled in with an appropriate word. Four alternatives are suggested; choose
the correct alternative out of them as your answer.

He worked ________ of his ability.

A) at the best

B) with the best

C) in the best

D) to the best

Q38. Directions: In the following question, the sentence given with a blank is to
be filled in with an appropriate word. Select the correct alternative out of the four.

Puresh wandered ____ through life, with no clear ideas of where he was heading
and what he wanted to accomplish.

A) madly

B) excitedly

C) crazily

D) aimlessly

www.careers360.com Answer 328


Q39. Directions: In the following question, the sentence given with a blank is to
be filled in with an appropriate word. Select the correct alternative out of the four.

A man is known___ the company he keeps.

A) By

B) For

C) In

D) Of

Q40. Directions: In the following question, the sentence given with a blank is to
be filled in with an appropriate word. Select the correct alternative out of the four.

Sally parked and got ___ the car quickly.

A) Out of

B) In

C) Over

D) On

Q41.Directions: In the following question, the sentence given with a blank is to be


filled in with an appropriate word. Select the correct alternative out of the four.

Anna is not popular. She has__ friends.

A) A few

B) Few

C) A little

D) Little

Q42.Directions: In the following question, the sentence given with a blank is to be


filled in with an appropriate word. Select the correct alternative out of the four.

The convict was ___ on a Monday.

A) Hanging

www.careers360.com Answer 329


B) Hanged

C) Hung

D) Hang

Q43. Directions: In the following question, the sentence given with a blank is to
be filled in with an appropriate word. Select the correct alternative out of the four.
___ she can't drive, Anita has bought a car.

A) Since

B) Even if

C) Even though

D) Whether or not

Q44. Directions: In the following question, a sentence is given with a blank that is
to be filled in with an appropriate word. Four alternatives are suggested; choose
the correct alternative out of them as your answer.

If you do well in the exam next year then my efforts ________ successful.

A) Have been

B) Would have been

C) Will be

D) Has been

Q45. Directions: In the following question, a sentence is given with a blank that is
to be filled in with an appropriate word. Four alternatives are suggested; choose
the correct alternative out of them as your answer.

We _________ not hurry, we have plenty of time.

A) Must

B) Would

C) Need

D) Should

www.careers360.com Answer 330


Q46. Directions: In the following question, the sentence given with a blank is to
be filled in with an appropriate word. Select the correct alternative out of the four.

They were worried about a new cult that ____ many followers.

A) Is gaining

B) Must be gaining

C) Was gaining

D) Were gaining

Q47. Directions: In the following question, a sentence is given with a blank that is
to be filled in with an appropriate word. Four alternatives are suggested; choose
the correct alternative out of them as your answer.
Legislation was passed to punish brokers who____ their client's funds.

A) devastate

B) devour

C) defalcate

D) dawdle

Q48.Directions: In the given question, a sentence is given with a blank to be filled


in with an appropriate word. Four alternatives are suggested for each question.
Choose the correct alternative out of the four alternatives.

The warning bells ___ several times before anyone realized the danger.

A) were ringing

B) had rung

C) had rang

D) would have rung

GENERAL KNOWLEDGE

Q49. Who determines the retirement age for High Court judges in India?

www.careers360.com Answer 331


A) The Chief Justice of India
B) The President of India
C) The Prime Minister of India
D) State Legislatures

Q50. What is the maximum number of members that the Lok Sabha can have?

A) 543
B) 545
C) 250
D) 12

Q51.______ scheme launched by the Central Government aims to promote


entrepreneurship among people from Scheduled Castes or Scheduled Tribes.

A) Pradhan Mantri Fasal Bima Yojana

B) Gram Uday Se Bharat Uday Abhiyan

C) Stand Up India scheme

D) National Rurban Mission

Q52. _____scheme launched by the Central Government aims to improve rural


livelihoods, promote rural development, and strengthen the Panchayati Raj
across the country.

A) Pradhan Mantri Fasal Bima Yojana

B) Gram Uday se Bharat Uday Abhiyan

C) Stand up India scheme

D) National RU URBAN Mission

Q53. ______ was launched by Prime Minister Narendra Modi as a part of the Beti
Bachao Beti Padhao campaign.

A) Sukanya Samriddhi Account

B) Bal Swachhta Mission

C) Pradhan Mantri Jan Dhan Yojana

www.careers360.com Answer 332


D) Beti Bachao Beti Padhao Yojana

Q54. ____ is a welfare scheme started by the Prime Minister to provide LPG
connections to domestic households that fall under the below-the-poverty line
category.

A) Gram Uday Se Bharat Uday Abhiyan

B) Pradhan Mantri Ujjwala Yojana

C) Pradhan Mantri Surakshit Matritva Yojana

D) Vidyanjali Yojana

Q55. From the national point of view, which of the following indicates a micro
approach?

A) Study of the scale of mobile phones by BSNL

B) Unemployment among women

C) Per capita income in India

D) Inflation in India

Q56. The law of demand states that

A) If the price of a good increases, the demand for that good decreases

B) If the price of a good increases, the demand for that good increases

C) If the price of a good increases, the quantity demanded of that good decreases

D) If the price of a good increases, the quantity demanded of that good increases

Q57. The garden inside the Taj Mahal is known as ____.

A) Mughal Garden

B) Taj Bageecha

C) Taj Mahal Garden

D) Mahal Bageecha

www.careers360.com Answer 333


Q58. Noor Jahan was the wife of which Mughal Emperor?

A) Akbar

B) Aurangzeb

C) Jahangir

D) Shah Jahan

Q59. In which of the following market forms, a firm does not exercise control over
price?

A) Monopoly

B) Perfect competition

C) Oligopoly

D) Monopolistic competition

Q60.Which of the following brings out the 'consumer price index number' for
industrial workers?

A) RBI

B) The Labour Bureau

C) Commerce Department

D) NITI Aayog

www.careers360.com Answer 334


ANSWER KEY
____________________________________________________________________________

1 2 3 4 5 6 7 8 9 10
C D A A A D D A C D
11 12 13 14 15 16 17 18 19 20
A A B B C A A B A A
21 22 23 24 25 26 27 28 29 30
D A A C A B D B B C
31 32 33 34 35 36 37 38 39 40
C B C A B C D D A A
41 42 43 44 45 46 47 48 49 50
B B C C C C C B D B
51 52 53 54 55 56 57 58 59 60
C B A B A C D C B B

www.careers360.com Solutions 335


SOLUTIONS
____________________________________________________________________________

LOGICAL REASONING
1-C
According to the given statements, the Venn diagram will be –

Now, let's analyze the conclusions –


Conclusion (I): Some white are clouds – From the diagram, we can see that the circle
that represents white overlaps the circle that represents clouds completely which means
the relation of 'some' between white and clouds can be drawn from this. So, this
conclusion follows.
Conclusion (II): Some fogs are clouds – From the diagram, we can see that some part
of the circle that represents fogs partially overlaps the circle that represents clouds
which means the relation of 'some' between fogs and clouds can be drawn from this.
So, this conclusion follows.

So, from the above, both Conclusion I and II follow. Hence, the third option is correct.

www.careers360.com Back to Index 336


2-D

Let's analyze the conclusions

Conclusion (I): Some blue are white.

From the Venn diagram, we can see that the circle representing blue overlaps with the
circle representing white. So, some blue are white.

Conclusion (II): Some yellow are white.

As it is given in statement (I) that some white are yellow. The converse of this is also
true. So, Some yellow are white. So, This conclusion follows.

So, both conclusions I and II follow.

Hence, fourth option is correct.

3-A
Ice is less dense than liquid water because of the unique properties of water, which are
due to hydrogen bonding. This lower density is why ice floats.

4-A
The Earth's axial tilt causes varying amounts of sunlight to reach different parts of the
planet, resulting in the changing seasons.

5-A
The chemical symbol for copper is Cu, and the chemical symbol for cobalt is Co.

www.careers360.com Back to Index 337


6-D
Both statements are true. An arachnid is not an insect, but a spider is indeed an
arachnid.

7-D
Both arguments provide valid perspectives, but they don't directly follow from the
statement. Argument I emphasizes the benefits of religious education, while Argument II
highlights potential drawbacks.

8-A
Argument I supports government regulation for health reasons, while Argument II
argues against it for reasons of personal choice and market freedom.

9-C
Let's check the options —

First option: QJ

Q and J are opposite in the English alphabetical series.

Second option: SH

S and H are opposite in the English alphabetical series.

Third option: LN

L and N are opposite in the English alphabetical series.

Fourth option: UF

U and F are opposite in the English alphabetical series.

Hence, the third option is correct.

www.careers360.com Back to Index 338


10-D
Let's check the options —

First option: NT → N + 6 = T

Second option: WC → W + 6 = C

Third option: JP → J + 6 = P

Fourth option: GE → G – 2 = E

Therefore, the fourth option is different from the other three. So, it's an odd one out.

Hence, the fourth option is correct.

11-A
First, we need to draw a family tree.

Now, from the above diagram –


The only daughter of Amar's maternal grandfather is Amar's mother.
So, the man is Amar's mother's husband is Amar's father.
Therefore, the man is Amar's father. Hence, the first option is correct.

www.careers360.com Back to Index 339


12-A
Given:

Total number of cars = 63


Position of the black car from the right end = 32

Therefore, the position of the black car from the left end

= (63 – 32) + 1
= 31 + 1
= 32

Hence, the first option is correct.

LEGAL REASONING
13-B
The 86th Amendment of the Indian Constitution added Article 21A, making the Right to
Education a fundamental right.

14-B
Pleadings are formal written statements submitted by the parties involved in a legal
dispute.

15-C
Aiding and abetting involves intentionally helping someone else commit a crime.

16-A
The Paris Convention is an international treaty dealing with the protection of intellectual
property.

17-A
Adverse Possession is a legal concept where a person can acquire ownership of
another's property through continuous use.

18-B
Civil Law deals with private disputes between individuals or organizations.

www.careers360.com Back to Index 340


19-A
The Doctrine of Precedent, or stare decisis, aims to provide consistency and
predictability in legal decisions.

20-A
The ICJ is the principal judicial organ of the United Nations and settles legal disputes
between states.

21- D
An executory contract is one where both parties still have obligations to fulfill.

22-A
The presumption of innocence is a fundamental principle in criminal law.

23-A
Rescission refers to the cancellation of a contract, returning the parties to their
pre-contractual state.

24-C
As of my last knowledge update in January 2022, the Personal Data Protection Bill
addresses the protection of personal data and information in India.

ANALYTICAL REASONING
25-A

Let Abhay's speed be

Then,

26-B
Let the distance travelled by

www.careers360.com Back to Index 341


Then,

Time taken to travel


So, Robert started 6 hours before 2 P.M. i.e., at 8 A.M.

Required speed =

27-D
Man's rate in still water =
Man's rate against the current =

28-B

A's 2 day's work =

(A+B+C) 's 1 day's work =

Work done in 3 days =

Now, work is done in 3 days.


\therefore Whole work will be done in =15 days.

www.careers360.com Back to Index 342


29-B

Now, it is the turn of A and B and part is filled by A and B in 1 hour.


\therefore Total time taken to fill the tank =

30-C
Let the capacity of each = 100 ml
So 300 ml removed and poured into second
First vessel = 200 ml alcohol
Second = 500 ml water+ 300 ml A
Ratio = 5:3
Proportion of alcohol = 3/8
If 300 ml removed from second = 300 x 5/8 = 187.5 ml water
= 112.5 ml alcohol
Now poured so vessel = 200 + 112.5 = 312.5 A
= 187.5 ml
Hence ratio = 5:3 proportion of water = 3/8
So A= B

31-C

Let the measure of the fourth angle be x°

From the concept, the sum of the angles of a quadrilateral is 360°

110°+ 80°+95°+ x°= 360°

www.careers360.com Back to Index 343


x°= 75°

32-B

33-C
Let the thickness of the bottom be

34-A
Let max. Marks in each of these subjects be 100
so candidate scored aggregate of (60/100)* 300= 180 marks
Let marks scored be 4x, 5x & 6x
then 4x + 5x+6x = 180
x= 12
therefore marks scored by candidates are 48, 60 & 72
Hence candidate has scored more than 60% in 1 subject.

35-B
Given:
2 * 3 * 5 = 523
6 * 4 * 1 = 164
8 * 2 * 4 = 482

www.careers360.com Back to Index 344


Here, the numbers given in the equation are shuffled in a particular order, i.e., the third
number replaces the first number, the first number replaces the second number, and the
second number replaces the third number.
(2 * 3 * 5) = 523
(6 * 4 * 1) = 164
(8 * 2 * 4) = 482
Similarly, for (6 * 8 * 2) = 268

Hence, the second option is correct.

36-C
Given:
Line : Square :: Arc : ?

A square is a geometrical shape that has four equal sides and each side can be drawn
using a line.
Similarly, a circle is a geometrical shape that consists of a continuous curved line called
an arc.

Hence, the third option is correct.

READING COMPREHENSION

37-D
The fourth option is the correct answer to be filed in as 'to the best' means 'to the
greatest degree' fits contextually in the sentence and makes the meaning of the
sentence as 'he worked with the highest quality of his ability'. Other options don't fit
contextually because different prepositions used in the options make the options
incorrect.

Therefore, the correct statement should be "He worked to the best of his ability."

38-D
The fourth option is the correct answer. The word "aimlessly" is the most appropriate
choice in this context because it conveys that Purush was wandering without a clear
purpose or direction in life.

The other options, "madly," "excitedly," and "crazily," do not fit the context as well
because they suggest different emotions or states of mind.

So the correct complete sentence is, "Purush wandered aimlessly through life, with no
clear ideas of where he was heading and what he wanted to accomplish."

www.careers360.com Back to Index 345


39-A

The correct choice is the first option "by."

The complete sentence is a proverbial expression that conveys the idea that people are
often judged or assessed based on the friends and associates they choose to be
around.

The options ''for", ''in", and ''of'' don't fit contextually in the sentence.

Therefore, the complete sentence is "A man is known by the company he keeps".

40-A

The first option is the correct choice.

''Out of'' fits perfectly in the sentence, which means "from". It means that Sally parked
the car and quickly exited it.

The prepositions in the other options, ''in'', ''over'' and ''on'' are incorrect and should
not be used in the sentence.

The meanings of the other options are as follows:

● Got in: succeeded in entering a place, especially by using force or a trick.


● Got over: crossed or surmounted something.
● Got on: had a friendly relationship with somebody.

Therefore, the complete sentence is "Sally parked and got out of the car quickly".

41-B
The second option is the correct choice.

"Few'' is used to indicate a small number of something, often implying scarcity, and in
this case, "few" fits perfectly in the sentence, as the sentence has a negative
connotation to it.

The meanings of the other options are as follows:

● "A few" also means less in quantity, but is used in positive cases as it implies
sufficiency, which is not the intended meaning of the sentence.
● "Little" and "a little" are used for quatites that are not countable in nature and
hence are not correct to use in this sentence.

www.careers360.com Back to Index 346


Therefore, the complete sentence is "Anna is not popular, she has few friends".

42-B
The correct choice is the second option "hanged".

"Hanged" means the execution of a person by suspending them by the neck as a form
of punishment. It fits perfectly in the sentence conveying the intended meaning.

"Hung" is the past participle form of "hang" which means to suspend from above with
the lower end dangling free. Both of these options are incorrect to use in the sentence
as "hanged" is used specifically for the purpose discussed above.

Therefore, the complete sentence is "The convict was hanged on a Monday".

43-C
The third option is the correct answer since ''even though'' conveys the meaning that
despite the fact that Anita can't drive, she has bought a car, emphasising the contrast.

The option ''since'' in the sentence can mean ''because of"' ''even if'' is used to
introduce a hypothetical condition and ''whether or not'' is used to present alternatives,
all of which don't fit contextually.

Therefore, the complete sentence is "Even though she can't drive, Anita has bought a
car".

44-C
The correct choice to fill in the blank is the third option.

The simple present tense is followed by the simple future tense as per the first
conditional. It conveys that the success of the speaker's efforts is contingent upon the
future outcome.

Therefore, the complete sentence is, "If you do well in the exam next year, then my
efforts will be successful."

45-C
The correct choice is the third option.

"Need" implies a necessity or requirement. In this context, it suggests that there is no


need or requirement to hurry, which aligns with the idea that there is plenty of time.

The meanings of the other options are as follows:

Must: It denotes a strong obligation or necessity.

www.careers360.com Back to Index 347


Would: It is a modal verb often used to express a future possibility or a polite request.

Should: It is used to express a recommendation or advice.

Therefore, the complete sentence is, "We need not hurry; we have plenty of time."

46-C

The correct choice is the third option "was gaining."

This choice fits grammatically in the sentence and conveys the intended meaning that
they were worried about a new cult that was actively acquiring many followers.

The options ''is gaining'' and ''must be gaining'' are incorrect as they are not following
the tense in the sentence. ''Were gaining'' is incorrect as per subject-verb agreement.

Therefore, the complete sentence is "They were worried about a new cult that was
gaining many followers".

47-C

Devastate: To cause severe destruction or damage.


Devour: To consume something with great eagerness, often with greed.
Defalcate: To embezzle funds that have been entrusted to someone.
Dawdle: To waste time or move slowly, often aimlessly or without purpose.

Therefore, the correct sentence is Legislation was passed to punish brokers who
defalcate their client's funds.

48-B
The correct choice is the second option, had rung because the sentence is in the past
perfect tense, indicating that the action of ringing the warning bells occurred before
another past action (realising the danger). In the past perfect tense, had is followed by
the past participle of the verb, which is rung in this case.

Therefore, the correct sentence is, The warning bells had rung several times before
anyone realized the danger.

GENERAL KNOWLEDGE

49-D
The retirement age for High Court judges in India is determined by the respective state
legislatures.

www.careers360.com Back to Index 348


50-B
The Constitution of India allows for a maximum of 550 members in the House, with 530
members representing the States and 20 representing the Union Territories. At present,
the Lok Sabha has 543 seats filled by elected representatives.

51-C
The correct option is Stand Up India scheme

The Indian government launched the Stand Up India campaign to support women
entrepreneurs and people from the SC and ST communities. This program provides
bank loans to qualifying participants ranging from Rs.10 lahk to Rs. 1 crore to help them
start new enterprises. New enterprises or greenfield businesses are the target audience
for the Stand Up India loans. It indicates that it is for recently established enterprises.

52-B
The correct answer is Gram Uday se Bharat Uday Abhiyan.

The comprehensive scheme aims to promote social harmony and empower people in
villages by upgrading Panchayati Raj Institutions with basic amenities such as drinking
water, sanitation, etc. This nationwide program involves all stakeholders, including the
Central Government, State Governments, Panchayats, and the general public.

53-A
The correct answer is the Sukanya Samridhi Account.

Sukanya Samriddhi Account is a government-backed savings scheme designed to


encourage parents to save for their girl child's future education and marriage. The Beti
Bachao Beti Padhao initiative was spearheaded by Prime Minister Narendra Modi
beginning in 2015. It provides a high interest rate and tax benefits, and the account
matures after 21 years.

54-B
The correct answer is Pradhan Mantri Ujjwala Yojana.

The Pradhan Mantri Ujjwala Yojana (PMUY) is the scheme. Prime Minister Narendra
Modi started it in 2016 for the 50 million Indian households below the poverty line (BPL).
The PMUY offers a free LPG connection to the household's head, who must be an adult
woman. A stove and a regulator are included in the connection. The recipient also
receives a three-year supply of free LPG petrol refills.

www.careers360.com Back to Index 349


55-A
The correct option is a study of the scale of mobile phones by BSNL.

The study of the scale of mobile phones by BSNL is a microeconomic analysis that
focuses on the performance of a specific firm. The other choices in the question,
inflation, per capita income and the unemployment rate for women are macroeconomic
indicators that show how the economy is performing overall.

56-C
The correct answer is if the price of a good increases, the quantity demanded of
that good decreases.

According to the law of demand, there is an inverse connection between the price of
an item and the amount wanted. The rule of demand is based on the assumption that
customers have a limited budget and must make decisions about how to spend it. When
the price of a product rises, buyers must spend more money to purchase the same
amount of the product. As a result of having less money to spend on other things,
customers will desire less of the item whose price has soared.

57-D
The answer is Mughal Garden.

The Mughal Garden is located inside the Taj Mahal. It's a Persian-style garden with
paths or water channels dividing it into four equal sections. The Charbagh, a formal
garden featuring terraced flowerbeds, water channels, and pathways, is a key
component of the Taj Mahal's overall architectural design.

58-C
The answer is Jahangir.

Mehr-un-Nissa, known as Noor Jahan, was the wife of Mughal Emperor Jahangir. Her
influence was substantial in the 17th-century Mughal court, owing to her political
acumen and cultural patronage. Noor Jahan left an indelible mark with her contributions
to the arts and architecture of the Mughal Empire.

59-B
The correct option is Perfect competition.

● Perfect competition is an ideal type of market structure where all producers and
consumers have full and symmetric information and no transaction costs.

www.careers360.com Back to Index 350


● There are a large number of producers and consumers competing with one
another in this kind of environment.
● Perfect competition is theoretically the opposite of a monopolistic market.
● Since all real markets exist outside of the plane of the perfect competition model,
each can be classified as imperfect.
● The opposite of perfect competition is imperfect competition, which exists when a
market violates the abstract tenets of neoclassical pure or perfect competition.

60-B
The correct option is The Labour Bureau.

The Labour Bureau, under the Ministry of Labour and Employment, is responsible for
compiling and releasing the Consumer Price Index Number for Industrial Workers
(CPI-IW) every month. This index is derived from the fluctuations in prices of a selection
of goods and services that are typically used by industrial workers and their families.

www.careers360.com Back to Index 351


OTHER USEFUL RESOURCES : YOUR
GUIDE TO SUCCESS!

W
elcome to your guide to success in Law Entrances Exams. To facilitate your path to suc-
cess, we are excited to introduce our comprehensive eBook Series. These carefully curated
eBooks are designed to provide you with invaluable resources and insights to boost your
preparation. Let’s explore what each eBook in our series has to offer:

CUET Legal Studies Concepts:


The Cuet Legal Studies Concepts PDF provides comprehensive study material for legal studies with
chapters organized into individual concepts for easy understanding.

Download Now

MH CET Law ( 3 and 5 Years LLB):


Download the MHCET 10 free mock tests along with answer keys and detailed solutions.Alongwith
that, it also provides the exam pattern and preparation strategies for both the 3-year and 5-year law
programs.

3 Years LLB 5 Years LLB

Criminal Law Concepts and Practice Questions


Enhance your legal preparation with comprehensive Criminal Law concepts and practice questions
to master this crucial chapter.

Download Now

CLAT Comprehensive Guide Series:

CLAT Free Sample Paper:


Experience the actual CLAT exam with our complimentary sample paper.

Download Now

CLAT 20 Mock Tests with Solutions eBook:


Access a variety of mock tests along with detailed solutions to effectively track your progress.

Download Now

www.careers360.com Back to Index 352


CLAT Legal Reasoning Previous Year Question Papers with Detailed Solutions
Enhance your preparation with comprehensive insights by accessing Legal Reasoning Previous Year
Question Papers from 2020 to 2023, complete with proper answer keys and detailed solutions.

Download Now

100+ Quantitative Techniques Practice Questions:


Download this invaluable CLAT Quantitative Techniques resource by Careers360, featuring compre-
hensive questions, answers, and detailed solutions to master the subject.

Download Now

We extend our best wishes for your success in your preparations!


Keep striving for excellence, and success will undoubtedly be
within your reach.

With warm regards,


“Best Wishes.”

Team Careers360

www.careers360.com Back to Index 353

You might also like